Sunteți pe pagina 1din 275

The University of Hong Kong

Department of Physics

PHYS2250 Introductory Mechanics


Teaching notes

September 2018
Course title Introductory Mechanics

Aim This course covers the foundation of mechanics in one semester. It serves as
a core course for students who are planning to take physics, astronomy, or
mathematics/physics as major. It also serves students who intend to take
physics as minor. Both conceptual ideas and mathematical treatment in
mechanics are emphasized.

Contents Kinematics, Newton’s Laws of Motion and Their Applications, Linear Mo-
mentum and its Conservation, Variable Mass Problems, System of Parti-
cles and Centre of Mass, Torque and Rotation, Angular Momentum and
its Conservation, Work, Energy and its Conservation, Gravitation, Simple
Harmonic Motions, Damped and Driven Oscillations, Resonance, Energy in
Wave Motion, Interference and the Principle of Superposition.

Length One semester (The course is offered in the first semester and it is repeated
in the second semester.)

Assessment 50% from a two-hour written examination and 50% continuous assessment
(including laboratory works (15%), assignments (10%) and a 90-minute quiz
(25%)).

Textbooks P. A. Tipler and G. Mosca: Physics for Scientists and Engineers (Freeman,
6th edition).

References R. D. Knight: Physics for Scientists and Engineers (Pearson, 2008).


D. Kleppner and Robert J. Kolenkow: An Introduction to Mechanics (Mc-
Graw Hill, 1978, International Edition).
R. Resnick, D. Halliday and K. Krane: Physics Volume 1 (John Wiley and
Sons, 2002).
R. Serway and J. W. Jewett: Physics for Scientists and Engineers (Thom-
son, 2014, 9th edition).
T. Duncan: Advanced Physics (John Murray Publishers, 2000, 5th edition).
Atam P. Arya, Introduction to Classical Mechanics (Prentice Hall).
Contents

Table of Contents i

1 Dimensional Analysis and Vectors 1


1.1 Dimensions of physical quantities . . . . . . . . . . . . . . . . . . . . . . . 1
1.2 Dimensional analysis and empirical formula . . . . . . . . . . . . . . . . . 1
1.3 Vectors . . . . . . . . . . . . . . . . . . . . . . . . . . . . . . . . . . . . . . 3
1.4 Adding vectors . . . . . . . . . . . . . . . . . . . . . . . . . . . . . . . . . 4
1.5 Components of vectors . . . . . . . . . . . . . . . . . . . . . . . . . . . . . 6
1.6 Subtracting vectors . . . . . . . . . . . . . . . . . . . . . . . . . . . . . . . 8
1.7 Solving extreme value problems with vectors . . . . . . . . . . . . . . . . . 11

2 Kinematics 14
2.1 Position, velocity and acceleration vectors . . . . . . . . . . . . . . . . . . 14
2.2 One dimensional motion . . . . . . . . . . . . . . . . . . . . . . . . . . . . 19
2.3 Non-constant acceleration . . . . . . . . . . . . . . . . . . . . . . . . . . . 26
2.4 Projectile motion . . . . . . . . . . . . . . . . . . . . . . . . . . . . . . . . 29
2.5 Projectile on an inclined plane . . . . . . . . . . . . . . . . . . . . . . . . . 33

3 Newton’s Law of Motion 39


3.1 Newton’s first law . . . . . . . . . . . . . . . . . . . . . . . . . . . . . . . . 39
3.2 Reference frame and relative motion . . . . . . . . . . . . . . . . . . . . . . 40
3.3 Newton’s second law . . . . . . . . . . . . . . . . . . . . . . . . . . . . . . 42
3.4 Newton’s third law . . . . . . . . . . . . . . . . . . . . . . . . . . . . . . . 44
3.5 Application of Newton’s law in one dimension . . . . . . . . . . . . . . . . 44
3.5.1 Pushing a packing crate . . . . . . . . . . . . . . . . . . . . . . . . 44
3.5.2 Mass hanging in a lift . . . . . . . . . . . . . . . . . . . . . . . . . 45
3.6 Tension and normal force . . . . . . . . . . . . . . . . . . . . . . . . . . . . 49

i
3.7 Pulley system . . . . . . . . . . . . . . . . . . . . . . . . . . . . . . . . . . 50

4 Motion in a Plane 58
4.1 Uniform circular motion . . . . . . . . . . . . . . . . . . . . . . . . . . . . 58
4.2 Polar coordinate system . . . . . . . . . . . . . . . . . . . . . . . . . . . . 62

5 Application of Newton’s Law 70


5.1 Frictional force . . . . . . . . . . . . . . . . . . . . . . . . . . . . . . . . . 70
5.1.1 Static friction . . . . . . . . . . . . . . . . . . . . . . . . . . . . . . 70
5.1.2 Kinetic friction . . . . . . . . . . . . . . . . . . . . . . . . . . . . . 71
5.2 A Revisit to circular motion . . . . . . . . . . . . . . . . . . . . . . . . . . 73
5.3 Non-inertial observer and fictitious force . . . . . . . . . . . . . . . . . . . 81

6 Impulse and Momentum 87


6.1 Definition of linear momentum . . . . . . . . . . . . . . . . . . . . . . . . . 87
6.2 Collision between bodies . . . . . . . . . . . . . . . . . . . . . . . . . . . . 87
6.3 Impulse and momentum . . . . . . . . . . . . . . . . . . . . . . . . . . . . 88
6.4 Conservation of linear momentum . . . . . . . . . . . . . . . . . . . . . . . 92
6.5 Collision of particles in two dimensional space . . . . . . . . . . . . . . . . 94
6.6 Coefficient of restitution (e) . . . . . . . . . . . . . . . . . . . . . . . . . . 98

7 Systems of Particles 100


7.1 Center of mass . . . . . . . . . . . . . . . . . . . . . . . . . . . . . . . . . 100
7.2 Motions of the Center of mass . . . . . . . . . . . . . . . . . . . . . . . . . 102
7.3 Center of mass of some rigid bodies . . . . . . . . . . . . . . . . . . . . . . 105
7.4 Momentum of system of particles . . . . . . . . . . . . . . . . . . . . . . . 119
7.5 System of variable mass . . . . . . . . . . . . . . . . . . . . . . . . . . . . 121

8 Rotational Kinematics 131


8.1 Rotation with constant angular acceleration . . . . . . . . . . . . . . . . . 133
8.2 Relation between linear and angular variables . . . . . . . . . . . . . . . . 134

9 Rotational Dynamics 136


9.1 Torque . . . . . . . . . . . . . . . . . . . . . . . . . . . . . . . . . . . . . . 136
9.2 Rotational inertia . . . . . . . . . . . . . . . . . . . . . . . . . . . . . . . . 138
9.3 Parallel axis theorem . . . . . . . . . . . . . . . . . . . . . . . . . . . . . . 144
9.4 Rotational inertia of solid bodies . . . . . . . . . . . . . . . . . . . . . . . 146

ii
9.5 Perpendicular axis theorem . . . . . . . . . . . . . . . . . . . . . . . . . . 152
9.6 Equilibrium of rigid body . . . . . . . . . . . . . . . . . . . . . . . . . . . 155
9.7 Non-equilibrium situation: pure rotation . . . . . . . . . . . . . . . . . . . 159
9.8 Non-equilibrium situation: rotational and translational motion . . . . . . . 161

10 Angular Momentum 174


10.1 Definition . . . . . . . . . . . . . . . . . . . . . . . . . . . . . . . . . . . . 174
10.2 Angular momentum and angular velocity . . . . . . . . . . . . . . . . . . . 176
10.3 Conservation of angular momentum . . . . . . . . . . . . . . . . . . . . . . 178
10.4 Stability of spinning object . . . . . . . . . . . . . . . . . . . . . . . . . . . 179

11 Work, Kinetic Energy and Potential Energy 188


11.1 Work done by a constant force . . . . . . . . . . . . . . . . . . . . . . . . . 188
11.2 Work done by a variable force . . . . . . . . . . . . . . . . . . . . . . . . . 189
11.2.1 One dimensional case . . . . . . . . . . . . . . . . . . . . . . . . . . 189
11.2.2 Two dimensional case . . . . . . . . . . . . . . . . . . . . . . . . . . 191
11.3 Work-energy theorem . . . . . . . . . . . . . . . . . . . . . . . . . . . . . . 192
11.4 Work done and kinetic energy in rotational motion . . . . . . . . . . . . . 193
11.5 A combination of rotational and translational motions . . . . . . . . . . . . 194
11.6 Kinetic energy in collision . . . . . . . . . . . . . . . . . . . . . . . . . . . 195
11.7 Conservative force . . . . . . . . . . . . . . . . . . . . . . . . . . . . . . . . 195
11.8 Potential energy . . . . . . . . . . . . . . . . . . . . . . . . . . . . . . . . . 197
11.9 Conservation of mechanical energy . . . . . . . . . . . . . . . . . . . . . . 198
11.10One dimensional conservative system . . . . . . . . . . . . . . . . . . . . . 199

12 Conservation of Energy 201


12.1 Internal energy in a system of particle . . . . . . . . . . . . . . . . . . . . 203
12.2 Some examples of conservation of energy . . . . . . . . . . . . . . . . . . . 204

13 Gravitation 217
13.1 Newton’s law of universal gravitation . . . . . . . . . . . . . . . . . . . . . 217
13.2 Gravitational field . . . . . . . . . . . . . . . . . . . . . . . . . . . . . . . . 218
13.3 Gravitational force due to a spherical shell . . . . . . . . . . . . . . . . . . 219
13.4 Gravitational field due to the earth . . . . . . . . . . . . . . . . . . . . . . 220
13.5 Effect of earth’s rotation . . . . . . . . . . . . . . . . . . . . . . . . . . . . 221
13.6 Gravitational potential energy . . . . . . . . . . . . . . . . . . . . . . . . . 223

iii
13.7 Potential energy of many-particle system . . . . . . . . . . . . . . . . . . . 225
13.8 Energy consideration of satellite motion . . . . . . . . . . . . . . . . . . . . 226

14 Oscillations 227
14.1 Simple harmonic motion . . . . . . . . . . . . . . . . . . . . . . . . . . . . 227
14.2 Time varying quantities in SHM . . . . . . . . . . . . . . . . . . . . . . . . 229
14.3 Phase diagram . . . . . . . . . . . . . . . . . . . . . . . . . . . . . . . . . 233
14.4 Potential energy and kinetic energy in a spring-mass system . . . . . . . . 234
14.5 Second order differential equations . . . . . . . . . . . . . . . . . . . . . . 239
14.6 Damped oscillation . . . . . . . . . . . . . . . . . . . . . . . . . . . . . . . 241
14.7 Driven oscillation . . . . . . . . . . . . . . . . . . . . . . . . . . . . . . . . 244
14.8 Resonance phenomena . . . . . . . . . . . . . . . . . . . . . . . . . . . . . 246

15 Wave Motions 248


15.1 Mechanical waves . . . . . . . . . . . . . . . . . . . . . . . . . . . . . . . . 248
15.2 Sinusoidal waves . . . . . . . . . . . . . . . . . . . . . . . . . . . . . . . . 249
15.3 Wave speed on a stretched string . . . . . . . . . . . . . . . . . . . . . . . 252
15.4 Derivation of wave equation . . . . . . . . . . . . . . . . . . . . . . . . . . 253
15.5 The rate of energy transmission . . . . . . . . . . . . . . . . . . . . . . . . 254
15.6 Reflection and transmission . . . . . . . . . . . . . . . . . . . . . . . . . . 255
15.7 Doppler effect . . . . . . . . . . . . . . . . . . . . . . . . . . . . . . . . . . 258
15.8 Shock waves . . . . . . . . . . . . . . . . . . . . . . . . . . . . . . . . . . . 261
15.9 Superposition . . . . . . . . . . . . . . . . . . . . . . . . . . . . . . . . . . 261
15.10Standing wave . . . . . . . . . . . . . . . . . . . . . . . . . . . . . . . . . . 264
15.11Beats . . . . . . . . . . . . . . . . . . . . . . . . . . . . . . . . . . . . . . . 267

iv
Chapter 1

Dimensional Analysis and Vectors

1.1 Dimensions of physical quantities


All the physical quantities used in mechanics are originated from the three basic dimen-
sions:

Dimensions SI units
Mass kg
Length m
Time s

For example: frequency [T−1 ], velocity [L T−1 ],


acceleration [L T−2 ], force [M L T−2 ],
energy [M L2 T−2 ], momentum [M L T−1 ],
mass density [M L−3 ], moment of inertia [M L2 ]
angular velocity [T−1 ] torque [M L2 T−2 ]

1.2 Dimensional analysis and empirical formula


For any equation to be held, dimension on the right hand side equals to that of left hand
side. Dimensional analysis is a simple and direct method to obtain the physical equation.
The assumption is that the physical quantities relate each other by the power law. The
equation so obtained is called the empirical formula. It is not the final formula because
the proportional constant of the equation has to be determined by experiments.

1
Chapter 1 Dimensional Analysis and Vectors 2

Example
We know that a force is required to keep an object in circular motion. Suppose that the
magnitude of force depends on the object mass, speed and the radius of the motion. Find
the empirical formula to relate these quantities.

Solution

F ∝ ma v b rc

[F ] = [ma ] [v b ] [rc ]
[M L T−2 ] = [M]a [L T−1 ]b [L]c
= [M]a [L]b+c [T]−b

mv 2
∴ a = 1, b = 2, c = −1, and F ∝ .
r
Example
The frequency of vibration of an object of mass m at the end of a spring of force constant
k is given by f , where f ∝ ma k b . Use dimensional analysis to find a and b. It is known
that [f ] = [T]−1 .

Solution
Consider the dimension of force constant k first. According to the Hooke’s law, the
magnitude of restoring force of a spring F relates to the extension x by F = kx. Using
dimensional analysis, [F ] = [M] [L] [T]−2 and [x] = [L]. Therefore,

[F ] [M] [L] [T]−2


[k] = = = [M] [T]−2
[x] [L]

Assume that the frequency f relates m and k by the power law. We have

f ∝ ma k b ⇒ [T]−1 = [M]a [MT−2 ]b = [Ma+b T−2b ]


∴ a + b = 0 and 2b = 1
1
⇒ a = −b = −
2
r
k
Hence, we obtain the relation f ∝ .
m
Chapter 1 Dimensional Analysis and Vectors 3

Example
The wave speed on a stretched string is believed to relate the linear density and tension
of a string respectively. Find an empirical formula of the wave speed v to relate µ and F .

Solution
Assume that v ∝ µα F β , where the proportional constant is dimensionless. We consider
the dimension of quantities on both sides.

[v]=[L] [T]−1 and [µ]=[M] [L]−1 , [F ]=[M] [L] [T]−2

We have [µ]α [F ]β =[M]α+β [L]−α+β [T]−2β . Therefore, we have αs


+ β = 0, −α + β = 1 and
F
−2β = −1. The solutions are α = −1/2 and β = 1/2. i.e. v ∝ .
µ

1.3 Vectors
A vector is a quantity which has both magnitude (length) and direction. The vector ~v has
magnitude denoted as |~v | or simply v. The position of the vector in space is immaterial,
as shown in the figure. Vectors having the same direction and same length, originated
~ B
from different points in the space are identical. Three vectors A, ~ and C
~ show in the
figure are equivalent.

The properties of vectors are listed as follows.

• Scalar Multiplication:
For any real number λ, the scalar product of λ and ~v is λ~v which is a vector having
its length |λ| times as long as ~v . We call λ a scalar in contrast to the vector ~v .

1. If λ = 1, the product is 1~v = ~v .


2. If λ = 0, we have 0~v = ~0, the null vector or zero vector. The magnitude of it
is zero.
Chapter 1 Dimensional Analysis and Vectors 4

3. If λ = −1, the product is −1~v = −~v , the direction of it is opposite to ~v but


having the same length as ~v . We read −~v as minus ~v .
4. If λ = 1/|~v |, we obtain the unit vector v̂ = ~v /|~v | which points in the direction
of ~v but the magnitude of it is 1.

• Addition and Subtraction:

~+B
1. A ~ =B
~ +A
~ (commutative law)
~ + (−B)
2. A ~ =A
~−B
~
~ + (B
3. A ~ + C)
~ = (A
~ + B)
~ +C
~ (associative law)
~ = (αβ)A
4. α(β A) ~ (associative law)
~ + B)
5. α(A ~ = αA
~ + αB
~ (distributive law)
~ = αA
6. (α + β)A ~ + βA
~ (distributive law)

1.4 Adding vectors


An object acting by two forces F~1 and F~2 is equivalent to being acted by F~3 , where
F~3 = F~1 + F~2 . The sum of these vectors gives the resultant of the applied forces. The
direction and magnitude of F~3 can be obtained by constructing a parallelogram as shown
in figure.

Notice that the vectors can be translated as shown in the left diagram of the figure below.
Then we obtain two vector diagrams in the form of a triangle, e.g. F~1 + F~2 = F~3 in the
middle diagram and F~2 + F~1 = F~3 in the right diagram. Both diagrams give the same
Chapter 1 Dimensional Analysis and Vectors 5

result for F~3 . The results can be extended to a many-force system and the vector diagram
becomes a polygon.
If we label F~1 , F~2 , and −F~3 by A,
~ B,
~ and C
~ respectively, we have A
~ +B
~ +C
~ = 0. In other
words, when the sum of three vectors equals zero, the vector diagram is a triangle with
the vectors pointing in well order (i.e. it is in a cyclic way, clockwise or counterclockwise).
The vector diagrams are equivalent as shown in the figure below. If an object is at
equilibrium due to three forces, the force diagram is a triangle with the force vectors
pointing in well order.

Example
A particle of mass 2 kg is suspended under the ceiling when it is acted by a horizontal
force F . If the tension in the string is 40 N and the angle made by the string and the
vertical is θ. Find F and θ.

Solution
Let m be the mass of the particle and T be the tension in the string. Since the particle is
at equilibrium, the force vectors: m~g , T~ , and F~ form a right-angled triangle which gives
Chapter 1 Dimensional Analysis and Vectors 6

 mg 
θ = cos−1
p
F = T 2 − m2 g 2 and .
T
Putting m = 2 kg, g = 9.8 ms−2 , and T = 40 N, we have F = 34.9 N and θ = 60.7◦ .

1.5 Components of vectors


~ = Ax î + Ay ĵ, the horizontal and vertical components of it is represented
Given that A
~ = A2x + A2y . If B~ = Bx î + By ĵ, we
p
by Ax = A cos α and Ay = A sin α, where A = |A|
~ =A
have C ~+B
~ = (Ax + Bx ) î + (Ay + By ) ĵ, where Bx = B cos β, By = B sin β and
~ = Bx + By2 . The figure below shows the components of A~ and B~ along the î
p 2
B = |B|
~ =A
and ĵ directions. The resultant vector C ~ +B
~ is shown in the figure. It is the diagonal
of the parallelogram formed by A ~ and B.
~

Example
1
The motion of a particle is described by x = R cos2 ωt and y = R sin 2 ωt, where R and
2
ω are constants.

(a) Find the acceleration of the particle along with its locus.

(b) Find the acceleration of the particle normal to its locus.


Chapter 1 Dimensional Analysis and Vectors 7

Solution
(a) Method 1: By direct differentiation

 x = R cos2 ωt
 y = 1 R sin 2 ωt
2

~v = ẋ î + ẏ ĵ
= −2 ωR cos ωt sin ωt î + ωR cos 2 ωt ĵ
= −ωR sin 2 ωt î + ωR cos 2 ωt ĵ
= ωR (− sin 2 ωt î + cos 2 ωt ĵ)

~a = ~v˙
= ẍ î + ÿ ĵ
= −2 ω 2 R (cos 2 ωt î + sin 2 ωt ĵ)

Since ~v · ~a = 0, we have ~v ⊥ ~a.


Denote ~aT as the acceleration along the locus of the particle (i.e. ~aT // ~v ). We have ~aT = 0.

(b) Denote ~aN as the acceleration normal to the locus of the particle (i.e. ~aN ⊥ ~v and
~a = ~aT + ~aN ).
We have ~aN = ~a because ~aT = 0 thus
1
|~aN | = 2 ω 2 R (cos2 2 ωt + sin2 2 ωt) 2 = 2 ω 2 R

Again, we have ~aN = −2 ω 2 R (cos 2 ωt î + sin 2 ωt ĵ).

Method 2: A graphical method



 x = R cos2 ωt = R (1 + cos 2 ωt)

2
R
 y=
 sin 2 ωt
2
The particle performs a uniform circular motion with an angular speed 2ω. The locus
R R
of it has a radius and it’s center is located at ( , 0). The tangential acceleration
2 2
of the particleis ~
aT = 0. One can see that the centripetal acceleration has magnitude
R
|~aN | = (2 ω)2 = 2 ω 2 R and its direction points along − cos 2 ωt î−sin 2 ωt ĵ. Finally,
2
we get ~aN = −2 ω 2 R (cos 2 ωt î + sin 2 ωt ĵ).
Chapter 1 Dimensional Analysis and Vectors 8

1.6 Subtracting vectors


~−B
We can construct A ~ by adding A ~ and −B.
~ The left figure shows the way to obtain
~ B.
A− ~ We produce −B ~ first and form the parallelogram using A
~ and vecB. The resultant
~ =A
vector is D ~ − B.
~ It is along the diagonal of the parallelogram. The left figure shows
~ without constructing the parallelogram.
D

Two objects A and B are located at different place on the 2d-plane. The position vectors
of them are ~rA and ~rB respectively. The position of object A relative to object B is given
by ~rAB = ~rA − ~rB , the left diagram in the figure below. The direction and magnitude of
~rAB provide information about the position of A relative to B. The meaning of relative
position vector is straight forward by considering the following cases. Suppose that there
is an observer located at B and he tries to state the position of A. He will say ~rAB . If
Chapter 1 Dimensional Analysis and Vectors 9

the observer is located at A and he tries to state the position of B. Then, he will say
~rBA , where ~rBA = ~rB − ~rA . The right diagram in the figure below shows the direction of
~rBA . The concept can be extended to relative velocity. The velocity of A relative to B
is ~vAB = ~vA − ~vB , where ~vA and ~vB are velocity vectors of A and B respectively.

Example
A truck has a h = 2 m rear door. When it stops on the road with the rear door opened,
the rain drops can fall into the truck up to d = 1 m from the rear door (as shown in
figure). The rain drops no longer can fall into the truck once the truck increases its speed
to 15 km h−1 while moving on a level road. Find the velocity of the rain drops (relative
to the ground).

Solution
Because of the relativeness of the velocity, we have

~vrain to car = ~vrain to ground − ~vcar to ground (1.1)

Simply, we would say


~vrain to car = ~vrain − ~vcar ,

where ~vrain = ~vrain to ground and ~vcar = ~vcar to ground . Since the rain cannot fall into
the car when the car increase its velocity to 15 km hr−1 relative to the ground, ~vrain to car
should point vertically. Then, the three velocity vectors will have the relation as shown
in the figure.
Chapter 1 Dimensional Analysis and Vectors 10

The angle α is given by


   
−1 h −1 2m
α = tan = tan = 63.4◦
d 1m

Using eq. (1.1), we obtain the speed of the rain drops

vcar 15 km h−1
vrain = = ◦
= 33.5 km h−1
cos α cos 63.4
Example
A ship A, which can sail at a constant speed 60 km/hr to meet a second ship B which is
100 km away in the direction of S 60◦ W and is sailing due east at constant speed 30 km/hr.
Find the sailing direction of A and the time required to meet B.

Solution

We subtract ~vB from the two ships and then construct the vector diagram. Denote vA and
vB as the magnitude of ~vA and ~vB respectively. Note that the velocity of ship A relative
to ship B is ~vAB = ~vA − ~vB and vAB = |~vA − ~vB |. Applying Sine rule to the triangle
Chapter 1 Dimensional Analysis and Vectors 11

formed by ~vA , −~vB , and ~vAB , we get


vA vB

=
sin 30 sin θ
60 30

=
sin 30 sin θ
1
sin θ =
4
θ = 14.5◦

So the direction of ~vA is S 45.5◦ W (i.e. 60◦ − 14.5◦ = 45.5◦ ).


Next, vAB = vA cos θ + vB cos 30◦ = 60 cos 14.5◦ + 30 cos 30◦ = 84.1 km/hr
100 100
The required time is = = 1.19 hr.
vAB 84.1

1.7 Solving extreme value problems with vectors


Example
Two straight roads cross at right angles at O. A cyclist is travelling at 5 m/s due north.
When passing through O, he notices a motorcar on the other road 200 m from O. The
motorcar is travelling at 12 m/s due east towards O. Find the velocity of the motorcar
relative to the cyclist, the least distance between the cyclist and the motorcar and the
time required for it to occur.

Solution
Velocity of cyclist: ~vc = 5 ĵ
Velocity of motorcar: ~vm = 12 î
Velocity of motorcar relative to the cyclist: ~vmc = ~vm − ~vc = 12 î − 5 ĵ

2 2
The speedof the motorcar relative to the cyclist is |~vmc | = 12 + 5 = 13 m/s. As
5
θ = tan−1 = 22.6◦ , the motorcar travels S 67.4◦ E (90◦ − 22.6◦ = 67.4◦ ) relative to
12
the cyclist.
Chapter 1 Dimensional Analysis and Vectors 12

In order to find the shortest distance between the cyclist and the motorcar, we construct
the vector diagram as shown below. We subtract ~vc from both of them, then the cyclist
does not move and the motorcar moves with 12 î − 5 ĵ relative to the cyclist.

Let the shortest distance between them be d. It is the normal distance from O to the
path along ~vmc , then

d = 200 sin θ = 200 sin 22.6◦ = 76.9 m.


200 cos θ 200 cos 22.6◦
The time required is = = 14.2 seconds.
|~vmc | 13

Example
A ship A is 5 km due north of a ship B. A is streaming due west at 15 km/hr and B is
streaming due north-west at 10 km/hr. Find the distance and the time of their nearest
approach to each other.

Solution
Chapter 1 Dimensional Analysis and Vectors 13

~vAB = ~vA − ~vB


= (−15 î) − (−10 cos 45◦ î + 10 sin 45◦ ĵ)
10 10
= (−15 î) − ( √ î + √ ĵ)
2 2
10 10
= (−15 + √ ) î − √ ĵ
2 2
~vAB = −7.93 î − 7.07 ĵ

|~vAB,x | 7.93
Therefore, tan θ = = , i.e. θ = 48.28◦ . From the right figure, we observe that
|~vAB,y | 7.01
the shortest distance between A and B is d, where d = 5 sin 48.28◦ = 3.73 km. Hence,
5 cos 48.28◦
t= = 0.31 hr,
|~vAB |

where |~vAB | = 7.932 + 7.072 km/hr = 10.62 km/hr.
Chapter 2

Kinematics

2.1 Position, velocity and acceleration vectors


Position vector
It is a vector to describe the position of a particle at time t. Generally, it is a vector
function of t. It is a constant vector if the particle is stationary.

y
~r(t) = rx (t)î + ry (t)ĵ
∆r ∆~r = ~r(t2 ) − ~r(t1 )

r(t1) displacement
vector
r(t2 )
x
Velocity vector
Average velocity in time period t1 to t2 :

∆~r ~r(t2 ) − ~r(t1 )


~vave = =
∆t t2 − t1
Instantaneous velocity at time t:
If the time interval ∆t is infinitesimal small, i.e. ∆t → 0, the instantaneous velocity at
time t is obtained.
∆~r d~r(t)
~vinst = lim = = ~r˙ (tangential to curve ~r(t))
∆t→0 ∆t dt

14
Chapter 2 Kinematics 15

Acceleration vector
Likewise, the average acceleration in time period t1 to t2 :
∆~v ~v (t2 ) − ~v (t1 )
~aave = =
∆t t2 − t1
∆~v d~v (t) d2~r(t) ¨
And, the instantaneous acceleration ~ainst = lim = = = ~r
∆t→0 ∆t dt dt2

When we talk about velocity or acceleration, we are usually referring to the instantaneous
velocity and instantaneous acceleration.

Example
An object performs a uniform circular motion of radius r and speed v. Describe the
dr d~r dv d~v
meanings of , , , and .
dt dt dt dt
Solution
dr
= 0 , because the radius of rotation is a constant.
dt
d~r
= ~v , it is the instantaneous velocity of the object. It is a vector which points tangen-
dt
tial to the path of motion.

dv
= 0 , because v is a constant.
dt
d~v
= ~a , it is the centripetal acceleration of the object. It is a vector pointing toward the
dt
center of rotation.

Example
A car is travelling along a straight road and is decelerating. Does the car’s acceleration a
necessarily have a negative value?

Solution
We begin with the meaning of the term “decelerating” which has nothing to do with
whether the acceleration a is positive or negative. The term means only that the ac-
celeration vectors points opposite to the velocity vector and indicates that the moving
object is slowing down. When a moving object slows down, its instantaneous speed (the
magnitude of the instantaneous velocity) decreases. One possibility is that the velocity
Chapter 2 Kinematics 16

vector of the car points to the right, in the positive direction, as shown in figure (a). The
term “decelerating” implies that the acceleration vector points to the left, which is the
negative direction. Here, the value of the acceleration a would indeed be negative.

However, there is another possibility. The car could be travelling to the left, as shown in
figure (b). Now, since the velocity vector points to the left, the acceleration vector would
point to the right, according to the meaning of “decelerating”. But right is the positive
direction, so the acceleration a would have a positive value in figure (b). We see, then,
that a decelerating object does not necessarily have a negative acceleration.

Example
d~v
Describe the motion of an object if its velocity satisfies the relation = 0. How about
dt
d|~v |
the case if its velocity satisfies the relation = 0?
dt

Solution
d~v d~v
Since the acceleration ~a = , so = 0 means the magnitude of the acceleration is
dt dt
zero, i.e. its velocity does not
change
in both magnitude and direction as time passes.
d~v
So, if a car moves such that = 0, then it is either moving in a straight line with a
dt
constant speed or at rest.
d|~v |
But if the car now moves under the constraint that = 0, then only the magnitude
dt
of the velocity will keep constant. Although the car moves with constant speed, it may
change its direction of motion.
Chapter 2 Kinematics 17

Example
The projectile motion:

Some vectorial representations of physical quantities are shown below.

~a(t) = −g ĵ
~v0 = v0 cos θ î + v0 sin θ ĵ
~v (t) = (v0 cos θ) î + [(v0 sin θ) − gt] ĵ
1
~r(t) = (v0 cos θ)t î + [(v0 sin θ)t − gt2 ] ĵ
2

Example
A river of width L is streaming due north. Along the middle line of the river, the stream
has a constant speed u0 , while at both banks it is stationary.
 2 The speed at any point
L
in the river varies and has the relation u = u0 + k x − , where x is the perpen-
2
dicular distance measured from the west bank and k is a constant. A boat starts from
the west bank and it moves due north east with speed v0 . Find the locus of the boat
and the location of it when it reaches the east bank. You may consider a coordinate sys-
tem where the origin is the starting point of the boat and the y-axis is along the west bank.

Solution
Chapter 2 Kinematics 18

Given that u = u0 + k(x − L/2)2 .


As x = 0, u = 0, we have 0 = u0 + k(L2 /4). i.e. k = −4u0 /L2 .
 2
4u0 L 4u0 x  x
Hence, we have u = u0 − 2 x − , which gives u = 1− .
L 2 L L
 
v0 v0
Let ~v be the velocity of the boat relative to the bank. i.e. ~v = √ î + √ + u ĵ.
Z t Z L   Z L  2 2
dt vy v0
We have, y = vy dt = vy dx = dx, where vx = √ and
0 0 dx 0 vx 2
v0 4u0 x x
vy = √ + (1 − ). That is
2 L L

√ Z L 
2 v0 4u0 x x
y = √ + (1 − ) dx
v0 0 2 L L
√ √
2 2u0 2 4 2u0 3
y = x+ x − x
v0 L 3v0 L2
√ √ !
2 2u0 4 2u0
⇒ y |x=L = L 1 + −
v0 3v0
Z t Z t 
v0 v0
Alternatively, we have, x = vx t = √ t and y = vy dt = √ + u dt.
2 0 0 2
Z t 
v0 4u0 x  x
∴ y = √ + 1− dt
0 2 L L
Z t  
v0 4u0 v0 v0
= √ + √ t 1− √ t dt
0 2 2L 2L
Z t
2u0 v02 2

v0 4u0 v0
= √ + √ t− t dt
0 2 2L L2

v0 2u0 v0 2 2u0 v02 3
= √ t+ t − t
2 L 3L2
√ √
2 2u0 2 4 2u0 3
= x+ x − x
v0 L 3v0 L2
When x = L,
√ √
2 2u0 4 2u0
y = L+ L− L
v0 3v0
√ √ !
2 2u0 4 2u0
∴ y = L 1+ −
v0 3v0
Chapter 2 Kinematics 19

2.2 One dimensional motion


A stone is thrown from the ground towards the sky with vertical speed v0 .

just before
way max. way touching
y t=0 up height down ground
v=0

a=g a=g a=g a=g a=g


v<v0
v=v0 ymax
y v<v0 y
0
v=v0

decelerate accelerate

Take upward as positive y direction.


dv dv d2 y dy
a= = −g (a = = 2 , where v = )
Zdt Z dt dt dt
v= a dt = − g dt = −gt + A, A = constant

To determine A, substitute the initial condition at t = 0,




v = A = v0
t=0
∴ v = v0 − gt (2.1)

Alternatively, one can apply definite integral.


Z v Z t
dv
= −g ⇒ dv = − g dt ⇒ v − v0 = −gt ⇒ v = v0 − gt
dt v0 0

dy
v = = v0 − gt (2.2)
Zdt
1
⇒ y = (v0 − gt) dt = vo t − gt2 + B, B = constant
2
To determine B, substitute t = 0 again,

y(t = 0) = B = 0
1
∴ y = v0 t − gt2 (2.3)
2
Chapter 2 Kinematics 20


dy v0
At maximum height, = 0, i.e. v0 − gtmax = 0 ⇒ tmax = .
dt t=tmax
g
 2
v2
 
v0 1 v0
∴ ymax = v0 − g = 0
g 2 g 2g
Time for the stone to hit the ground, say t0 ,
1 2v0
y = v0 t0 − gt20 = 0 ⇒ t0 = 0 or t0 =
2 | {z } g
initial time
a

t
−g

v0

t
v0

y
v02
2g
v0 t− 1 gt 2
2
t
v0 2v0
g g
Chapter 2 Kinematics 21

Example
A screw at the ceiling of an elevator falls down while the elevator is moving upward with
an acceleration a. The height of the elevator is h and the gravitational acceleration is g.
Find the time when the screw just touches the floor of the elevator. Evaluate the time
when a = 1.22 ms−2 and h = 2.74 m.

Solution

1
The screw: y − h = ut − gt2
2
1
The elevator: y − 0 = ut + at2
2
Eliminating y by subtraction of the above equations, we have
1
h = (a + g) t2
2
s
2h
t =
a+g
s
2(2.74 m)
When a = 1.22 ms−2 , t = = 0.705 s.
(1.22 + 9.80) ms−2

Example
Mass B has a downward velocity in meters per second given by vB = t2 /2 + t3 /6, where
t is in seconds. Calculate the acceleration of A in terms of t.
Chapter 2 Kinematics 22

Solution

Let the total length of the string be l. We


have

4 l1 + 2 l2 + constant = l ,

l˙2
which gives 4 l˙1 + 2 l˙2 = 0 or l˙1 = − ,
2
vB
i.e. vA = − .
2
1 t2 t3 t2 t3

∴ vA = − + =− −
2 2 6 4 12
The velocity of A is negative as the length
l1 decreases with time when A is moving up.
t t2
∴ aA = − − . Here, aA is negative,
2 4
because mass A is accelerating upward. The
t t2
magnitude of it is aA = + .
2 4
Example
A boy of height h is standing under a light pole of height H. He moves away from the
pole along a horizontal and straight path with a constant speed v. Find the rate of change
of the length of his shadow.

Solution
Let the coordinates of boy and one end of his shadow be x and x0 respectively. Since
x0 /H = (x0 − x)/h, we have x0 = (xH)/(H − h). Note that l = x0 − x, which gives
Chapter 2 Kinematics 23

dl dx0 dx
= −
dt dt dt
Therefore, the rate of change of length of
shadow
 
dl d xH dx
= −
dt dt H − h dt
Hv
= −v
H −h
hv
=
H −h
where v = dx/dt.

Example
A rocket at rest is launched to move upwards. At the first 30 seconds, the rocket moves
with an upward acceleration of 18 ms−2 . Then the rocket’s engine shuts down. As a
result, the rocket keeps on moving upward for a while and then falls back to the ground.

(a) Calculate the maximum height that the rocket can reach.

(b) Calculate the total time of flight of the rocket.

Solution
After the lift-off, the motion of the rocket can be divided into the two parts: 1) moving
with upward acceleration 18 ms−2 during the first 30 seconds, 2) free-fall to the ground
with downward acceleration g = 9.8 ms−2 after its engine is shut down. (Note that we
have taken upward as positive direction and neglect the air resistance.)

(a) Suppose the rocket reach the maximum height H after shutting down its engine for
time t2 . Then the height H is given by
1 1
H = at21 + v1 t2 − gt22
2 2
where a = 18 ms−2 , t1 = 30 s is the time for the upward acceleration and the velocity
of the rocket is v1 (at the instant that the engine shuts down).
Besides, the velocity v1 is equal to

v1 = at1 = (18 ms−2 ) (30 s) = 540 ms−1


Chapter 2 Kinematics 24

Obviously, the velocity of the rocket should be zero at the maximum height H. So
we can find the time t2 as follows:
v1
0 = v1 − gt2 ⇒ t2 =
g
Substituting the result of t back into eq.(5.8), we obtain
   2
1 2 v1 1 v1
H = at1 + v1 − g
2 g 2 g
1 2 v12
= at +
2 1 2g
1 (540 ms−1 )2
= (18 ms−2 ) (30 s)2 +
2 2 × 9.8 ms−2
4
= 2.30 × 10 m

(b) The total time of flight is given by

t = t1 + t2 + t3 ,

where t3 is time taken for the rocket in free-fall to reach the ground form the
maximum height H. For the flight from the height H to ground,
s
1 2H
−H = − gt23 ⇒ t3 =
2 g

Therefore,

t = t1 + t2 + t3
s
v1 2H
= t1 + +
g g
r
540 ms−1 2 × 2.30 × 104 m
= 30 s + +
9.8 ms−2 9.8 ms−2
= 153.6 s

Example
A man near the cliff is pulling a boat in the river (see figure). The cliff is of height h. The
velocity and acceleration of the man are v and a respectively, when the rope makes an
angle φ to the horizontal. At the same instant the velocity and acceleration of the boat
are v 0 and a0 respectively. The rope keeps taut during the motion, find v 0 if the rope keeps
taut during the motion. Find also the acceleration a0 . Express your answer in terms of
the given parameters.
Chapter 2 Kinematics 25

Solution
There are two approaches to find the speed v 0 of the boat. The first one bases on concep-
tual idea and the second one relies on mathematical treatment.

Method I:
Observe that v 0 is not a component of v along the horizontal. But, instead v is a component
of v 0 along the rope because v 0 is the final velocity of the boat and the rope is always
tight.

v 0 cos φ = v
v
⇒ v0 =
cos φ
Method II:

Notice the right-angled triangle in the figure, we have l2 = h2 + x2 . The lengths l and x
vary with time while h is a constant. Differentiate on both sides with time, we obtain

2l l˙ = 2 xẋ
⇒ −2 lv = −2xv 0
lv
⇒ v0 =
x
0 v
⇒ v =
cos φ
dl dx
where l˙ = = −v and ẋ = = −v 0 . The negative sign appears as l and x decreases
dt dt
with time.
Chapter 2 Kinematics 26

v
To find the acceleration a0 , we differentiate the expression v 0 with time. Since v 0 = ,
cos φ
we have

dv 0
 
0 d v a cos φ + v φ̇ sin φ
a = = =
dt dt cos φ cos2 φ

dv
where a = . The problem is solved if φ̇ is known.
dt

Let’s consider the right-angled triangle again.

h = l sin φ
⇒ 0 = l φ̇ cos φ + l˙ sin φ
⇒ 0 = l φ̇ cos φ − v sin φ
v tan φ
⇒ φ̇ =
l
Plugging the above result to Eqn. (2.4), we obtain
 
v tan φ
a cos φ + v sin φ
0 l
a =
cos2 φ
0 a v tan2 φ
2
a = +
cos φ l cos φ
a v tan3 φ
2
a0 = +
cos φ h
where h = l sin φ.

2.3 Non-constant acceleration


Example
A particle is projected vertically upwards with speed u. In addition to the retardation
due to gravity, this particle is also subjected to a retarding force which has a magnitude k
times its speed, where k is a positive constant. Find the time this particle takes to reach
its greatest height.

Solution
Set upward motion as positive and the point of projection as the origin. Let y be the
displacement of the particle at time t. The resultant force exerted on the particle is
Chapter 2 Kinematics 27

−mg − kv, where v is the instantaneous velocity of the particle. So


dv
m = −mg − kv
dt
Separating the variables, we have
dv 1
= − dt
mg + kv m
Integrating both sides of the equation with definite integral, we have
Z 0
1 t
Z
dv
= − dt
u mg + kv m 0
0
1 t
ln (mg + kv) = −
k m
 u
1 mg t
ln = −
k mg + ku m

Therefore,
 
m ku
t = ln 1 +
k mg

Example
Dropping a stone at a height of h from the floor. The air resistance is not ignorable. Given
that the air drag force per unit mass = −kv, where k is a positive constant and v is the
instantaneous velocity of the stone. Find the variations of velocity and distance with time.

Solution

a=−g−kv
y(0)=h

0
Chapter 2 Kinematics 28

Convention: g is positive, k is positive, and v is negative as it points downward

a = −g − kv
dv
⇒ = −g − kv
dt
Separating the variables and integrating both sides with indefinite integral, we have
Z Z
dv
= − dt.
g + kv

1
⇒ ln(g + kv) = −t + A, where A = constant and g + kv > 0
k
⇒ g + kv = ek(−t+A) = Be−kt , where B = ekA
1
⇒ v = (Be−kt − g)
k
At t = 0, v(0) = 0 ⇒ B − g = 0 ⇒ B = g.
1 −kt g
∴ v= (ge − g) = − (1 − e−kt ),
k k
dy g
i.e. = − (1 − e−kt )
dt k
v
g
k

t
Z
g
y=− (1 − e−kt )dt + C, where C = constant.
k
g g
= − t − 2 e−kt + C
k k
g g
At t = 0, y = h ⇒ h = − 2 + C ⇒ C = h + 2 . Therefore,
k k
g g g
y = − t − 2 e−kt + h + 2
k k k
g g
= h − t + 2 (1 − e−kt )
k k
Chapter 2 Kinematics 29

2.4 Projectile motion

v0

θ x

( (
vx = v0 cos θ vy = v0 sin θ
At t = 0, , .
ax = 0 ay = −g

Let us study the horizontal and vertical motions of the object in turn.

I. Horizontal motion
d2 x
=0
dt2
dx
⇒ = vx = A, A = constant
dt
dx
At t = 0, = vx = v0 cos θ
dt
∴ vx (t) = v0 cos θ
Z
x(t) = v0 cos θ dt + B = (v0 cos θ) t + B, B = constant

∵ At t = 0, x = 0 ⇒ B = 0

∴ x(t) = (v0 cos θ) t

II. Vertical motion


d2 y
= −g
dt2
dy
⇒ = vy = −gt + C, C = constant
dt
dy
∵ At t = 0, = vy = v0 sin θ ⇒ C = v0 sin θ
dt
∴ vy (t) = −gt + v0 sin θ
Z
1 2 1
y(t) = − gt + v0 sin θ dt + D = − gt2 + (v0 sin θ) t + D, D = constant
2 2
Chapter 2 Kinematics 30

At t = 0, y = 0 ⇒ D = 0
1
∴ y(t) = (v0 sin θ) t − gt2
2

In conclusion:
ax (t) = 0 ay (t) = −g
vx (t) = v0 cos θ vy (t) = v0 sin θ − gt
x(t) = (v0 cos θ) t y(t) = (v0 sin θ) t − 21 gt2
or:

~a(t) = −g ĵ
~v (t) = v0 cos θ î + (v0 sin θ − gt) ĵ
1
~r(t) = (v0 cos θ) t î + [(v0 sin θ) t − gt2 ] ĵ
2
Let t0 be the time of flight, then
1
0 = (v0 sin θ) t0 − gt20
2
⇒ t0 = 0 (neglected as it is the initial status)
2v0 sin θ
or t0 = (2.4)
g
Let R be the horizontal range, then
2v02 sin θ cos θ v 2 sin 2θ
R = x(t0 ) = (v0 cos θ) t0 = = 0 (2.5)
g g
∴ Maximum range occurs at θ = 45◦ .

Example
A particle is projected from a point O on the horizontal floor. The range of the projectile
is R and the maximum height that the particle can reach is h.

Show that the equation of trajectory of the particle is


y 4x  x
= 1−
h R R
Chapter 2 Kinematics 31

Solution
The projectile intersection the floor at x = 0 and x = R. So the trajectory equation is
given by

y = A (x − 0) (x − R) ,

where A is a constant to be determined. Simply, we can write

y = A x (x − R) ,

R R
Knowing that the trajectory is symmetric about x = , then ( , h) satisfies the above
2 2
equation. So
  
R R
h = A −R
2 2
4h
A = − 2
R
4h
Therefore, we get y = − [x (x − R)], then
R2
y 4x  x
= 1−
h R R
Example

1
0m
0
1
R

1
0
0
1
v0
φ
x
At time t = 0, the mass m is released and a bullet is fired by a gun with a velocity ~v0
directed to the mass. Suppose that the range of the bullet covers the horizontal distance
between the mass and the gun. Show that the bullet can always hit the mass.
Chapter 2 Kinematics 32

Solution
~ be the initial position vector of the mass. Let ~r1 and ~r2 be the position vector of
Let R
the bullet and the mass at time t respectively.
About the mass:
d2~r2
= −g ĵ
dt2
d~r2 ~ ~ = constant vector
∴ = −gt ĵ + A, A
dt
~=0
∵ At t = 0, ~v2 = 0 ⇒ A
d~r2
∴ = −gtĵ
dt
1 ~
~r2 (t) = − gt2 ĵ + B
2
~ ⇒ B
∵ At t = 0, ~r2 = R ~ =R
~

~ − 1 gt2 ĵ
∴ ~r2 (t) = R
2
About the bullet:
d2~r1
= −g ĵ
dt2
d~r1 ~ ~ = constant vector
∴ = −gt ĵ + C, C
dt
~ = ~v0
∵ At t = 0, ~v1 = ~v0 ⇒ C
d~r1
∴ = −gt ĵ + ~v0
dt
1 ~
~r1 (t) = ~v0 t − gt2 ĵ + D
2
~ =0
∵ At t = 0, ~r1 = 0 ⇒ D
1
∴ ~r1 (t) = ~v0 t − gt2 ĵ
2

Rewriting ~r1 , ~r2 into î, ĵ components,

~ = R cos φ î + R sin φ ĵ
R
~v0 = v0 cos φ î + v0 sin φ ĵ
Chapter 2 Kinematics 33

~
where R = |R|.
1
∴ ~r1 (t) = (v0 cos φ) t î + [(v0 sin φ) t − gt2 ] ĵ
2
1 2
~r2 (t) = R cos φ î + [R sin φ − gt ] ĵ
| {z } 2
const.
To check if they will collide, we can check:
As the bullet passes through x = R cos φ, are the y position component of the bullet and
the mass the same?
Let t0 be the time that the bullet passes x = R cos φ.
R
(v0 cos φ)t0 = R cos φ ⇒ t0 =
v0
 2 
1 R R
∴ y-component of the bullet, y1 (t0 ) = v0 sin φ − g
2 v0v0
 2
1 R
y-component of the mass, y2 (t0 ) = R sin φ − g = y1 (t0 )
2 v0
Thus the bullet will hit the mass.

Remark:
The range of the bullet must be greater than Rscos φ. Using the result of equation (2.5),
2 v02 sin φ cos φ gR
we have ≥ R cos φ, so v0 ≥ . If the bullet is fired with the
g 2 sin φ
minimum velocity, it just hits the mass at y = 0.

2.5 Projectile on an inclined plane


A particle is projected with velocity v0 , from a point on a plane inclined at an angle β to
the horizontal. The elevated angle of projection is θ from the horizontal. The path of the
particle is in a vertical plane through a line of greatest slope of the plane. Investigate the
time of flight and the range of the particle.
Chapter 2 Kinematics 34

Instead of constructing the Cartesian coordinate system along the horizontal and vetical
respectively, we build a Cartesian coordinate system along and normal to the inclined
plane.
 
1
x = [v0 cos(θ − β)] t − g sin β t2 (2.6)
2
 
1
y = [v0 sin(θ − β)] t − g cos β t2 (2.7)
2
Set y = 0, equation (2.7) gives t = 0 (the initial time) and also the time of flight of the
projectile.
2 v0 sin(θ − β)
t=
g cos β
Putting this to equation (2.6), we obtain the range of the projectile
   
1
x = t v0 cos(θ − β) − g sin β t
2
   
2 v0 sin(θ − β) 1 2 v0 sin(θ − β)
= v0 cos(θ − β)] − g sin β
g cos β 2 g cos β
2
2 v0 sin(θ − β)
= {cos β cos(θ − β) − sin β sin(θ − β)} (Read the remarks)
g cos2 β
2 v02 sin(θ − β) cos θ
=
g cos2 β
2 v02
 
1
= 2
(sin(2θ − β) − sin β)
g cos β 2
v02
= (sin(2θ − β) − sin β)
g cos2 β
π π β
If 2θ − β = , x reaches its greatest value. So if we set θ = + , the range of the
2 4 2
projectile becomes the greatest and it is given by
v02
xmax = (1 − sin β)
g cos2 β
v02
=
g (1 + sin β)
Remarks:
Two trigonometric identities have been applied in the above derivation.

• cos(A + B) = cos A cos B − sin A sin B


1
• sin A cos B = [sin(A + B) + sin(A − B)]
2
Chapter 2 Kinematics 35

Example
The following figure shows a ball being thrown upward from a point on the side of a
hill which slopes upward uniformly at an angle of 28◦ . Initial velocity of the ball: v0 =
33 ms−1 , at an angle of θ0 = 65◦ (with respect to the horizontal). At what distance up
the slope does the ball strike and in what time?

Solution
Instead of using the method suggested by the last example, let’s construct a Cartesian
coordinate system along the horizontal and vertical respectively. When the projectile
moves, the horizontal component of the velocity remains constant at the initial value v0x ,
i.e. x = v0x t = (v0 cos θ0 ) t. So
x
t=
v0 cos θ0
where v0 is the initial velocity and θ0 is the angle of projection. Using this relation, we
obtain the trajectory equation of the projectile
   2
1 2 x 1 x
y = v0y t − gt = (v0 sin θ0 ) − g
2 v0 cos θ0 2 v0 cos θ0
2
gx
y = x tan θ0 − 2 (2.8)
2 v0 cos2 θ0

Note that v0y = v0 sin θ0 is the vertical component of the initial velocity.
Putting the parameters of the ball into the above equation, we find its trajectory equation

(9.8) x2
y = x tan 65◦ − = 2.14 x − 0.0252 x2
2 (33)2 cos2 65◦

The points on the incline are related by the equation: y = (tan 28◦ )x = 0.532 x.
At the value of x for which the ball hits the incline, we have

0.532 x = 2.14 x − 0.0252 x2


⇒ 1.608 x − 0.0252 x2 = 0 ⇒ x = 63.8 m
Chapter 2 Kinematics 36

(The solution, x = 0, is rejected since it refers to the point of projection.) The range on
the incline, R, obeys
x 63.8 m
x = R cos 28◦ ⇒ R = ◦
= = 72.3 m
cos 28 cos 28◦
And the time to reach this point is:
x 63.8 m
t= = = 4.57 s
v0 cos θ0 (33 ms−1 ) cos 65◦

Remarks:
(1) One may consider a coordinate system having axes along and normal to the inclined
plane, e.g. x0 and y 0 . The ball strikes on the incline when y 0 = 0.

(2) The trajectory equation in equation (2.8) is sometimes written as

gx2
y = x tan θ0 − 2
(1 + tan2 θ0 )
2 v0

Example
Suppose that there are numerous particles projected simultaneously with the same initial
speed v0 at the origin at time t = 0 and the projections are in all directions and in the
same vertical plane. What is the shape of the curve that the particles lie on at time t > 0?
Solution
Let the projection angle of any one of the particles be θ. The horizontal and vertical
displacements of this particle at time t are

 x = (v0 cos θ) t
 y = (v0 sin θ) t − 1 gt2
2
Rearrange the above equations, we obtain

 x2 = v02 t2 cos2 θ
 (y + 1 gt2 )2 = v02 t2 sin2 θ
2
By considering the identity sin2 θ + cos2 θ = 1, we obtain an equation without θ
1
x2 + (y + gt2 )2 = v02 t2
2
The above equation states that if numerous particles are projected simultaneously with
the same initial speed v0 at the origin at time t = 0, then at time t > 0 the particles lie
Chapter 2 Kinematics 37

on a circle. In particular, the circle is centered at (0, − 21 gt2 ) and has a radius v0 t.

Example
In one contest, a spring-loaded plunger launches a ball at a speed of 3.0 m/s from one
corner of a smooth, flat board that is tilted up at a 20◦ angle. To win, you must make
the ball hit a small target at the adjacent corner, 2.50 m away. At what angle θ should
you tilt the ball launcher?

Solution
The problem is the same as the projectile motion with reduced gravitational acceleration
a = g sin 20◦ .

The component g cos 20◦ is cancelled by the normal force. So, we have

 x = (v0 cos θ) t
(2.9)
 y = (v0 sin θ) t − 1 at2
2
x
From the equation set, we have t = , and hence
v0 cos θ
   2
x 1 ◦ x
y = v0 sin θ − (g sin 20 ) .
v0 cos θ 2 v0 cos θ
That is
g sin 20◦
 
2
y = x tan θ − x (2.10)
2v02 cos2 θ
Chapter 2 Kinematics 38

Plugging the given parameters x = 2.5 m, y = 0 m, and v0 = 3 m/s into the equation, we
obtain
7.2
tan2 θ − tan θ + 1 = 0
g sin 20◦
After solving, we have tan θ = 1.4659 or 0.6822. That is θ = 55.7◦ or 34.3◦ .
Chapter 3

Newton’s Law of Motion

3.1 Newton’s first law


If there is no net force acting on a body, the body will preserve its state of motion, i.e. if
the body is at rest, it remains at rest; if the body moves with a velocity, it will keep on
moving with that constant velocity. The mass of an object is a measure of the inertia of
it to oppose change in state. Greater the mass of the object, greater is the resistance to
change the state of motion of it.

Let’s consider an example of projectile under gravity. The vertical velocity of the bullet
changes with time because the gravitational force is vertical. However, the horizontal
velocity of the bullet never changes and it keeps its state. Thus the horizontal displace-
ments are the same at equal time intervals. If gravity is ignored, the bullet moves along
a straight path.

39
Chapter 3 Newton’s Law of Motion 40

3.2 Reference frame and relative motion


Reference frame: Where do we observe the motion of an object?

S : Earth’s frame

S 0 : Car’s frame moving with ~vS 0 S


rmS rmS’
S’ with respect to S
v S’S
rS’S
S

where ~rS 0 S - Position vector of the car observed from the earth,
~rmS - Position vector of the object observed from the earth,
~rmS 0 - Position vector of the object observed from the moving car.
Here, we have assumed the earth as an inertial frame (non-accelerating). In fact, S could
be any non-accelerating frame.
From the vector diagram,

~rmS = ~rS 0 S + ~rmS 0 (3.1)


d~rmS d~rS 0 S d~rmS 0
⇒ = +
dt dt dt
or ~vmS = ~vS 0 S + ~vmS 0 (3.2)

where ~vS 0 S - velocity of the car observed from the earth,


~vmS - velocity of the object measured from the earth,
~vmS 0 - velocity of the object measured from the moving car.
Differentiating equation (3.2) with respect to time, we obtain

~amS = ~aS 0 S + ~amS 0

If ~vS 0 S is a constant, i.e. ~aS 0 S = 0, the observers are inertial to each other. Hence,
~amS = ~amS 0 . The observers in S and S 0 have the same observation on the acceleration of
a moving object and they write down the same equation of motion (i.e. they conclude
identical equations based on Newton’s second law, F = ma).

From equation (3.1), we obtain

~rmS 0 = ~rmS − ~rS 0 S


Chapter 3 Newton’s Law of Motion 41

It states the position vector of the object with respect to the observer in the moving car
(i.e. frame S 0 ). Sometimes, we simply write

~rmS 0 = ~rm − ~rS 0

From equation (3.2), we obtain

~vmS 0 = ~vmS − ~vS 0 S

It states the velocity of the object with respect to the observer in the moving car (i.e.
frame S 0 ). Sometimes, we simply write

~vmS 0 = ~vm − ~vS 0

Example
One man, A, is travelling due south at 20 km/hr, another man, B, is at a distance of
30 km due NW of A and is travelling due east at 14 km/hr. Find the position vector of
A with respect to B. Find also the magnitude and direction of the relative velocity of A
with respect to B.
Solution

Let ~vA and ~vB be the velocity of men A and B respectively. The position vector of A
relative to B is
30
~rAB = 30 (cos 45◦ î − sin 45◦ ĵ) = √ (î − ĵ)
2
The direction of ~rAB is S 45◦ E.
As ~vA = −20 ĵ and ~vB = 14 î, then the velocity of A with respect to B is

~vAB = ~vA − ~vB = −14 î − 20 ĵ.


Chapter 3 Newton’s Law of Motion 42

p
2 2
The magnitude of the relative velocity is |~vAB | = (−14) + (−20) = 24.4 km/hr.
20
As α = tan−1 = 55◦ , then the direction of the relative velocity is S (90◦ −55◦ ) W = S 35◦ W.
14

3.3 Newton’s second law


If there is a net force F~ acting on an object m, then F~ equals to the rate of change of the
d~p
object momentum, i.e. F~ = .
dt
OR in a simple statement: if the ~
 mass of the object is a constant m, we have F = m~a,
d~p d(m~v ) d~v
as F~ = = =m = m ~a, where ~a is the object acceleration.
dt dt dt
Unit of force: Newton = kg m s−2 ([MLT−2 ])
One Newton of force is the magnitude of force acting on a 1 kg object that will accelerate
the object with acceleration of 1 ms−2 . Newton’s second law is valid when the observer
is an inertial one, i.e. the observer is not accelerating, he is either moving with constant
speed or at rest. When he constructs the the equation of motion of the object, the accel-
eration a must be measured with respect to him as an inertial observer.

Example
A ball bearing is released from rest and drops through a viscous medium. The retarding
force acting on the ball bearing has magnitude kv, where k is a constant depending on
the radius of the ball and the viscosity of the medium, and v is the bearing’s velocity.
Find the terminal velocity acquired by the ball bearing and the time it takes to reach a
speed of half the terminal velocity.

Solution
As the ball falls through the medium, it is accelerated by gravity and the viscous force
(which has direction opposite to that of the gravitational force). To find the acceleration
of the bearing, we use Newton’s second law i F~i = m ~a to relate the net force on the
P

ball to its acceleration.


Chapter 3 Newton’s Law of Motion 43

Taking downward as positive direction,


dv
mg − kv = ma = m
dt
where k becomes positive and a is the acceleration of the ball at any time. The initial
value of a is g, since at the moment of release v = 0. As the value of v increases, the
acceleration decreases until a = 0 when v = v0 , the terminal velocity. Therefore,
mg
mg − kv0 = 0 ⇒ v0 =
k
In order to find out at which time v = v0 , we must calculate t as a function of v (or vice
versa).
At any time t, it will be found that
dv
mg − kv = m ,
dt
thus

(mg − kv) dt = m dv
m dv
⇒ dt =
mg − kv
Z t Z v
m dv
⇒ dt =
0 0 mg − kv

where, in the integration limits, v = 0 at t = 0 and v = v at t = t. Hence,


m v d (mg − kv)
Z
t=−
k 0 mg − kv
Therefore,
v
m
t = − ln(mg − kv)
k 0
 
m mg − kv
t = − ln
k mg
 
m kv
= − ln 1 −
k mg
Chapter 3 Newton’s Law of Motion 44

v0 mg
The time to acquire half the terminal velocity, T , is found by inserting v = = in
2 2k
the above equation
      
m k mg  m 1 0.69 m
T = − ln 1 − · = − ln =
k mg 2k k 2 k

3.4 Newton’s third law


As a body exerts a force on another body, the second body also exerts a force on the first.
The two forces are equal in magnitude but opposite in direction.

F −F

3.5 Application of Newton’s law in one dimension

3.5.1 Pushing a packing crate


A worker w is pushing a packing crate of mass m1 = 4.2 kg. In front of the crate is a
second crate of mass m2 = 1.4 kg. Both crates slide across the floor without friction. The
worker pushes on crate 1 with a force F = 3.0 N. Find the accelerations of the crafts and
the force exerted by crate 1 on crate 2.

a a

R1
R3
m1 m1
m2 F R2 m2
R2

m1 g m2 g
Chapter 3 Newton’s Law of Motion 45

where R1 - reaction from the ground on m1 ,


R2 - action reaction pair between m1 and m2 ,
R3 - reaction from the ground on m2 .
Note that R1 = m1 g, R3 = m2 g and no vertical motion (i.e. a = 0).
Taking right side as positive. Using Newton’s 2nd law, we find the equation of motions:

F − R2 = m1 a (3.3)
R2 = m2 a (3.4)

Substitute (3.4) into (3.3), we get:

F − m2 a = m1 a
F
⇒ a=
m1 + m2
m2 F
∴ R2 = m2 a =
m1 + m2

Remark: It is interesting to compare the reaction force if the two crates are interchanged.
When we move two crates along a straight line, the crate of smaller mass should be put
in front of the heavier one in order to reduce damage.

3.5.2 Mass hanging in a lift


A mass m is hanging by a massless string which is attached to the ceiling of a lift. The
mass of lift is M .

Case 1: The lift has no acceleration but moves upward with constant speed v (see figure
below).

T2

T2 T2

T1 T1
T1 T1
00
11 00
11
11
00 11
00
00
11 00
11

mg mg

Mg Mg
Chapter 3 Newton’s Law of Motion 46

Taking upward as positive. Since a = 0, the equation of motions are given by


T1 − mg = 0 ⇒ T1 = mg
T2 − T1 − M g = 0 ⇒ T2 = (m + M )g

N.B. Results are the same for the following cases:


1) a = 0 and v > 0,
2) a = 0 and v < 0,
3) a = 0 and v = 0
Case 2: Lift accelerates upward with acceleration a.
Note that a must be positive for this case. Therefore, the equation of motions are now
given by
T1 − mg = ma ⇒ T1 = m(g + a)
T2 − T1 − M g = M a ⇒ T2 = (m + M )(g + a)

Note that T1 and T2 in this case are larger than that in the previous case (a = 0).

Example
Compute the least acceleration with which a 45-kg woman can slide down a rope if the
rope can withstand a tension of only 300 N.

Solution
According to Newton’s second law,
X
F~i = m~a .
i

Therefore, taking down as positive direction, the tension of the rope T and the acceleration
of the woman a are thus related by:

mg − T = ma

where m is the mass of the woman.


Now it is given that the maximum tension of the rope is Tmax = 300 N. Hence, the
minimum acceleration of the woman amin is given by:
mg − Tmax [(45 kg)(9.8 ms−2 ) − 300 N]
amin = = = 3.13 ms−2
m 45 kg
Example
What acceleration a must the cart Y have in order that block B does not slide down (it
is at rest relative to the cart)? The inclined surface of the cart is smooth.
Chapter 3 Newton’s Law of Motion 47

Solution
When there is no relative motion between the block and the cart, the block moves together
with the cart, i.e. an acceleration a along the horizontal. Obviously, the block is at
equilibrium along the vertical. Consider the free-body diagram of the block.

(
N sin θ = ma
N cos θ = mg

The above equations give tan θ = a/g or a = g tan θ.

Example
A movable system is placed on a frictionless floor, as shown in the figure. Assume that
all surfaces are smooth, the pulley and string are massless. Find the external horizontal
force F which should be exerted on m1 such that m2 has no relative motion to m1 .
Chapter 3 Newton’s Law of Motion 48

Solution
Let a be the common horizontal acceleration of m1 , m2 , and m3 when m2 and m3 have
no motions relative to m1 . The following figures show the free-body diagrams of m2 and
m3 respectively, where N2 and N3 are the normal forces exerted on them by m1 .

The equation of motion of m2 along the horizontal and the equation of motion of m3
along the vertical are

T = m2 a (3.5)
T = m3 g (3.6)
m3 g
Elimininating T from Eqns. (3.5) and (3.6), we get a = .
m2
Since F = (m1 + m2 + m3 ) a, we have
m3 g
F = (m1 + m2 + m3 ) .
m2
Example
A mass system consists a wedge W and two blocks A and B; their masses are mW , mA
and mB respectively. Blocks A and B are connected by light string over a massless pulley,
as shown in the figure. The wedge is at rest on a smooth horizontal plane and the vertical
side of it is adjacent to a fixed block. When the system is released, the following are
observed: block A accelerates down the wedge; block B moves vertically upward; wedge
W remains at rest. The elevated angle of wedge is α and the surfaces of all blocks, wedge,
and pulley are smooth.

(a) Find the acceleration of block A.


Chapter 3 Newton’s Law of Motion 49

(b) Hence, obtain the normal force exerted on the wedge by the fixed block.

Solution
(a) Since the objects mA and mB are connected by a tight string. The tensions in the
string are internal forces of this two-body system which has total mass mA + mB . The
acceleration of this system is resulted from the net of two forces mA g sin α and mB g. So

mA g sin α − mB g = (mA + mB ) a
g (mA sin α − mB )
a =
mA + mB
(b) Let the normal force exerted on the wedge be N . It is horizontal. Consider mA , mB ,
and mW to form a three-body system. Since mB and mW have no horizontal motions,
then

N = mA a cos α
mA g (mA sin α − mB ) cos α
=
mA + mB

3.6 Tension and normal force

A block of mass m is at rest on a table. The middle and right figures show the free-
body diagrams of the block and the table respectively. As the block is in equilibrium:
R − W = 0, where W = mg. Thus, the reaction force on the block from the table is
R = mg. Notice that R and W are not the action and reaction pair stated in Newton’s
third law of motion, because the forces are acting on the same object. On the other hands,
Newton’s three law of motion asserts that the action force on the table by the block has
magnitude R = mg and a downward direction, as shown in the right figure. The reaction
force on the table from the ground is N = R + W 0 . The mass and the weight of table are
m0 and W 0 respectively.
Chapter 3 Newton’s Law of Motion 50

Example
A mass is hanging by a string on a frictionless inclined plane.
y
x
T
T T
R R
m m
θ
θ mg mg

~ + T~ + m~g = 0
R

x-direction: T − mg sin θ = 0 ⇒ T = mg sin θ


y-direction: R − mg cos θ = 0 ⇒ R = mg cos θ

Example y
x
T

T α T α
R R
m m
θ
θ mg mg

x-direction: T cos α − mg sin θ = 0 ⇒ T = mg sin θ sec α


y-direction: R + T sin α − mg cos θ = 0 ⇒ R = mg cos θ − mg sin θ tan α

3.7 Pulley system


Under the ceiling is a mass system which has a non-stretchable and massless string, a
frictionless axis, a massless pulley, and two masses m1 and m2 .

T3

T2
T1

T2
T1
m2

m2 g m1

m1 g
m1 > m2
Chapter 3 Newton’s Law of Motion 51

As the pulley has no linear motion, we can write T3 = T1 + T2 . Here, we do not assert that
T1 = T2 = T , but instead we try to prove it by contradiction. Assume that T1 6= T2 , there
is a non-zero torque τ acting on the pulley and the pulley will has angular acceleration.
As the mass of the pulley is zero and thus its moment of inertia I is also zero. Then a
finite τ = I θ̈ gives τ /I = θ̈ → ∞ which is impossible. The knowledge of torque will be
introduced in latter chapters.
Analyzing the force on the two masses, we find that

m1 g − T = m1 a and T − m2 g = m2 a

Then a and T can be found by solving these two equations.

Example
A mass m2 is hanging over a pulley by a string which has its next end connected to a
block m1 . The block can slides on a frictionless inclined surface, as shown in the figure.
The pulley is massless and frictionless.

T For the block m1 :


R T
m1 x-direction: T − m1 g sin θ = m1 a
m2
⇒ T = m1 (a + g sin θ)
m1g m2g
θ y-direction: R − m1 g cos θ = 0
⇒ R = m1 g cos θ
x
a T
R For the block m2 :
y T
m1
m2 m2 g − T = m2 a ⇒ T = m2 (g − a)
θ a
m1g m2g

Combining the two expressions of T , we obtain:


(m2 − m1 sin θ)g
m2 (g − a) = m1 (a + g sin θ) ⇒ a =
m1 + m2
Example
A massless string is hanging over a massless and frictionless pulley. Its two ends are held
by two men A and B, A of mass m and B of mass m + M . At time t = 0, both men
Chapter 3 Newton’s Law of Motion 52

are at the same distance h below the pulley, and they start to climb up along the string.
Find the distance of B from the top when A arrives the top of the system at time t.
Solution

Denote the accelerations of A and B be a1 and a2 respectively.


(
T − mg = ma1
T − (m + M ) g = (m + M ) a2

Then
(
T = m (g + a1 )
T = (m + M ) (g + a2 )

Eliminating T , we get

(m + M ) (g + a2 ) = m (g + a1 )

Then

(m + M ) a2 = ma1 − M g

Hence,
m M
a2 = a1 − g (3.7)
M +m M +m
1 2
Multiplying both sides of Eqn. (3.7) by
t , we have
2
   
1 2 m 1 2 M 1 2
a2 t = a1 t − gt
2 M +m 2 M +m 2
Chapter 3 Newton’s Law of Motion 53

1 1
Knowing that h = a1 t2 and we denote S = a2 t2 . So
2 2
 
m M 1 2
S = h− gt
M +m M +m 2
 
1 M 2
= mh − gt
M +m 2

Therefore, the required distance is


 
1 M 2
h−S = h− mh − gt
M +m 2
 
M 1 2
= h + gt
M +m 2

Example
With what force F must a man pull on a rope in order to support the platform on which
he stands, if the weight of the man is W1 and that of the platform is W2 ? What is the
maximum weight of platform that the man can support sustainably? Assume that all
pulleys and ropes are massless and frictionless.

Solution
Suppose an equilibrium is maintained when the man applies a force F to the rope. The
forces in the pulley system are stated in the figure. As the pulley is massless, the net
force exerted on each pulley must be zero (Fnet, pulley = mpulley apulley = 0).
Chapter 3 Newton’s Law of Motion 54

Construct the free-body diagrams of the man and the platform respectively.

For the man:

F + N = W1 (3.8)

For the platform

3F = W2 + N (3.9)

W1 + W2
Solving equations (3.8) and (3.9), we have F = .
4
The maximum weight of platform that the man can support sustainably can be found
when the man applies his body weight to pull the rope, i.e F = W1 . Then equation (3.8)
gives N = 0 and equation (3.9) gives W2 = 3 W1 .
Example
A pulley system is shown in the figure where the upper pulley and the strings are massless.
The lower pulley has mass m. When the system is released m1 and m3 move downward
relative to the upper pulley, and m2 moves upward relative to the upper pulley. Find the
acceleration of m3 . Assume that there is no friction.
Chapter 3 Newton’s Law of Motion 55

Solutions
Let the acceleration of m3 be a0 relative to the upper pulley and the acceleration of m2
be a relative to the lower pulley. Denote the tension in the string connected to m3 as T 0
and the tension in the string that connected to m2 as T .

m1 : m1 g − T = m1 (a − a0 ) (3.10)
m2 : T − m2 g = m2 (a + a0 ) (3.11)
m3 : m3 g − T 0 = m3 a0 (3.12)
m: T 0 − 2T − mg = m a0 (3.13)

Eliminating a from equations (3.10) and (3.11), we have

2 m1 m2 − (m1 + m2 ) T = −2 m1 m2 a0 (3.14)

Eliminating T 0 from equations (3.12) and (3.13), we have

(m3 − m) g − 2 T = (m + m3 ) a0 (3.15)
Chapter 3 Newton’s Law of Motion 56

Eliminating T from equations (3.14) and (3.15), we have

[(m3 − m) (m1 + m2 ) − 4 m1 m2 ] g
a0 = (3.16)
(m1 + m2 ) (m + m3 ) + 4 m1 m2
Example
A massless rod of length l has its two ends connected to two identical particles A and
B. The system is held to align with a smooth vertical wall where both particles meet
the wall and the lower particle B lies on the smooth horizontal floor. A disturbance is
given so that the system slides down with A moving down along the wall and B leaving
the wall along the floor. You may consider a reference frame where its origin O locates
at the joint of the wall and the floor, OA = y and OB = x, where x and y are positive
quantities. Show that cos θ = 2/3 when A leaves the wall, where θ is the acute angle that
the rod makes with the wall.

Solution
From the figure, we have the relation x2 + y 2 = l2 . Hence, we obtain

xẋ + y ẏ = 0 (3.17)
and xẍ + ẋ2 + y ÿ + ẏ 2 = 0 (3.18)

The conservation of mechanical energy gives


1 1
mg (l − y) = mẋ2 + mẏ 2
2 2
Chapter 3 Newton’s Law of Motion 57

⇒ 2g (l − y) = ẋ2 + ẏ 2 (3.19)

Eq. (3.18) becomes

xẍ + y ÿ + 2g (l − y) = 0 (3.20)

Consider A: Note that T is a compressive force.


(
R1 − T sin θ = 0
T cos θ − mg = mÿ

Consider B:
(
T sin θ = mẍ
R2 − mg − T cos θ = 0

When A leaves the wall, R1 = 0, we obtain T = 0 and ÿ = −g. Also, we have ẍ = 0 when
T = 0.
From Eq. (3.20), x(0) + y(−g) + 2g (l − y) = 0, which gives y = 2l/3. Since y = l cos θ,
2l
we can write = l cos θ, i.e. cos θ = 2/3.
3
Chapter 4

Motion in a Plane

4.1 Uniform circular motion


- On a horizontal frictionless plane

y
v1 vp
P
P1 θ P2

θ θ v2
θ ~v1 = v cos θ î + v sin θ ĵ
x ~v = v cos θ î − v sin θ ĵ
2
r

2rθ
Time required for moving from P1 to P2 : ∆t = (θ is in radian)
v
Note that v = |~v1 | = |~v2 |
v2x − v1x
x-component of average acceleration: aav,x =
∆t
∵ v2x = v cos θ, v1x = v cos θ
∴ aav,x = 0

58
Chapter 4 Motion in a Plane 59

v2y − v1y
y-component of average acceleration: aav,y =
∆t
∵ v2y = −v sin θ, v1y = v sin θ
 2 
−2v sin θ v sin θ
∴ aav,y = =−
2rθ/v r θ
sin θ
lim =1
θ→0 θ

Now we can take the limit θ → 0 to obtain the instantaneous acceleration at P .


  2   2
v sin θ v
ay = lim − =−
θ→0 r θ r
Notice that with this condition, ax = 0.

At P , the instantaneous acceleration is purely y-direction, pointing towards the center


of the circular motion and has a magnitude of v 2 /r.
For a uniform circular motion, the centripetal acceleration is a = v 2 /r and is always
pointing towards the center of the circle.
The tangential velocity is given by v = r θ̇. Let ω be θ̇, then we have v = r ω. So, the
centripetal force can be written as a = ω 2 r.
y
v

x
r

Example
A disk of mass m on a frictionless table is attached to a hanging cylinder of mass M by
a cord through a hole in the table. Find the speed with which the disk must move in a
circle of radius r for the cylinder to stay at rest.
Chapter 4 Motion in a Plane 60

Solution

The above figures show the free-body diagram of the disk and the cylinder. For M to
remain at rest the tension T in the cord must equal the gravitational force M g exerted
on M , figure (b). The tension supplies the centripetal force that keeps
r m in its circular
2 2
mv mv M gr
ordit, figure (a). So T = . Thus M g = and we obtain v = . The vertical
r r m
forces exerted on the disk as shown in figure (a) are irrelevant to the discussion of the
problem (i.e. N and mg).

Example
A mass m slides without friction on the roller coaster track shown in the below figure.
The curved sections of the track have radius of curvature R. The mass begins its descent
from the height h. At certain value of h, the mass will begin to lose contact with the
track. Indicate on the diagram where the mass loses contact with the track and calculate
the minimum value of h for which this happens.
Chapter 4 Motion in a Plane 61

Solution

Just before the inflection point A of the track (i.e. α > 30o ), the normal reaction of the
track on the mass is
mv 2
N= + mg sin α > 0,
R
where v is the velocity of the mass and its direction points toward O. The mass should
not lose contact before reaching A, because both terms in the RHS of the above equation
is positive. The normal force N is decreasing when the mass is climbing up and before
reaching A.
R 1
At the inflection point A, the angle α = β = 30◦ since sin α = sin β = = . Just after
2R 2
point A (i.e. β > 30◦ ), the normal reaction is

mv 2
N = mg sin β − ,
R
Chapter 4 Motion in a Plane 62

which could take zero or negative value. The direction of N is away from O0 . Here,
mv 2
the force mg sin β increases and decreases when the mass is away from A (i.e. β
R
is increasing), and thus N increases when the mass moves away from A. To conclude,
The earliest the mass can start to lose contact with the track is at A when N ≤ 0, i.e.
mv 2
≥ mg sin 30◦ .
R
3R
The conservation of mechanical energy mg [h − R sin 30◦ ] = 21 mv 2 then requires h ≥ .
4
3R
Hence the minimum h required is .
4

4.2 Polar coordinate system


In many situations, it is convenient to use plane polar coordinates (r, θ) instead of rectan-
gular coordinates (x, y) to describe the motion of a particle, where r is the length of the
position vector. We define two unit vectors in polar coordinates that are perpendicular to
each other. ( The unit vectors (î, ĵ) are for rectangular, their directions are fixed; another
pair of unit vectors (êr , êθ ) are for polar, their directions vary with the position vector.)
(
êr = cos θ î + sin θ ĵ
(4.1)
êθ = − sin θ î + cos θ ĵ
dêr


 = − sin θ î + cos θ ĵ = êθ
 dθ

(4.2)
 dêθ = − cos θ î − sin θ ĵ = −ê



r

The position vector ~r in terms of polar coordinates is given by ~r = rêr . (Note that êr
and êθ are functions of θ, i.e. êr = êr (θ) and êθ = êθ (θ). They are unit vectors, but not
constant vectors because their directions are varying.)
Chapter 4 Motion in a Plane 63

Thus the velocity ~v is


d~r d dr d êr
~v = = (rêr ) = êr + r (4.3)
dt dt dt dt
dêr dêr dθ
( = · = θ̇êθ ) (4.4)
dt dθ dt
= ṙ êr + rθ̇ êθ (4.5)
= vr êr + vθ êθ (4.6)

where vr is radial velocity component of ~v along êr and vθ is angular velocity component
of ~v along êθ .
The acceleration of the system is given by
d~v d
~a = = (ṙ êr + rθ̇ êθ ) (4.7)
dt dt
dṙ dêr dθ dr dθ̇ dêθ dθ
= êr + ṙ · + θ̇êθ + r êθ + rθ̇ ·
dt dθ dt dt dt dθ dt
= r̈êr + ṙ(êθ )θ̇ + ṙθ̇êθ + rθ̈êθ + rθ̇(−êr )θ̇
= (r̈ − rθ̇2 ) êr + (rθ̈ + 2ṙθ̇) êθ (4.8)

where (r̈ − rθ̇2 ) = ar is radial acceleration and (rθ̈ + 2ṙθ̇) = aθ is the angular component
of the acceleration.

Example
A radar monitors a flying object with trajectory r = 1.0 + 0.5 cos θ, the measurement is
in meter. At the position θ = 45o , the radar records the angular velocity of the object to
be 0.6 rad/s. Find the speed of the object at the instant.

Solution

Since r = 1.0 + 0.5 cos θ, we have ṙ = −0.5 θ̇ sin θ. The velocity vector in the polar
coordinate system gives ~v = ṙ êr + rθ̇ êθ = −0.5 θ̇ sin θ êr + rθ̇ êθ . Plugging in the condi-
tions, i.e. θ = 45o , r = (1.0 + 0.5 cos 45o ) m = 1.354 m and θ̇ = 0.6 rad/s. We obtain
p
~v = −0.212 r̂ + 0.812 θ̂. The speed of the object is (−0.212)2 + (0.812)2 = 0.839 m/s.
Chapter 4 Motion in a Plane 64

Example
A particle of mass m is released inside a smooth tube AB when the tube is rotating
with constant angular speed ω about A horizontally. The initial position of the particle
is r0 from A and the tube has length l. Find the speed of the particle will it leaves the tube.

Solution

Since the tube is smooth, there is no force acting on the particle along the tube. The
equation of motion of the particle along the tube is given by

r̈ − ω 2 r = 0
 
dṙ dṙ dr dṙ
Using the chain rule, we have r̈ = = = ṙ. The equation of motion
dt dr dt dr
becomes
 
dṙ
ṙ − ω 2 r = 0
dr
By considering the separation of variables, we obtain

ṙ dṙ = ω 2 r dr

Integrating on both sides,


Z vr Z l
2
ṙ dṙ = ω r dr
0 r0
vr2 = ω 2 (l − r02 )
2

The speed of the particle when it leaves the tube is given by


q
v = vr2 + vθ2
q
= ω 2 (l2 − r02 ) + l2 ω 2
q
= ω 2 (2l2 − r02 ).
Chapter 4 Motion in a Plane 65

Example
A smooth rod OA of length 2l rotates in a horizontal plane about its end O with con-
p
stant angular speed 6g/l. One end of a light elastic string of natural length l and force
constant 2mg/l is attached to O and the other end to a small smooth ring P of mass m
which passes around the rod. The ring is projected from the midpoint of the rod with

speed 3 gl relative to the rod in the direction OA. Show that while the ring is in contact
4gr
with the rod its distance r from O satisfies the equation r̈ − = 2g. Find also the
l
boundary conditions of this equation.

Solution

Since the rod and the ring are smooth, the only force that acts on the ring along the rod
is the restoring force due to the elastic string. The magnitude of the force is given by
2mg
(r − l). The motion of the ring is governed by
l
2mg
− (r − l) = m(r̈ − rθ̇2 ) . (4.9)
l
On the left side of Eq. (4.9), a negative sign appears and it simply represents the direction
of the restoring force, pointing from the ring to the rotation center O. The terms on the
right side comes from the radial part of Eq. (4.8). Plugging into Eq. (4.9) the angular
p
speed θ̇ = 6g/l, we have
 
2g 6g
− (r − l) = r̈ − r .
l l
4gr
After simplification, we have r̈ − = 2g. There are two boundary conditions associated
l √
with this equation, e.g. r(0) = l and ṙ(0) = 3 gl.
Chapter 4 Motion in a Plane 66

Example
A man near the cliff is pulling a boat in the river by a rope, as shown in the figure. The
cliff has a height h above the river. The velocity and acceleration of the man are v and a
respectively when the rope makes an angle φ to the horizontal. Denote the velocity and
acceleration of the boat at the same instant as stated as v 0 and a0 respectively. The rope
keeps taut all the times. Construct the polar coordinates system (r, θ), where the origin
O is located at the edge of the cliff and the position vector of the boat is ~r = rêr .

(a) By using the expression for the velocity vector: V~ = ~r˙ = ṙ êr + rθ̇ êθ , find the velocity
of the boat v 0 . Express your answer in terms of v and φ.

~ = V~˙ = (r̈−rθ̇2 ) êr +(rθ̈+2ṙθ̇) êθ ,


(b) By using the expression for the acceleration vector: A
find the acceleration of the boat a0 . Express your answer in terms of h, φ, v, and a.

Solution
(a) The position vector of the boat is ~r as shown in the figure below.
Chapter 4 Motion in a Plane 67

The velocity of the boat is V~ = ~r˙ = ṙ êr + rθ̇ êθ = vr êr + vθ êθ . Along the radial direction,
the rope is always taut, so we have

vr = ṙ
−v 0 cos φ = −v
v
Hence, v 0 = .
cos φ
(b) The acceleration along the radial direction is ar = r̈ − r θ̇2 .
As θ = 2 π − φ, it gives θ̇ = −φ̇, so ar = r̈ − r φ̇2
By considering the right-angled triangle, we have

r sin φ = h
ṙ sin φ + r φ̇ cos φ = 0
v tan φ
φ̇ =
r
Hence,

v 2 tan2 φ
ar = r̈ − (4.10)
r
But ar = −a0 cos φ and r̈ = −a, so Eqn. (4.10) becomes

v 2 tan2 φ h
−a0 cos φ = −a − (Because r = )
h sin φ
sin φ
a v 2 tan3 φ
a0 = +
cos φ h
Example
Two particles A and B of equal masses are attached to the ends of a light and inextensible
string which passes through a small and fixed smooth ring O on a smooth horizontal table
such that the portions OA and OB of the string are straight and perpendicular with
OA = a and OB = b. Initially, the particle A is given a speed u in the direction of OB,
as shown in the figure. Let (r, θ) be the polar coordinates of particle A at time t in the
subsequent motion, where a ≤ r ≤ a + b. The origin of the coordinate system is at the
ring O and OA lies on the x-axis when t = 0.
Chapter 4 Motion in a Plane 68

a2 u2
(a) Show that 2r̈ = .
r3
u2 a2
 
2
(b) Show also that ṙ = 1− 2 .
2 r
s
b (2a + b) u2
(c) Hence, show that particle B has speed when it reaches the ring O.
2 (a + b)2

Solution

(a) Notice that the total length of the string is fixed. We have r + rB = a + b, where rB
is the instantaneous length of OB at time t. Thus

r̈ + r̈B = 0 (4.11)

Consider the equations of motion of particle A along and normal to the string respec-
tively.
Along the radial direction:

m(r̈ − rθ̇2 ) = −T (4.12)


Chapter 4 Motion in a Plane 69

Along the tangential direction:

rθ̈ + 2ṙθ̇ = 0 (4.13)

Eqn. (4.13) can be rewritten as


1 d 2
(r θ̇) = 0 (4.14)
r dt
And thus, we obtain r2 θ̇ = c, where c is a constant.
u u
At time t = 0, r = a and θ̇ = . We have a2 = c which gives c = au. Therefore,
a a
we can write
au
θ̇ = 2 (4.15)
r
Consider the equation of motion of particle B along the string. We have mr̈B = −T .
Using eqn.(4.11) we obtain

mr̈ = T (4.16)

Eqns. (4.12) and (4.16) imply m(r̈ − rθ̇2 ) = −mr̈ which gives 2r̈ = rθ̇2 . Use the
a2 u 2
result in Eqn. (9.38), hence we obtain 2r̈ = 3 .
r
(b) From the result in (a), we have
a2 u 2
2r̈ =
r3
dṙ
Rewrite r̈ in the L.H.S. of the above equation as ṙ , then we have
dr
dṙ a2 u 2
2ṙ= 3
dr r
Rearrange the above equation and use integration, we obtain
Z ṙ Z r
2 2 dr
2 ṙ dṙ = a u 3
0 a r
Therefore, we have
u2 a2
 
2
ṙ = 1− 2
2 r
2 2
(c) Particle B reaches O when r = a + b. Notice
2
 that ṙ2B =−ṙ and we have ṙB = ṙ . Use
2 u a
the result in (b), we have ṙB = ṙ2 = 1− . Hence, we obtain
2 (a + b)2
s
b (2a + b) u2
|ṙB | =
2(a + b)2
Chapter 5

Application of Newton’s Law

5.1 Frictional force

5.1.1 Static friction

N
fs
F F = fs
m µs N
fs

F
start to
mg move

If a mass m is placed on a table with friction. We have experience that we must have
large enough force to have it moved.
Before the mass is moved,
F = fs

and they have opposite direction. Thus the net force on the mass is zero.
There exist a maximum static friction force fs,max such that

fs ≤ fs,max = µs N

where µs is the coefficient of static friction.

70
Chapter 5 Application of Newton’s Law 71

5.1.2 Kinetic friction


As a mass m is moving on a frictional surface, there exist a frictional force, for which its
direction is against the motion. Its magnitude is always: fk = µk N .
N.B.: µk < µs for the same surface.

fs
f
k

F
fs

Example
v
A block is projected upward with speed v. Take upward motion
as positive: −µk mg cos θ − mg sin θ = ma.
nθ Hence, we obtain a = −µk g cos θ − g sin θ. That means the
m g si
fk block decelerates upward with µk g cos θ + g sin θ.
θ

0 A block is released from rest on an incline plane, its


v=
subsequent motion depends on the coefficient of static
friction µs :
fs
si nθ
mg a) if mg sin θ < fs,max = µs mg cos θ or µs > tan θ,
then it will continue stay at rest.
θ

v b) if mg sin θ > fs,max or µs < tan θ, it will slide down.

si nθ fk If mg sin θ > fs,max and on its way down,


mg
µk mg cos θ − mg sin θ = ma
θ
⇒ a = µk g cos θ − g sin θ < 0, it accelerates downward.

Example
Chapter 5 Application of Newton’s Law 72

A movable table has rough surface of frictional coefficient µ and a set up with it (see
figure). The set up contains one frictionless and massless pulley, one massless cord and
two blocks m and M , where m < M . Assume that the cord keeps tight during the motion
of the system. By considering the relative motion of the blocks with respect to the table,
find the acceleration of m relative to the table and the tension of the cord if the table is
accelerating downward with aT .

Solution

Consider the motion of m:

mg − N = maT (5.1)
T − f = ma0 , (5.2)

where f = µN and a0 is the acceleration of m relative to the table. Eqn. (5.2) becomes

T − µN = ma0 (5.3)

The motion of M along the vertical:

M g − T = M (a0 + aT ) (5.4)
Chapter 5 Application of Newton’s Law 73

Use Eqns. (5.3) and (5.4), we have

M g − µN − ma0 = M (a0 + aT ) (5.5)

Use Eqns. (5.1) and (5.5), we have

M g + µ (maT − mg) − ma0 = M a0 + M aT


M (g − aT ) − mµ (g − aT )
a0 =
M +m
Hence, we obtain
(g − aT ) (M − mµ)
a0 = (5.6)
M +m
From Eqns. (5.4) and (5.6), we obtain the tension

T = M (g − a0 − aT )
(g − aT ) (M − mµ)
= M (g − − aT )
 M +m 
gM + gm − M g + M aT + gmµ − mµaT − M aT − maT
= M
M +m
M
= (gm + gmµ − mµaT − maT )
M +m
M
= (gm (µ + 1) − maT (µ + 1))
M +m
Therefore,
mM (µ + 1) (g − aT )
T =
M +m

5.2 A Revisit to circular motion


a) Conical pendulum
y

T θ
y
T
T
θ
1
0 R
x 1
0 x
0
1 0
1

mg mg
Chapter 5 Application of Newton’s Law 74

y-direction:

T cos θ − mg = may = 0 ( ∵ no acceleration in y-direction)


mg
⇒ T =
cos θ
x-direction:
   mg   sin θ 
sin θ
T sin θ = max ⇒ ax = T = = g tan θ
m cos θ m

N.B. ax is pointing towards the circle center and is also perpendicular to ~v . It is thus
the centripetal acceleration for the uniform circular motion. In addition, the
string is never horizontal because the weight of the bob have to be balanced by
the vertical component of the tension in the string.

v2 p
∴ ax = g tan θ = ⇒ v = gR tan θ
R

Period T is given by s
2πR R cot θ
T = = 2π
v g

b) The rotor
In an amusement park, there is an exciting game called rotor. A hollow cylinder is
rotating about its axis with uniform circular motion. A person with his hands back
and is against the inner wall of the cylinder. The person does not fall when the floor
moves down and leaves the person’s feet.
Chapter 5 Application of Newton’s Law 75

For the person to stick on the wall without falling


down,
mg < fs,max = µs N (5.7)

The person is also under uniform circular motion and


the normal reaction from the wall provides the re-
quired centripetal acceleration.

mv 2
axis of fs ∴ N=
rotation R
Thus, from (5.7),
N s s
mv 2 gR gR
cylinder mg < µs or v> i.e.vmin =
wall R µs µs

mg If the rotor spins too slow, the person will fall down.

c) A bicycle moving around a curve on a level road

The bicycle is moving with constant speed v.

N = mg
N
and the friction provides the required centripetal
force.
mv 2
∴ fs =
R
For the bicycle not slipping, fs

mv 2 O R
fs = < fs,max = µs N = µs mg mg
R p
⇒ v 2 < µs Rg or v < µs Rg

i.e. if the bicycle runs too fast, it will slip.


Chapter 5 Application of Newton’s Law 76

d) A bicycle moving around a curve on a banked road

N
θ y

O R
θ
fs

θ mg

As a reminder, the discussion below regards the bicycle as a point mass.

y-direction:
mg + fs sin θ
N cos θ − fs sin θ − mg = 0 ⇒ N = (5.8)
cos θ
x-direction:
mv 2
N sin θ + fs cos θ = max = (5.9)
R

Remark: Even if there is no friction between the bicycle and the road, the bicycle can
turn around the curve without slipping, i.e.
mg mv 2
N= & N sin θ =
cos θ R
R mg
 
⇒ v2 = sin θ = Rg tan θ
m cos θ
p
⇒ v = Rg tan θ

For a rough road and no slipping, (5.8) and (5.9) give


fs mv 2
+ mg tan θ = (5.10)
cos θ R
2
mv
⇒ fs = cos θ − mg sin θ (5.11)
R
Chapter 5 Application of Newton’s Law 77


Case I: fs > 0 (Friction points downward), equation (5.11) gives v > Rg tan θ
Equation (5.10) indicates the direction of forces as shown in the following
figure.

fs
mg tan θ cos θ
Both the friction term and the mg term
contributes to the centripetal force, the
mv2 mv 2
R centripetal force is large.
R


Case II: fs < 0 (Friction points upward), equation (5.11) gives v < Rg tan θ
Equation (5.10) indicates the direction of forces as shown in the following
figure.

The centripetal force required is smaller


than the mg term, because the frictional
force has a direction against the mg term.

When will slipping occur?


Refer to case I: Frictional force points downward if v > Rg tan θ
What is the upper limit of v if slipping does not occur?
The magnitude of v is limited when the frictional force has maximum static friction
fs,max . From (5.8),

mg + µs N sin θ mg
N= ⇒ N=
cos θ cos θ − µs sin θ

From (5.9),
2
mvmax
N sin θ + µs N cos θ =
R
2
 
mg mvmax
⇒ (sin θ + µs cos θ) =
cos θ − µs sin θ R
s  
sin θ + µs cos θ
⇒ vmax = gR
cos θ − µs sin θ

When v > vmax , the bicycle slips outward.


Chapter 5 Application of Newton’s Law 78

s  
p sin θ + µs cos θ
To conclude, if Rg tan θ < v ≤ gR , the frictional force points
cos θ − µs sin θ
downward and there is no slipping.

p
Refer to case II: Frictional force points upward if v < Rg tan θ

What is the lower limit of v if slippling does not occur?


fs
Note that is in opposite direction to that of mg tan θ, i.e.
cos θ
fs mv 2

mg tan θ − =
cos θ R
mv 2
Note also that may be so small such that
R
2

fs,max mvmin
mg tan θ −
=
cos θ R

Clearly, the bicycle slips inward when v < vmin .

µs N v2
mg tan θ − = m min
cos θ R
But from (5.8),

mg − µs N sin θ mg
N= ⇒ N=
cos θ cos θ + µs sin θ

From (5.9),
2
mvmin
N sin θ − µs N cos θ =
R
v2
 
mg
⇒ (sin θ − µs cos θ) = m min
cos θ + µs sin θ R
s  
sin θ − µs cos θ
⇒ vmin = gR
cos θ + µs sin θ

When v < vmin , the bicycle slips inward.

s  
p sin θ − µs cos θ
To conclude, if Rg tan θ > v ≥ gR , the frictional force points
cos θ + µs sin θ
upward and there is no slipping.
Chapter 5 Application of Newton’s Law 79

Example
A test tube is filled with a liquid of uniform density ρ such that the height of the liquid is
L. The tube is set to rotate by a centrifuge. When the tube is rotating with a constant
angular speed ω about a fixed vertical axis, it is almost horizontal. The axis is at a dis-
tance d from the liquid surface while the tube remains horizontal. Find the pressure on
the bottom of the tube during the motion. The atmospheric pressure is p0 .

Solution

The force acting on the liquid element is S dP , where S is the cross sectional area and
dP is the pressure difference on the two sides of the liquid element. The mass and
acceleration of the element are given as ρ dx S and ω 2 x respectively. By Newton’s second
law of motion,

S dP = ρ dx S ω 2 x
⇒ dP = ρ ω 2 x dx

Integrating both sides, we have


Z p Z L+h
2
dP = ρ ω x dx
p0 h
L+h
x2 2
p − p0 = (ρ ω )
2 n
ρ ω2
p − p0 = [(L + h)2 − h2 ]
2
ρ Lω 2
p = p0 + (L + 2h)
2
Example
A wedge of mass M rests on a frictionless horizontal plane. A block of mass m is released
from rest on the smooth and inclined surface of the wedge which has an elevation angle θ.
Chapter 5 Application of Newton’s Law 80

The acceleration of the wedge and the acceleration of the block relative to the wedge are
a1 and aM respectively. Write down the equations of motion of the wedge and the block
relative to an inertial observer. Hence, find a1 and aM . Check the limit when M >> m.

Solution
Before doing this problem, we have to notice two important points.

• If M is not moving then we have the trajectory sketched in the below figure.

• If M is moving to the left, then the actual trajectory is sketched as below, where
aM is the acceleration of the block relative to the wedge of mass M , and a1 is the
acceleration of the wedge relative to the table.

Now, we look at the force diagram of the wedge and the block and write down the equa-
tions of motion along the horizontal and vertical respectively (if necessary).
Chapter 5 Application of Newton’s Law 81

For M : N2 sin θ = M a1 (5.12)


For m : mg − N2 cos θ = m aM sin θ (5.13)
N2 sin θ = m (aM cos θ − a1 ) (5.14)

This is a set of simultaneous equations with 3 unknowns. After solving, we obtain



mg sin θ cos θ

 a1 =
M + m sin2 θ



(5.15)
(M + m) g sin θ



 aM =

M + m sin2 θ
The reaction between the wedge and the block is given by
mM g cos θ
N2 = (5.16)
M + m sin2 θ
Consider the equation set (5.15), when M is very large, e.g. M >> m, we have

a1 → 0 and aM → g sin θ.

From eq. (5.16), N2 → mg cos θ.

5.3 Non-inertial observer and fictitious force


A non-inertial observer is an observer who accelerates with respect to an inertial reference
frame. When a non-inertial observer describes the motion of an object, Newton’s second
law is not valid and modification is required. In fact, a fictitious force acts on the object
to be described with respect to this observer. The fictitious force acts directly opposite
to the acceleration of the observer.
Refer to equation (3.2) we have

~amS = ~aS 0 S + ~amS 0 ,

where S is an inertial frame. Let S 0 be an accelerating frame, then ~amS 0 represents the
acceleration of the object measured by an non-inertial observer in frame S 0 . Rearranging
the equation and multiplying both sides by m (mass of the object), we have

m ~amS 0 = m ~amS − m ~aS 0 S ,

then m ~amS 0 = FmS + Ffictitious , where FmS = m ~amS is the net force acting on the object
and Ffictitious = −m ~aS 0 S is the fictitious force to be added in order to rewrite Newton’s
Chapter 5 Application of Newton’s Law 82

second law for an non-inertial observer. Let’s state again Newton’s second law for an
inertial observer, it is F = ma, where a is measured by an inertial observer. Then, for an
non-inertial observer, we have

FmS + Fm, fictitious = m ~amS 0

Example
An observer is fixed at point O to watch the motion of a block of mass m which is
connected by a light string. If the block has an acceleration of magnitude a towards
right, then he concludes that a force F acts on the block through the string such that it
accelerates with a leaving him. With respect to the observer (he is an inertial observer),
the equation of motion of the block based on Newton’s second law is F = ma.

Repeat the experiment again in such a way that the string is cut and no net force is
applied to the block. If the observer accelerates towards left with a, then he will say, ”the
block accelerates with a towards right”. It is the motion with respect to him. But, how
can he write down the equation of motion correctly? For him, he knows that there is no
string connected to the block and thus there is no horizontal force exerted on it. The net
force on the block is zero, i.e. F = 0. However, he recognizes that the acceleration of the
block is a towards right. So, his first attempt to the equation is putting zero on the LHS
of the equation and ma on the RHS of the equation, i.e.

0 = ma ??

which is absolutely wrong!

What’s wrong with this? To his observation, the block accelerates with a towards right
and the net force exerted on it is ”zero”. Nevertheless, the observer is accelerating!
Newton’s second law is valid for inertial observer only. In fact, the observer does not
Chapter 5 Application of Newton’s Law 83

realize that he is accelerating, so he can never write a correct equation of motion unless
he includes the concept of ”fictitious force”. This force exerts on the block in a direction
that it is opposite to the acceleration of the observer and the force is ma towards right in
order to guarantee the correctness of the equation. So he writes the equation again.

0 + ma = ma ,

As a reminder, when an non-inertial observer plans to write the equation of motion of an


object, he should include the ”fictitious force” in the equation.
Example
Repeat the above example but a non-inertial observer that is fixed on the wedge. Write
down the equations of motion of the block relative to this observer, along and normal to
the inclined surface of the wedge. Find a1 and aM again.

Solution

Consider the motion of the block relative to the wedge.


Along the incline:
mg sin θ + ma1 cos θ = maM , (5.17)

where ma1 cos θ is the fictitious force acting on the block along the incline.
Normal to the incline:
N2 − mg cos θ + ma1 sin θ = 0 , (5.18)

where ma1 sin θ is the fictitious force acting on the block normal to the incline. As a
reminder, the fictitious force points opposite to the motion of the observer. Using eqs.
(5.12), (5.17) and (5.18), we solve the answers of a1 and aM again.

Example
A wedge is fixed to a rotating turntable as shown in the figure. A block rests on the in-
clined surface of the wedge and it keeps from sliding down when the turntable maintains
its rotation at a constant angular velocity. The coefficient of static friction between the
Chapter 5 Application of Newton’s Law 84

inclined surface and the block is µs = 1/4. The block remains at a position 40 cm from
the center of rotation of the turntable. Find the minimum angular velocity ω. Please
follow the below methods.

Method 1: Consider the motion of the block relative to an inertial observer

(a) Write down the equations of motion of the block along and normal to the horizontal.

(b) Find ω.

Method 2: Consider the motion of the block relative to an observer on the


rotating wedge

(a) Write down the equations of motion of the block along and normal to the inclined
surface.

(b) Find ω.

Solution
Method 1: Consider the motion of the block relative to an inertial observer
Chapter 5 Application of Newton’s Law 85

(a) Let N and f be the normal force and frictional force exerted on the block. For limiting
case, we have f = µs N . The equations of motion along the horizontal and vertical are

N sin α − µs N cos α = mω 2 r
N cos α + µs N sin α = mg

(b) The above equations become

N (sin α − µs cos α) = mω 2 r (5.19)


N (cos α + µs sin α) = mg (5.20)

Eliminating N from equations (5.19) and (5.20), we have


sin α − µs cos α ω2r
=
cos α + µs sin α g
g (sin α − µs cos α)
ω2 =
r (cos α + µs sin α)
2 9.81 ( 35 − 14 · 45 )
ω = 2 4
( + 14 · 35 )
5 5
ω 2 = 10.3 rad2 s−2
ω = 3.2 rad s−1

Method 2: Consider the motion of the block relative to an observer on the


rotating wedge

(a) The equations of motion relative to an observer on the rotating wedge are

Along the incline : mg sin α − µs N − mω 2 r cos α = 0 (5.21)


Normal to the incline : N − mg cos α − mω 2 r sin α = 0 (5.22)

where mω 2 r is the fictitious force which points outward horizontally. One should note
that the direction of a fictitious force is opposite to the acceleration of the observer.
Chapter 5 Application of Newton’s Law 86

(b) Multiplying µ to equation (5.22) and adding the result to equation (5.21), we eliminate
N , then

mg (sin α − µs cos α) = mω 2 r (cos α + µs sin α)


g (sin α − µs cos α)
ω2 =
r (cos α + µs sin α)

Similarly, we obtain ω = 3.2 rad s−1 .


Chapter 6

Impulse and Momentum

6.1 Definition of linear momentum


v
m p = mv

Newton’s 2nd law stated in momentum:


The rate of change of the momentum of a body is equal
to the total external force acting on the body. F3
X d~p
i.e. F~i =
i
dt

If the mass of the body is unchanged, m F2


X d~v F1
F~i = m = m~a
i
dt

6.2 Collision between bodies


In a collision between particles, even though we do not know the force acting on the
particles during the process, we can still relate the motions before and after the collision
whenever there is no external force acting on the system. For example, the mass and
momentum can transfer to each other.

m1~v1 + m2~v2 = m01 v~0 1 + m02 v~0 2

87
Chapter 6 Impulse and Momentum 88

6.3 Impulse and momentum


Consider two bodies in collision and pay attention to one of these two bodies.
d~p
It experiences a force, F~ = , during the collision.
dt
F

t
∆t
ti tf

For the small time interval dt,

d~p = F~ dt
Z p
~f Z tf
⇒ d~p = F~ dt
p
~i ti
Z tf
⇒ p~f − p~i = F~ dt
ti

~ It is the
The integral of the force over the time ti ≤ t ≤ tf is defined as the impulse J.
change of momentum of the object when the force acts on the object in the time interval.
def
J~ = ∆~p

On the other hand, we have

J~ = ∆~p = F~ave ∆t

Example
A particle of mass m is released from a height and it hits the floor with speed v1 . The
particle rebounds with speed v2 directly from the floor. The collision time is t. What is
the impulse exerted on the particle by the floor due to the collision? Please take upward
as positive.

Solution
Chapter 6 Impulse and Momentum 89

Let the force exerted on the particle during the collision be F . The total force acting on
the particle when collison occurs is
dv
F − mg = m
dt
(F − mg) dt = m dv
Z tf Z v2
(F − mg) dt = m dv
ti −v1
J − mg (tf − ti ) = m [v2 − (−v1 )]
J − mg t = m [v1 + v2 ]
J = m [v1 + v2 ] + mgt

Example
Three particles A, B and C, of masses 4, 6 and 8 kg, respectively, lie at rest on a smooth
horizontal table. They are connected by taut light inextensible strings AB and BC and
∠ABC = 120o . An impulse I is applied to C. If the magnitude of I is 88 Ns and it acts
in the direction BC, find the initial speeds of A, B and C.

Solution

Let I1 and I2 be the impulsive tensions (impulse) in the strings BC and AB respectively.
Since A is acted on only by I2 , its initial speed u will be in the direction of AB. Since
the string AB is taut, B must have a speed u in the direction AB. Also, let B have a
speed v in a direction perpendicular to AB [this is necessary because I1 acts in a different
direction to I2 ]. Finally, since both I and I1 , the impulses acting on C, have the same
direction BC, let C have an instantaneous speed V in the direction BC.
Since BC is taut, the speeds of B and C in the direction BC are equal.

o o u 3v
u cos 60 + v cos 30 = V ⇒ + =V (6.1)
2 2
Chapter 6 Impulse and Momentum 90

Considering the motion of A,

4u = I2 (6.2)

Considering the motion of B along and perpendicular to AB,


I1
6u = I1 cos 60o − I2 ⇒ 6u = − I2 (6.3)
2

3I1
6v = I1 cos 30o ⇒ 6v = (6.4)
2
Considering the motion of C,

8V = 88 − I1 (6.5)

Eliminating I2 from equations (6.2) and (6.3)

I1
6u = − 4u ⇒ I1 = 20u (6.6)
2
Substituting from equation (6.6) into equation (6.4),

20 3u 5u
6v = ⇒ v=√ (6.7)
2 3
Substituting from equation (6.7) into equation (6.1),
√ !
u 5 3u
+√ =V ⇒ V = 3u (6.8)
2 3 2

Substituting for V and I1 [equations (6.6) and (6.8)] into equation (6.5),

8(3u) = 88 − 20u ⇒ u=2


10
From (6.7), v = √ .
3
From (6.6), V = 6.
Speed of A is 2 m/s. r r
√ 100 7
2 2
Speed of B is u + v = 4 + =4 m/s.
3 3
Speed of C is 6 m/s.

Example
A smooth ring P of mass m is threaded on to a fixed horizontal wire OX and a second
smooth ring Q of equal mass is threaded on a fixed wire OY where Y is vertically below
Chapter 6 Impulse and Momentum 91

O. The rings are connected by a light inextensible string of length 2a. Initially P and Q

are at rest with P at a distance 3 a from O, and Q at O. Ring Q is then released, find
the speed of P and Q immediately after the string has become taut. You may assume
that the string keeps tight after the jerking.

Solution

p
When the string is taut, the line joining P Q becomes straight and OQ = (P Q)2 − (OP )2 =
q √
(2 a)2 − ( 3 a)2 = a. As the total energy of Q is conserved when it falls freely, we can
write
1 2
mga = mv ,
2 0

where v0 is the speed of Q just before the string becomes taut. We get v0 = 2ga.
Let I be the impulsive tension in the string when the string is just taut. Denote the speed
of P and Q just after the string is taut as vx0 and vy0 respectively.

Particle P : −I cos 30◦ = m (−vx0 − 0)


Particle Q: −I cos 60◦ = m (vy0 − v0 )
Chapter 6 Impulse and Momentum 92

So, we have
 √
 3 I = m v0

x
2
1 p
− I = m (vy0 − 2ga)


2
Eliminating I, we have

v0 p
− √x = vy0 − 2ga (6.9)
3
Let’s consider the speed of the particles along the string. We have vy0 cos 60◦ = vx0 cos 30◦
which gives

vy0 = 3 vx0 (6.10)

6ga 3p
Solving equations (6.9) and (6.10), we get vx0 = and vy0 = 2ga.
4 4

6.4 Conservation of linear momentum


Rather than looking into the experience of one of the body in the collision process, we
look into the total momentum of the two bodies before and after the collision.

• Consider two particles having initial


Fe1
momentum p~i1 and p~i2 before the col-
lision.
Pi1
F12 F21
m1 m2 Pi2 • During the collision process, the im-
pulse force acting on m1 from m2 is
Fe2
F~12 and that on m2 from m1 is F~21 .
Before collision
• They are also experiencing external
forces of F~e1 and F~e2 .

As F~12 and F~21 are action-reaction pair.

F~12 = −F~21

Before the collision, total momentum of the system:

~ i = p~i1 + p~i2
P
Chapter 6 Impulse and Momentum 93

In the collision time interval ∆t,

∆~p1 = (F~12 + F~e1 )∆t and ∆~p2 = (F~21 + F~e2 )∆t

where ∆~pj is the change in momentum of mass mj during the collision.


∴ After the collision, final momentum of m1 and m2 :

p~f 1 = p~i1 + ∆~p1 = p~i1 + (F~12 + F~e1 )∆t


p~f 2 = p~i2 + ∆~p2 = p~i2 + (F~21 + F~e2 )∆t

∴ After the collision, total momentum of the system:

~ f = p~f 1 + p~f 2 = p~i1 + p~i2 + (F~e1 + F~e2 ) ∆t + (F~12 + F~21 ) ∆t


P
| {z } | {z }
~e,tot
=F =0

~
Hence, ~f − P
P ~ = F~e,tot ∆t ⇒ ∆P = F~e,tot
~ i = ∆P
∆t

To generalize the case, consider a N particles system con-


Fei
sisting of masses m1 , m2 , . . . , mN .
Define: P~ = P mi~vi .
i
For each particle mi , it experiences an external force F~ei
and other forces arising from its neighbors mj , i.e. F~ij .

m1 ~
dP X
∴ = F~e,i = F~e,tot
Fi1 dt i
m2

Fi3 mi Fi2 Only the external forces, but NOT the internal forces,
m3 play roles on changing the total momentum of the system.
FiN
mN

Moreover, if F~e,tot = 0, then


dP~
=0
dt
i.e. if the total external force acting on a system of particles is zero, the total momentum
of the system conserves.
Chapter 6 Impulse and Momentum 94

6.5 Collision of particles in two dimensional space

u1
y m1 m2
u2 Before collision

v1
x m1 m2 v2 After collision

If there is no external force acting on the system, we can write

m1 u1x + m2 u2x = m1 v1x + m2 v2x


m1 u1y + m2 u2y = m1 v1y + m2 v2y

Example
A particle of mass m is released on the smooth inclined face of a wedge of mass 2m and
inclination angle α, which is initially at rest and is free to move on a smooth horizontal
plane. Find the ratio of the acceleration of the wedge to that of the particle relative to
the wedge when the particle is sliding down.

Solution

Let the velocity of the wedge be V and that of the particle relative to the wedge be u.
Since there is no horizontal force acting on the system, by the conservation of horizontal
momentum, 2mV + m(V − u cos α) = 0.
u cos α
∴ V =
3
Differentiate both sides with time, we have

a0 cos α
a = ,
3
Chapter 6 Impulse and Momentum 95

dV
where the acceleration of the wedge, a = and the acceleration of the particle relative
dt
du
to the wedge, a0 = . Therefore, we have a : a0 = cos α : 3.
dt

Example
A small block of mass M is placed on the top of a fixed and smooth sphere which has
radius R. A moving particle of mass m and speed v0 collides with the block horizontally,
it sticks on the block and the composite slides down the sphere.

(a) Find the angle θ when the composite leaves the sphere, where θ is measured from
the vertical to the line joining the composite and the center of sphere.

(b) Find the minimum initial speed of the particle such that the composite so formed
leaves the sphere immediately when the collision occurs.

Solution

(a)

Let V be the instantaneous speed of the composite when the particle is just embed-
ded into the block. Consider the conservation of linear momentum in the horizontal:

mv0 = (m + M )V (6.11)
mv0
⇒ V = (6.12)
M +m

Let v be the instantaneous speed of the composite when it is just leaving the sphere.
Consider the conservation of mechanical energy:
1 1
(M + m)V 2 + (M + m)gR(1 − cos θ) = (M + m)v 2 (6.13)
2 2
Chapter 6 Impulse and Momentum 96

The composite performs the circular motion on the sphere:

v2
(M + m)g cos θ − N = (M + m) (6.14)
R
The composite leaves the sphere when N = 0. From Eq. (6.14),

Rg cos θ = v 2 (6.15)

From Eq. (6.13) and Eq. (6.15), we obtain


1 2 1
V + gR(1 − cos θ) = Rg cos θ
2 2
V 2 + 2gR
⇒ cos θ = (6.16)
3gR

Using Eq.(6.12) and Eq.(6.16) gives

m2 v02 2
⇒ cos θ = 2
+
3gR(M + m) 3

(b) For the composite to leave the sphere when it is at the top of sphere:

v2

(M + m) g cos θ − N = (M + m)

R
 N ≤ 0, when θ = 0

V2
∴ ≥ g
R
m2 v02
⇒ ≥ gR
(M + m)2

gR (M + m)
⇒ v0 ≥
m

Example
Suppose that a boy stands at one end of a boxcar sitting on a railroad track. Let the
mass of the boy and the boxcar be M . He throws a ball of mass m with velocity ~v0
(relative to the ground) toward the other end, where it collides elastically with the wall
and travels back down the length (L) of the car, striking the other side perfectly inelas-
tically. If there is no friction in the wheels of the boxcar, describe the motion of the boxcar.
Chapter 6 Impulse and Momentum 97

Solution

Along the horizontal, no external force is acting on the system which consists the boxcar,
the boy and the ball. Therefore, if V~ and ~v are the velocity of the boxcar (also the
boy) and the velocity of ball respectively (both quantities are relative to the ground), the
conservation of linear momentum gives
m
M V~ + m~v = 0 ⇒ V~ = − ~v (6.17)
M
at all times.
Before the first collision, ~v = ~v0 and so
m L
V~ = − ~v0 for 0<t<
M (1 + m/M )v0

where we have on the right the time expression for the ball to reach the boxcar wall,
traveling at speed
m  m
v0 + V = v0 + v0 = 1 + v0
M M
with respect to the floor of the boxcar.
The effect of the first elastic collision will be simply to reverse both velocity vectors in
eq. (6.17). Thus, ~v = −~v0 and

m L 2L
V~ = + ~v0 for <t<
M (1 + m/M )v0 (1 + m/M )v0

Finally, after the second collision (i.e. perfectly inelastic), the ball and boxcar have
common velocity. Hence,
2L
V~ = 0 for t>
(1 + m/M )v0
Chapter 6 Impulse and Momentum 98

Notice that a nonzero common velocity will move the center of mass of the system, which
is not allowed as there is no external force acting on the system.
It is seen that the boxcar first moves to the left a distance
m   
L m
v0 = L
M (1 + m/M )v0 m+M
and then moves an equal distance to the right, coming to rest at its starting point. This
result — that there is no net displacement of the boxcar if the ball return to its initial
position within the boxcar — holds whether or not the first collision is elastic (because
the center of mass of the system must remain at rest).

6.6 Coefficient of restitution (e)

The above figure depicts two bodies. Since the total momentum is conserved, we have

m1 u1 + m2 u2 = m1 v1 + m2 v2

This one equation is not sufficient to calculate v1 and v2 and we have recourse to Newton’s
experimental law. If the velocities both before and after impact are taken relative to the
same body, then, for two bodies impinging directly, their relative velocity after impact
is equal to a constant (e) times their relative velocity before impact and in the opposite
direction. e is known as the coefficient of restitution.

v1 − v2 = −e (u1 − u2 )

In the case of oblique impact, the result holds for the components the velocities in the
direction of the common normal at impact. The value of e has to be found by experiment
and varies from 0 for completely inelastic bodies to practically 1 for nearly perfectly elas-
tic bodies. Note that the quantities u1 , u2 , v1 , and v2 mentioned above are in the same
direction.
Chapter 6 Impulse and Momentum 99

Example
Three identical spheres are arranged as shown in figure. If sphere C is projected with
velocity u while A and B are at rest. Given that the coefficient of restitution is e for each
sphere, find the subsequent velocities of each sphere. Show also that the condition for
sphere C to pass through and beyond the two spheres A and B is e < 1/9.

Solution
Let u be the velocity of sphere C before impact, v the velocity of sphere C after impact
and w the velocities of A and B after impact.
By conservation of momentum:

mu = mv + 2 mw cos 30◦ =⇒ v + 2 w cos 30◦ = u (6.18)

By Newton’s law of restitution:


w − v cos 30◦
e=− ◦
=⇒ w − v cos 30◦ = eu cos 30◦ (6.19)
0 − u cos 30

u 3u
Solving the above equations, we get v = (2 − 3 e) and w = (1 + e).
5 5
u
Thus, the velocity of C after impact is (2 − 3 e) and the velocities of A and B after
√ 5
3u
impact are the same as (1 + e).
5

If the sphere C passes through and beyond the two spheres A and B, then
√ √
◦ u 3u 3
v > w cos 30 =⇒ (2 − 3 e) > (1 + e) ·
5 5 2
1
4 − 6 e > 3 + 3 e =⇒ e < .
9
Chapter 7

Systems of Particles

In previous chapters, we have deal with problems of point mass or particle. Now we turn
our focus to system containing many particles, e.g. rigid body.

7.1 Center of mass


y
m3
m1
Consider a system containing N particles
r3 r1
r2 m1 , m2 , . . . , mN .
m2
x ~ri (t) : position of mi at time t
~vi (t) : velocity of mi at time t
rN ~ai (t) : acceleration of mi at time t

mN

Definition
P
m1~r1 + m2~r2 + . . . + mN ~rN imi~ri
Center of mass ~rCM = =
m1 + m2 + . . . + mN M
P
where M = mi and mi /M is the weighting function of ~ri . The components of ~rCM are
i
P P P
mi x i m i y i mi zi
xCM = i , yCM = i , and zCM = i
M M M
Example
A uniform disk has a hole in it as shown in the figure. Show that the hole can be considered
as a negative mass when we calculate the center of mass of the disk.

100
Chapter 7 Systems of Particles 101

Solution

Denote the following quantities.


mA : The mass of the hole if it is filled.
mB : The mass of the shaded region in the disk.
mC : The mass of the disk with the hole being filled.

xA : The CM of the hole if it is filled.


xB : The CM of the shaded region in the disk.
xC : The CM of the disk with the hole being filled.

Notice that mC = mA + mB .
mA xA + mB xB
xC =
mA + mB
So
mA xA + mB xB
xC =
mC
xC mC = mA xA + mB xB
xB mB = mC xC − mA xA
mC xC − mA xA
xB =
mB
mC xC − mA xA
Hence, we have xB = . The hole is regarded as a negative mass.
mC − mA
mC xC + (−mA ) xA
xB =
mC + (−mA )
Chapter 7 Systems of Particles 102

7.2 Motions of the Center of mass


According to the definition of center of mass
P
m1~r1 + m2~r2 + . . . + mN ~rN i mi~ri
~rCM = =
m1 + m2 + . . . + mN M

Differentiating both sides, we have


d~rCM 1
~vCM = = (m1~v1 + m2~v2 + . . . + mN ~vN )
dt M
d~vCM 1
~aCM = = (m1~a1 + m2~a2 + . . . + mN ~aN )
dt M
Hence,
M~aCM = m1~a1 + m2~a2 + . . . + mN ~aN = F~1 + F~2 + . . . + F~N (7.1)

F~i is indeed the total force experienced by mass mi .

F~i = F~int,i + F~ext,i

where F~int,i — total internal force acting on mi originated from other particles mj6=i ,
F~ext,i — total external force acting on mi .
m1 F1i F~int,i = F~i1 + F~i2 + . . . + F~iN (no F~ii )
Fi1 m2
mi F2i ∴ From (7.1),
Fi2
FiN M~aCM
F~ext,1 + ( F~12 + F~13 + . . . + F~1,N −1 + F~1N )
FNi
mN + F~ext,2 + ( F~21 + F~23 + . . . + F~2,N −1 + F~2N )
= + F~ext,3 + ( F~31 + F~32 + ... + F~2,N −1 + F~2N )
+ ...
Fext,i
+ F~ext,N + ( F~N 1 + F~N 2 + F~N 3 + . . . + F~N,N −1 )

Notice F~ij = −F~ji and thus


X
M~aCM = F~ext,1 + F~ext,2 + . . . + F~ext,N = F~ext,i
i

i.e. to say N -particle system with external forces acting on individual particles and in-
ternal forces between each of the particle behaves as if a single point mass at the
P ~
position ~rCM experiencing a force of Fext,i . i
Chapter 7 Systems of Particles 103

Example

1N
CM
2N A uniform laminar of 0.5-kg mass is acting on by some forces.
X
F~ = −î + 2 î − 3 ĵ = î − 3 ĵ = (0.5) ~aCM
3N
y 1 −3
∴ ~aCM,x = = 2 ms−2 , ~aCM,y = = −6 ms−2
0.5 0.5

(Only motion of center of mass is known, but not the rotation.)


x

Example

11
00 m1 at t0 + ∆ t A particle of mass m0 explodes at
00
11
x y
( 0 , 0)
time t0 into two masses of m1 and
explode at t0 m2 . A short time ∆t after the ex-

v0 plosion, m1 was found at (x0 , y0 ).


projectile
1
0m2 at motion Find the position of m2 at t0 + ∆t.
φ0 0t0 +∆ t
1
0
1 x
m1
0
0

Consider m1 and m2 are pieces to form the system after explosion and we assume that
m2 does not hit the ground in the discussion.
Before the explosion, the CM of the system is just the position of the mass m0 . The
trajectory of it is a parabola given by ~rCM and the gravitational acceleration g points
vertically downward.

d2~rCM
∴ ~aCM = = −g ĵ
dt2
d~rCM
⇒ ~vCM = = (v0 cos φ0 ) î + (v0 sin φ0 − gt) ĵ
dt
1
⇒ ~rCM = (v0 cos φ0 ) t î + [(v0 sin φ0 ) t − gt2 ] ĵ
2
After the explosion, m0 splits into m1 and m2 . Therefore, m1 and m2 have their own
positions ~r1 (t) and ~r2 (t). But as the explosion only involves internal forces, the CM’s
position will follow the original parabola.
Chapter 7 Systems of Particles 104

At t = t0 + ∆t,
m1~r1 + m2~r2
~r1 (t0 + ∆t) = x0 î + y0 ĵ and ~rCM =
m0
m0 m1
⇒ ~r2 (t0 + ∆t) = ~rCM − ~r1
m m2
2     
m0 m1 m0 1 2 m1
= v0 cos φ0 t − x0 î + v0 sin φ0 t − gt − y0 ĵ
m2 m2 m2 2 m2
Example
A particle of mass m is released at the top of a wedge of height h, mass M and inclination
angle α. The wedge is initially at rest and is free to move on a smooth horizontal plane.
Find the displacement of the wedge when the particle reaches the bottom of the wedge.

Solution
Consider a system formed by the particle and the wedge. The center of mass of the
system has no horizontal displacement because there is no net horizontal force exerted on
the system externally. In fact, the frictional force (if the inclined surface is not smooth)
and the normal force acting between the particle and the wedge are internal forces.

Let the displacement of the wedge be x when the particle reaches the horizontal plane.
The corresponding horizontal displacement of the particle relative to the horizontal plane
is x − h cot α, so

M x + m (x − h cot α) = 0 (7.2)
mh cot α
x =
m+M
Remark:
Equation (7.2) is obtained because for a system of two objects, say m1 and m2 , if x1 and
x2 are the x-coordinates of the center of mass of objects 1 and 2 respectively, then the
center of mass of the system is given by m1 x1 + m2 x2 = (m1 + m2 ) xCM which implies

m1 ∆x1 + m2 ∆x2 = (m1 + m2 ) ∆xCM

where ∆x1 and ∆x2 represent the displacements of the center of mass of the objects. The
displacement of the center of mass of system is denoted by ∆xCM .
Chapter 7 Systems of Particles 105

Example

y
A solid ball with radius R1 is placed inside a
hollow sphere with radius R2 , as shown in the
figure. The ball is then released both the ball
1111
0000
0000
1111
R1
R2
x
and the sphere roll back and forth. What is the

g 0000
1111
0000
1111
final equilibrium position? The masses of the
ball and the sphere are both m.

ground Consider motion in x-direction, as external force


is zero, the x component of the CM does not
change.
At final state Before release,
y
m × 0 + m × (R1 − R2 )
xCM =
2m
R1 − R2
=
2

1111
0000
0000
1111
x After reaching equilibrium,

0000
1111 m × x0 + m × x0
0000
1111
xCM =
2m
0000
1111 ground = x0 =
R1 − R2
x0 2

7.3 Center of mass of some rigid bodies


A rigid body can be considered as a system of infinite number of particles. A rod, a disk,
and a solid sphere are examples of rigid bodies. The center of mass of a rigid body is
expressed by an integral instead of using summation notation.
Z
~r dm
~rCM = Z
dm

• If the object is a wire, dm = µ dl, where µ is the mass per unit length.

• If the object is a laminar, dm = σ dA, where σ is the mass per unit area.
Chapter 7 Systems of Particles 106

• If the object is a volume, dm = ρ dV , where ρ is the mass per unit volume.

Remark
Z Z
~v dm ~a dm
~vCM = Z and ~aCM = Z
dm dm

Example
Locate the center of mass of a triangular laminar, as shown in the figure. The laminar
has uniform mass distribution.

Solution
Imagine the laminar is a collection of rods which are parallel to BC. The CM of the
laminar lies on the line joining the CMs of rods. This line is the median of the triangle.
Likewise, imagine again the laminar is a collection of rods which are parallel to AC. Thus,
the CM of the laminar lies on the line joining the CMs of these rods. Therefore, the CM
of laminar ABC is located at the intersection of the medians.

From the knowledge of simple geometry, we know that the medians of ∆ABC meet at a
point G, the centroid of the triangle, and
2
AG = AE
3
2
BG = BF
3
2
CG = CD
3
Chapter 7 Systems of Particles 107

Remark:
To locate the CM of a semi-circular disk. One may divide the disk to many small sectors
which are considered as isosceles triangles having small subtended angle at the center of
disk.

Example

(a) Locate the center of mass of a uniform semi-circular arc C which has radius R.

(b) A uniform semi-circular disk D has radius R. Use the result in (a) to locate the center
of mass of D.

(c) Use the result of (b) to deduce the answer for (a) again.

(d) Locate the center of mass of D by considering the object as a collection of many small
sectors subtended from the disk centre.

Solution
(a)

The wire is symmetric about the y-axis and thus the CM of it must lie on the y-axis.
Consider the small arc segment as shown in the figure. It is at a distance y above the
x-axis, where
y = R sin φ

The mass of the segment is


dm = λR dφ

where λ is the density of the wire (mass per unit length). The mass of the arc is MC = λπR.
The y-coordinate of the center of mass of C is

R π
R
(R sin φ) (λR dφ)
Z
y dm 0 2R
ȳC = = = sin φ dφ =
MC λπR π 0 π
Chapter 7 Systems of Particles 108

(b) Let σ be the surface density (i.e. mass per unit area) of D. The mass of the semi-
circular disk D is
πR2
 
MD = σ .
2
Consider D as a collection of numerous arcs each having an infinitesimal width dr, then
the mass of each arc is dm = σπr dr.

The y-coordinate of the center of mass of D is


Z
y dm
ȳD =
MD
Z R
2r
(σπr dr)
0 π
=  2
πR
σ
2
Z R
4
= r2 dr
πR2 0
4R
=

(c) Let A be a semi-circular arc of finite width as shown in the figure.
Chapter 7 Systems of Particles 109

The y-coordinate of the center of mass of the semi-circular arc A is


 2  2
4R πR 4R1 πR1
·σ − ·σ
3π 2 3π 2
ȳA =  2  2
πR πR1
σ −σ
2 2
2σ 3
(R − R13 )
= πσ 3
(R2 − R12 )
2
4 R3 − R13
=
3π R2 − R12

The y-coordinate of the center of mass of the semi-circular arc C is

ȳC = lim ȳA


R1 →R
4 R3 − R13
= lim 2
3π R1 →R R − R12
4 R2 + RR1 + R12
= lim
3π R1 →R R + R1
2
4 3R
= ·
3π 2R
2R
=
π
(d) Divide D into numerous sectors each having a subtended angle dφ. The mass of the
1 2
sector is dm = σR2 dφ and the center of mass of it has a y-coordinate given by R sin φ.
2 3
Chapter 7 Systems of Particles 110

Hence, the y-coordinate of the center of mass of D is


Z
y dm
ȳD =
M
Z πD  
2 1 2
R sin φ σR dφ
0 3 2
=  2
πR
σ
2
Z π
1
σR3 sin φ dφ
3 0
=  2
πR
σ
2
π
2R
= − cos φ
3π 0
4R
=

Example
A uniform laminar of mass m has an area bounded by y = x2 and y = 2. Locate the
center of mass of the laminar.

Solution

Let the center of mass be (x̄, ȳ) and the surface density be σ. We notice that x̄ = 0,
because the laminar has symmetry about the y-axis. By the definition of center of mass,

R
y dm
we have ȳ = R , where dm = σ(2x) dy, 2 ≥ x ≥ 0. Rewrite the expression in terms
dm
Chapter 7 Systems of Particles 111


of y, we have dm = σ(2 y) dy, 2 ≥ y ≥ 0.
Z Z 2
3
y dm = 2σ y 2 dy
0
5
! 2
y 2
= 2σ 5
2

0
4σ  5

= 22
5

Z Z 2
1
dm = 2σ y 2 dy
0
3
! 2
y 2
= 2σ 3
2

0
4σ  3

= 22
3
Substitute the answers of both intergrals into the expression of ȳ, we obtain
4σ  5 
22 6
ȳ = 5   =
4σ 3 5
22
3
6
Therefore, the center of mass of this laminar is (0, ). Note also that if m is the mass of
5
R 4σ  3  3m
the laminar, m = dm = 2 2 and we can relate σ = √ though it is not required
3 8 2
in this problem.

Example
A rod of length 1 m has a non-uniform mass distribution on its length. The mass per
unit length of it varies with position according to ρ = ρ0 (1 − x/2), where x is measured
from one end of it along the rod. Locate the center of mass of the rod.

Solution R
x dm x
From the definition of CM, we have x̄ = R , where dm = ρ dx = ρ0 (1 − ) dx. Thus,
dm 2
we obtain
Chapter 7 Systems of Particles 112

Z 1
x
x ρ0 (1 − ) dx
2
x̄ = Z0 1
x
ρ0 (1 − ) dx
0 2
1
x2 x3
Z 1
x2 1 1
(x − ) dx ( − ) ( − )
2 2 6 4
= Z0 1 = 10 = 2 16 =
x 2
x 9
(1 − ) dx (x − ) (1 − )
0 2 4 0 4

4
Therefore, the CM is m from the heavier end of the rod.
9

Example

(a) Use direct integration to locate the center of mass of a uniform hemispherical shell
which has mass m and radius r.

(b) By using the result in (a), locate the center of mass of a uniform solid hemisphere
which mass m and radius r.

Solution

(a) The mass of small ring, as shown in the figure, is given by

width of ring
z }| {
dm = σ (2πr sin θ) (rdθ) = 2πσr2 sin θ dθ
| {z }
circumference of ring

where σ is the surface density of the hemisphere.


Chapter 7 Systems of Particles 113

Therefore,
R R
xdm r cos θ dm
x̄ = R =
dm 2πr2 σ
Rπ Z π
0
2
r cos θ (2πσr2 sin θ dθ) 2
= = r sin θ cos θ dθ
2πr2 σ 0
Z π π
2 r sin2 θ 2 r
= r sin θ d sin θ = =
0 2
0 2
r
That is, x̄ = . Due to symmetry of the hemisphere about the x-axis, ȳ = 0, thus,
2
r
we have (x̄, ȳ) = ( , 0).
2
(b) Let the mass of a spherical shell with width dx, as shown in the figure, be dm,
where dm = ρ (2πx2 dx) and ρ is the volume density of the hemisphere.
| {z }
volume of spherical shell
Chapter 7 Systems of Particles 114

Z r
x
Z
x
( ) dm ( ) (2πρx2 dx)
Z2 2
x̄ = = 0
2 3
dm πr ρ
3
Z r
3 3 3 r4 3r
= x dx = ( ) =
2r3 0 2r3 4 8
3r
That is, x̄ = . Due to symmetry of the hemisphere about the x-axis, ȳ = 0, thus,
8
3r
we have (x̄, ȳ) = ( , 0).
8
Example
A uniform solid cone of mass m has base radius r and height h.

(a) Find the center of mass of the cone measured from the base.

(b) A uniform conical hollow of the same mass, radius and height has no cap. Use the
result of (a), find the center of mass of it.

(c) Repeat part (b) by direct integration.

Solution
1
(a) Let the density and the mass of the cone be ρ and M respectively, where M = ρπr2 h.
3
Divide the cone into numerous disks, each having an infinitesimal small thickness dx.

r y r
From the figure, we note that the ratio = , then y = x. Thus, the mass of each
h x h
small disk is
ρπr2
 
2
dm = ρπy dx = x2 .
h2
Chapter 7 Systems of Particles 115

The center of mass of the cone is


Z
x dm
x̄ =
M
The numerator of it is
ρπr2 h 3
Z Z
x dm = x dx
h2 0
ρπr2 h4
= ·
h2 4
1
= ρπr2 h2
4
So
Z
1
x dm ρπr2 h2
x̄ = = 4
M 1
ρπr2 h
3
3
= h
4
1
Hence, the center of mass of the cone is located at h from the base of the cone.
4
(b) Let the semi-angle of the cone be θ as shown in the figure. A hollow cone of finite
thickness can be considered as a cone which has a big hole in it. The hole is in the form
of a similar cone of height h1 . Recall that a hole can be regarded as negative mass when
we calculate the center of mass of an object having a hole. Let the center of mass of the
hollow cone be x̄0 , then
   
1 1 2 1 1 2
h ρπr h − h1 ρπr1 h1
4 3 4 3
x̄1 = 1
3
ρπr2 h − 31 ρπr12 h1
1 r2 h2 − r12 h21
= ·
4 r2 h − r12 h1
1 h4 tan θ − h41 tan θ
= · 3 (r = h tan θ and r1 = h1 tan θ)
4 h tan θ − h31 tan θ
1 h4 − h41
= · 3
4 h − h31
Chapter 7 Systems of Particles 116

A hollow cone has infinitesimal small thickness, so

1 h4 − h41
lim x̄1 = lim ·
h1 →h h1 →h 4 h3 − h31
1 (h − h1 ) (h + h1 ) (h2 + h21 )
= lim ·
h1 →h 4 (h − h1 ) (h2 + h h1 + h21 )
1 (h + h1 ) (h2 + h21 )
= lim ·
h1 →h 4 (h2 + h h1 + h21 )
1
= h
3
(c) We compute the centre of mass of the hollow cone by direct integration. Let’s divide
the hollow into numerous rings, each
 of radius  y, as shown in the figure. The mass of each
2 πrxσ sec θ r
ring is dm = 2 πy σ (sec θ dx) = dx. We should note that y = x and
h h
the thickness of the ring is sec θ dx instead of dx.
Chapter 7 Systems of Particles 117

dx
We can see that if the ring is cut and opened, it is an arc of thickness = sec θ dx.
cos θ
From the definition of center of mass, we have
Z
x dm
x̄ = Z
dm

The numerator of the above fraction is


2 πrσ sec θ h 2
Z Z
x dm = x dx
h 0
2 πrσ sec θ h3
= ·
h 3
The denominator of equation (7.3) is
2 πrσ sec θ h
Z Z
dm = x dx
h 0
2 πrσ sec θ h2
= ·
h 2
Finally, we obtain
Z
x dm
2
x̄ = Z = h
3
dm

Example
A pulley has two masses m1 and m2 hanging over it by a string, as shown in the figure,
where m2 > m1 . Assume that the pulley and all strings are massless and frictionless,
please find the CM acceleration of the pulley system when the masses are released from
rest.

Solution
The pulley system is enclosed in the green box, as shown in the figure.
Chapter 7 Systems of Particles 118

Taking downward as positive and consider Newton’s second law of motion, we have Fnet =
m aCM , where m = m1 +m2 and Fnet = (m1 +m2 )g−R. Need to mention that the tensions
of the string are internal forces of the system and they do not appear in the net force
expression. For the motion of individual masses, we can relate the acceleration of them
by
(
T − m1 g = m1 a
(7.3)
m2 g − T = m2 a

Solving the above equations, we otain


2m1 m2 g
T =
m1 + m2
But, R = 2T because the pulley is massless. Therefore, we have
4m1 m2 g
R=
m1 + m2
The net force on the pulley system, Fnet = m aCM becomes
4m1 m2 g
(m1 + m2 )g − = (m1 + m2 ) aCM
m1 + m2
g (m2 − m1 )2
⇒ aCM =
(m1 + m2 )2
Chapter 7 Systems of Particles 119

Remark:
Using the definition of CM, we can write m1 x1 + m2 x2 = (m1 + m2 ) xCM . Hence,

m1 ẍ1 + m2 ẍ2 = (m1 + m2 ) ẍCM


−m1 a + m2 a = (m1 + m2 ) aCM
 
m1 + m2
a = aCM
m2 − m1
m1 + m2 g (m2 − m1 )2
 
a =
m2 − m1 (m1 + m2 )2
g (m2 − m1 )
a =
m1 + m2
The last expression is the same as that obtained by solving equation set (7.3).

7.4 Momentum of system of particles


Consider a system of N particles m1 , m2 , . . . , mN having position vectors ~r1 , ~r2 , . . . , ~rN .

Total momentum of the system


X X
~ =
P mi~vi = p~i (7.4)
i i
P
i mi~ri
But since ~rCM = .
M

d~rCM X d~ri
⇒ M = mi
dt i
dt
X
⇒ M~vCM = p~i (7.5)
i

Combining (7.4) and (7.5)

~
dP
~ = M~vCM
P and = M~aCM
dt

To find the total momentum, other than adding all p~i , we can also get it by finding M~vCM .
Or the system of N particles behaves as if it is a point mass having mass M , velocity ~vCM
and acceleration ~aCM .
Chapter 7 Systems of Particles 120

Moreover, from last chapter or the recall at the beginning of this chapter:
~
dP X
= F~ext,i = F~ext,tot
dt i

Therefore,
X
M~aCM = F~ext,i
i

~
dP
If the total external force is zero, we have ~ = 0. The direct result
= 0, or simply, ∆P
dt
~ = constant. This is referred to as the conservation of linear momentum for
of this is P
system of particles!

Example

vCE vmc
m A canon on a frictionless ground fires
M
a cannon ball. The canon ball is fired
with speed of vmc relative to the canon.

In x-direction, there is no external force.


∴ Momentum conserved which implies

0 = M vcE + mvmE

But vmE = vmc + vcE

⇒ 0 = M vcE + mvmc + mvcE


−mvmc
⇒ vcE =
m+M
Chapter 7 Systems of Particles 121

7.5 System of variable mass


Time t
• At t + ∆t, a small mass was
v system ejected out from the original
m being
mass.
studied

• ~u is the velocity of the small


mass relative to the Earth.
Time t +∆t However, it should be noted
that the small mass velocity is
u v+∆v system usually given in relative to the
−∆ m m+ ∆ m being
studied original mass.

At time t, total momentum:


~
P(t) = m~v

At time t + ∆t, total momentum:

~ + ∆t) = (m + ∆m)(~v + ∆~v ) + (−∆m)~u


P(t
= m~v + m∆~v + ∆m~v + ∆m∆~v − ∆m~u
= m~v + m∆~v + ∆m(~v − ~u) + ∆m∆~v

Hence, we find

~ = P(t
∆P ~ + ∆t) − P(t)
~ = m∆~v + ∆m(~v − ~u) + ∆m∆~v
~
∆P d~v dm
⇒ F~ext = lim =m + (~v − ~u)
∆t→0 ∆t dt dt

d~v dm
or F~ext = m − ~vrel
dt dt

where ~vrel = ~u − ~v is the velocity of the small particle relative to the original mass.
Sometimes, this equation is referred to as the rocket equation.

Notes:
 
∆m∆~v
• lim = 0, since ∆m → 0, and ∆~v /∆t becomes finite when ∆t → 0.
∆t→0 ∆t
Chapter 7 Systems of Particles 122

• Since ~v = ~vmE and ~u = ~v∆m,E , we have

~v − ~u = ~vmE + ~vE,∆m = ~vm,∆m = −~v∆m,m = −~vrel


Example
momentum conserved

u mass = M
1
0 0
1 0
1 0
1 A train on a frictionless rail with a
0
1
01
0
01
0
01
0
1 1 1 0
1 v machine gun firing at a rate of n bul-
lets per second. Mass of bullet is m.

momentum conserved

u
A rocket in space ejecting mass at
v
M rate of | dM |.
dt

where ~u: velocity of ejecting mass or bullet relative to the Earth.

For both case, F~ext = 0.

d~v dM
∴M = (~u − ~v )
dt dt
 (−mn)(~u − ~v ) = mn(~v − ~u)
 for the train,
= 
dM

(~u − ~v ) = dM (~v − ~u) for the rocket

 −

dt dt

This implies both the train and the rocket will accelerate as if there were a force (called
thrust). Indeed, it is not a real external force acting on the train or the rocket but only
to maintain the conservation of momentum.
Chapter 7 Systems of Particles 123

Example

+ve
Rocket ascending on earth by ejecting mass at a rate

M g of | dM
dt
| with velcoity ~vrel relative to the rocket.

d~v dM
F~ext = M − ~vrel
dt  dt 
dv dM
dM ⇒ −M |g| = M − − (−|vrel |)
dt dt dt

dv dM
⇒ M = |vrel | − M |g|
dt dt

vrel

Example

∆M
M
Raindrop falling and water vapor keeps on con-
densing on it with a rate of | dM
dt
|.
v d~v dM
F~ext = M − ~vrel
dt dt
g d~v dM
F~ext = M − (~umoisture − ~vrain )
M dt dt
+ dv dM
∆M ⇒ M |g| = M − (0 − v) ∵ ~umoisture = 0
+ve dt dt

dv 1 dM
v+∆ v ⇒ + v = |g|
dt M dt

Example
A soft and uniform string of length l and mass M is hanged in the way that its lower end
is touching the table. This string is initially at rest and then suddenly released to fall
freely from the top position. Find the instantaneous reaction force that the table acts on
the string after it has fallen a distance y. Find also the reaction force when the whole
string is just on the table.
Chapter 7 Systems of Particles 124

Solution
Method 1: Consider the small mass element at the junction.
After the string is released, it moves down under the gravity. The speed of the string
becomes v when it moves down a distance y, where v 2 = 2gy.

Denote the line density of the string be λ. During the short time interval ∆t, the length of
string element being stopped on the table due to collision is ∆y. This small element is at
the junction between the upper portion (i.e. the moving part) and the lower portion (i.e.
the stationary part). An upward force F acts on the lower end of this moving element in
order to stop it, where F has the relation

∆p (λ ∆y) (0 − v)
F ∼ =
∆t ∆t
Hence, we have
 
dy
F = −λ v
dt
2
⇒ F = −λv ,

where v = dy/dt and F is negative. Notice that F is not the magnitude of force. According
to Newton’s three law of motion, there is a downward force, f = −F , which acts on the
lower portion. Thus, we obtain f = λv 2 = 2λgy. The stationary portion of the string on
the table (i.e. enclosed by the green box) is at equilibrium, hence, we have

R = λgy + f
= λgy + 2λgy
⇒ R = 3λgy

When y = l, we obtain R = 3λgl = 3M g, where M = λl is the mass of the string.


Chapter 7 Systems of Particles 125

d~p
Method 2: Consider F~net =
dt

d
Fnet = M g − R = [λ(l − y)ẏ]
dt
⇒ M g − R = λlÿ − λy ÿ − λẏ 2

Since ÿ = g, M = λl and ẏ 2 = 2gy. We have

λlg − R = λlg − λyg − 2λyg


⇒ R = 3λyg

When the whole sting is just on the table (i.e. y = l), R = 3M g.

Method 3: The rocket equation.

Consider the stationary portion of the string (enclosed by the green box) as the interested
object in the rocket equation.
d~v dm
F~ext = m − ~vrel
dt dt
becomes

−R + λyg = m(0) − (ẏ − 0) λẏ


⇒ −R + λyg = −ẏλẏ
⇒ R = λyg + λẏ 2
Chapter 7 Systems of Particles 126

Since ẏ 2 = 2gy, we obtain R = λyg + 2λyg = 3λyg.


When the string has just fallen by y = l, R = 3λlg = 3M g.

Method 4: Consider the center of mass of the string at time t.

When the string falls by a distance y. The center of mass of the string measured from
the reference point is
 
l−y
lλy + y + λ (l − y)
2
yCM =
λl
1
ly + (l + y) (l − y)
= 2
l
1 2
l − y2

ly +
= 2
l

l2 − y 2
⇒ yCM = y +
2l

Differentiate both sides with respect to time, we obtain


1
ẏCM = ẏ − (y ẏ)
l
1 2 
ÿCM = ÿ − ẏ + y ÿ
l
Since ÿ = g and ẏ 2 = 2gy. we have
1
ÿCM = g − (2gy + yg)
l
1
ÿCM = g − (3gy)
l
Hence, we have
Chapter 7 Systems of Particles 127

g
ÿCM = (l − 3y)
l
According to Newton’s second law of motion, Fnet = ma, we can write

λlg − R = λl ÿCM
⇒ R = λlg − λl ÿCM
hg i
⇒ R = λlg − λl (l − 3y)
l
⇒ R = 3λgy

At the instant that the whole string is on the table, i.e. y = l, the reaction force
R = 3λgl = 3M g.

Example
A pile of string having a uniform mass density λ is staying on the top of table. One end of
it is pulled by a lifting force F such that it rises vertically upward with a constant speed
v0 . Find the required force F when the string is at a height y above the table.

Solution
d~p
Method 1: Consider the net force acting on the whole string, e.g. F~net = .
dt

When the string rises, the two portions of the string perform differently. The upper
portion moves with constant speed and it has increasing momentum as the mass of it
increases, but the lower portion has zero momentum as it is always at rest.

We first define our system having the entire string enclosed in the green box. Notice that
the interacting forces between the two portions (i.e. at the junction of them) are internal
forces instead of external forces exerted on the system. The net force acting on the whole
string is F − λyg, because the weight of the lower portion is balanced by the upward
Chapter 7 Systems of Particles 128

reaction force by the table. Therefore,


d~p
Fnet = F − λyg =
dt
d
⇒ F − λyg = (λy ẏ)
dt
⇒ F − λyg = λy ÿ + λẏ 2

Since ẏ = v0 = constant, we have ÿ = 0. Therefore,

F = λyg + λv02
⇒ F = λ(yg + v02 )

Method 2: By using the rocket equation

Consider the upper portion of the string (i.e. the portion enclosed by the green box) as
the interested object in the rocket equation.
d~v dm
F~ext = m − ~vrel
dt dt
We note that v = v0 = constant, m = λy and



 Fext = F − mg = F − λyg






vrel = 0 − v0 = −v0






 dv dv0

 = =0
dt dt








 dm = d(λy) = λẏ = λv0



dt dt
Therefore, we have F − λyg = m(0) − (−v0 )λv0 , which implies

F = λ (yg + v02 )
Chapter 7 Systems of Particles 129

Example
In a rainy day, an empty box of mass m0 and initial speed v0 is moving along a straight,
horizontal and smooth track in an outdoor area. If the box has no cover and raindrops are
collected at a constant mass rate α during the trip, please work out the following parts.

(a) By considering the methods as shown below, show that the velocity of the box at
time t is given by
m0 v0
v=
m0 + α t
(i) The conservation of linear momentum.
(ii) The rocket equation.

(b) Hence, find the displacement of the box at time t.

Solution
(a)(i) Due to the conservation of linear momentum along the horizontal, we have

m0 v0 + 0 = (m0 + α t) v

So
m0 v0
v=
m0 + α t
(ii) According to the rocket equation, we have
d~v dm
F~ext = m − ~vrel , where ~vrel = ~vraindrop − ~vbox .
dt dt
Let’s apply the rocket equation to the moving box along the horizontal at time t:
dv
0 = (m0 + α t) − α (0 − v)
dt
dv
α v = −(m0 + α t)
dt
Separating the variables and integrating both sides of the equation, we have
Z t Z v
dt 1 dv
= −
0 m0 + α t α v0 v
t v
1 1
ln(m0 + α t) = − ln v
α α
 0  v0 
1 m0 + α t 1 v
ln = − ln
α m0 α v0
m0 + α t v0
=
m0 v
m0 v0
v =
m0 + α t
Chapter 7 Systems of Particles 130

ds
(b) Let s be the displacement of the box after time t. Recall that v = , so
dt
ds m0 v0
=
dt m0 + α t
Z s Z t
dt
ds = m0 v0
0 m0 + α t
Z0 t
m0 v0 d(m0 + α t)
s =
α 0 m0 + α t
t
m0 v0
= ln(m0 + α t)
α 0
 
m0 v0 m0 + α t
= ln
α m0
 
m0 v0 αt
Finally, we get s = ln 1 + .
α m0
Chapter 8

Rotational Kinematics
z

y
r
P

x
To describe the rotation of a rigid body about a fixed axis. We can observe the motion
of a fixed point P in the rigid body. The motion of P is a circular motion about the axis
of rotation.
y

P
r s
φ
arc length s = rφ
x
angular s
φ= [unit: radian]
displacement r

131
Chapter 8 Rotational Kinematics 132

Surprisingly, angular displacement is a scalar instead of a vector because it does not obey
the commutative law. The figure below states that the order of the rotations will make
different to the result.

Like what is done in linear motion, average angular


velocity ωav and instantaneous angular velocity ω can
P at t 2 be defined as:
φ2 − φ1 ∆φ
ωav = =
P at t 1 t2 − t1 ∆t
φ2 [unit: rad s−1 ]
∆φ dφ
φ1 ω = lim
∆t→0 ∆t
=
dt
x

Similarly, average and angular instantaneous acceleration αav and α are defined by:

ω(t2 ) − ω(t1 ) ∆ω
αav = =
t2 − t1 ∆t
[unit: rad s−2 ]
∆ω dω
α = lim =
∆t→0 ∆t dt
Chapter 8 Rotational Kinematics 133

Angular velocity as a vector

z z

w
y y
P

P w
x x

Right Hand Screw Rule

8.1 Rotation with constant angular acceleration



Suppose α = = constant, we obtain dω = αdt. Integrating on both sides, we have
dt

ω = αt + A, where A = constant.

At t = 0, ω = ω0 = A where ω0 is the initial angular velocity.

∴ ω = ω0 + αt

Therefore,
dφ 1
= ω0 + αt ⇒ φ = ω0 t + αt2 + B
ω=
dt 2
At t = 0, φ = φ0 = B where φ0 is the initial angular displacement.

∴ φ = φ0 + ω0 t + 21 αt2
Chapter 8 Rotational Kinematics 134

8.2 Relation between linear and angular variables


y

• In a time interval ∆t, the rotating vector


~r moves through an angle ∆φ.
r ∆s
• If ∆t → 0, |∆~s| = ∆s = r∆φ.
∆φ
r
x

Thus the tangential velocity is


∆s dφ
vT = lim =r
∆t→0 ∆t dt
∴ vT = ωr

Moreover, tangential acceleration is given by:


dvT dω
aT = =r
dt dt
∴ aT = αr

From previous chapters, we also know for particle undergoing circular motion with con-
stant speed, the particle indeed accelerates toward the center (centripetal acceleration).
Thus the radial acceleration is equal to:

vT2
aR = = ω2r
r

Hence, the resultant acceleration:


~a = ~aT + ~aR
y

aT
a

r
aR
x
Chapter 8 Rotational Kinematics 135

N.B. For uniform circular motion, α = 0 ⇒ ~a = ~aR , pure radial acceleration!!

Example
Wheel A of radius rA = 10 cm is coupled by belt B to wheel C of radius rC = 25 cm. The
angular speed of wheel A is increased from rest at a constant rate of 1.6 rad/s2 . Find the
time for wheel C to reach a rotational speed of 100 rev/min, assuming the belt does not
slip.

Solution
As the belt has no slipping, then vA = vC . It implies that

aA = aC
rA αA = rC αC
rA αA
αC =
rC

(10 cm) (1.6 rad/s2 )


Hence, αC = = 0.64 rad/s2 .
25 cm
(100 rev/min) (2π rad/rev)
Note that ωC = 100 rev/min = = 10.47 rad/sec.
60 sec/min

Since ωC = 0 + αC t, we have
ωC 10.47 rad/s
t= = = 16.36 sec.
αC 0.64 rad/s2
Example
One can check the relations stated below with right-hand rule.
A particle moves on a plane about a fixed point O with constant angular velocity and
~ × ~r, where ω
constant radius. The tangential velocity is given by ~v = ω ~ and ~r are
the angular velocity and the position vector of the particle respectively. The centripetal
~ × (~ω × ~r).
acceleration of the particle is ω
Chapter 9

Rotational Dynamics

9.1 Torque
Definition

F A force F~ was acting on a body at a point P . The


torque about the point O is defined as
P ~τ = ~r × F~
r
~ .
where ~r = OP
x

y
F

θ
|~τ | = |~r||F~ | sin θ
P
FT pointing out of paper
r

136
Chapter 9 Rotational Dynamics 137

P θ τ = rF sin θ

pointing into the paper


r F

FT

Example

Point
O

τ
θ ~τ = ~r × F~
L ∴ τ = Lmg sin θ

r with direction pointing inward

mg

Point
O

τ
θ
τ = Lmg sin θ

r with direction pointing outward

mg
Chapter 9 Rotational Dynamics 138

9.2 Rotational inertia


Single particle

• A particle of mass m is connected to the z-axis by a massless


y
rod of length r.
F
From previous results (in the last chapter),
θ
aT = αr Fsinθ
m
∴ FT = maT = mαz r
r
But FT = F sin θ x
z
∴ F sin θ = mαz r

As τz = F r sin θ,
τz = mαz r2

N.B. Subscript z is added to specify the axis of rotation.

Define I = mr2 for single particle so that

τz = Iαz

This is referred to as Newton’s 2nd law for rotation.


y
More than one particle P

• Two masses m1 and m2 lie on the xy-plane. They are linked m1 T1r
by massless rods to the z-axis. m1 and m2 are linked to each T1 T2r
l
other by a similar rod. r1 m2
T2
• Rotation axis: z-axis. r2
x
• P~ is an external force. z
Chapter 9 Rotational Dynamics 139

P~
Total force on m1 : F1 = P~ + T~1 + T~1r
P~
Total force on m2 : F2 = T~2 + T~2r

y Total torque on the system about z-axis:


X X
τz = (~r1 × F~1 ) + (~r2 × F~2 )
θ1
= (~r1 × P~ ) + (~r1 × T~1 ) +(~r1 × T~1r )
T1r | {z }
→0
T2r + (~r2 × T~2 ) +(~r2 × T~2r )
r1 h2
h1 | {z }
θ2 →0

(∵ ~r1 // T~1 and ~r2 // T~2 )


r2 = (~r1 × P~ ) + (~r1 × T~1r ) + (~r2 × T~2r )
x
z

Notice that

|~r1 × T~1r | = r1 T1r sin θ1


|~r2 × T~2r | = r2 T2r sin θ2 (9.1)

(~r1 × T~1r ) and (~r2 × T~2r ) are in opposite directions.

Let the distance between m1 and m2 be `.

∴ h1 = ` sin θ1 & h2 = ` sin θ2

But area of triangle Om1 m2 is equal to

1
hr
2 1 1
= 12 h2 r2
⇒ ` sin θ1 r1 = ` sin θ2 r2
⇒ r1 sin θ1 = r2 sin θ2 (9.2)

Since action-reaction forces are equal in magnitude

T1r = T2r (9.3)

Substitute eq. (9.2) and (9.3) into eq. (9.1), we obtain

|~r1 × T~1r | = |~r2 × T~2r |


Chapter 9 Rotational Dynamics 140

Therefore, the total torque is


τz = ~r1 × P~

which is only dependent on external force. Torque created by internal force are cancelled
out.
X
Or τz = τext,i .
i

P~
Consider the total force Fi on mass mi with
i = 1 or 2. y
radial
direction
Σ F1T
P~
• Fi can be decomposed into two com-
ponents, namely the tangential and the m1 Σ F2T
radial. r1 radial
direction
X X
• τz = (~r1 × F~1 ) + (~r2 × F~2 ) m2
r2
X X
= r1 ( F1T ) +r2 ( F2T ) x
τz
| {z } | {z }
m1 a1T m2 a2T

∵ Radial components has no contribution to torque, i.e.

τz = r1 (m1 a1T ) + r2 (m2 a2T )


|{z} |{z}
αz r 1 αz r2

= (m1 r12 + m2 r22 ) αz

where aiT are tangential acceleration of mi and αz is the angular acceleration of the
system about the z axis.
Or
τz = Iαz

where I = m1 r12 + m2 r22 .

For a N -particle system


y
Masses m1 , m2 , . . . , mN are located at ~r1 , ~r2 , . . . , ~rN
m1
on the xy-plane.
r1 m2 Moment of inertia is defined by
r2 X
x I= mi ri2
rN mN i
Chapter 9 Rotational Dynamics 141

The total torque acting on this system:


X
τz = ~ri × F~i = Iαz
i

In a rigid body, the number of particles in it becomes infinite. The moment of inertia of
it about an axis is given by
Z
I= r2 dm

where r is the normal distance between the axis of rotation and an arbitrary particle (i.e.
small mass element) in the object. The mass of the particle is dm. One should note that
the particles within a rigid body have fixed relative position with respect to each others,
i.e. as if they are linked by massless rods.
Example

o
m2 30 m1 = 2.3 kg
θ m2 = 3.2 kg
5 m3 = 1.5 kg
3
θ
x
4
m1 m3

(a) Find moment of inertia of the system about axis perpendicular to xy plane and
passing m1 , m2 and m3 respectively.
(b) If a 4.5 N force is applied to m2 as shown and the system is free to rotate about the
axis perpendicular to the xy plane and passing through m3 . What is the angular
acceleration?

Solution

(a) By definition,
X
I1 = mi ri2 = (2.3 kg) (0 m)2 + (3.2 kg) (3 m)2 + (1.5 kg) (4 m)2
i
= 53 kg m2
Chapter 9 Rotational Dynamics 142

Similarly,

I2 = (2.3 kg) (3 m)2 + (3.2 kg) (0 m)2 + (1.5 kg) (5 m)2 = 58 kg m2


I3 = (2.3 kg) (4 m)2 + (3.2 kg) (5 m)2 + (1.5 kg) (0 m)2 = 117 kg m2
 
−1 3
(b) θ = sin ⇒ θ = 37◦
5
∴ τz = 4.5 N × 5 m × sin(30◦ + 37◦ ) = 20.7 N m

But τz = Iαz
τz
∴ αz = = 0.18 rad s−2 in clockwise direction
I3

Example
A uniform rod has mass m and length L. Find the moment of inertia about an axis
normal to it and through its center.

Solution

Partition the whole rod into many infinitesimal segments with length dx and consider one
of the segment at x. The mass of this small segment is

dm = λ dx, where λ is the mass density (i.e. mass per unit length)

The moment of inertia of the rod about an axis normal to it and through its center is
Z L Z L Z L
2 2 2
2 2
I= x dm = λ x dx = 2 λ x2 dx
−L
2
−L
2
0

M
But λ = , so
L L
2M L3
   
2λ 3
2 1
I= x =
= M L2
3 0 3L 8 12
Chapter 9 Rotational Dynamics 143

Example
An uniform circular ring of mass M and radius R rotates
about its axis. Find the moment of inertia of this ring.

z
Solution
Consider the small arc (i.e. the red one in the figure) having
an subtended angle dθ.
R

dm = λ (R dθ) λ = linear density (kg m)−1

Hence, its moment of inertia about the ring’s center is given


by y

Z Z 2π
dm
I = R2 dm = R2 (λR dθ)
0 R
θ
x
Z 2π
3
= λR dθ
0
 
M
= R3 (2π)
2πR
⇒ I = M R2

Example
An uniform circular disk of radius R and mass M is rotating about the disk center. Find
the moment of inertia of the disk about its axis.

Solution
Consider a disk as a collection of many rings.
Chapter 9 Rotational Dynamics 144

Think about a ring with radius r, thickness dr and surface density σ. The mass of this
small ring

dm = σ 2πr dr

Moment of inertia of the ring about an axis through its center is given by

dI = r2 dm = r2 (σ 2πr dr) = 2σ πr3 dr

Therefore, the total moment of inertia of the disk about its center is
Z Z R Z R    4
3 3 M R 1
I = dI = 2σ πr dr = 2σ π r dr = 2 2
π = M R2
0 0 πR 4 2

Remark:
The mass of the ring is given by

dm = σ [π(r + dr)2 − πr2 ]


= σ [π (dr)2 +2πr dr]
| {z }
higher order term → 0
= σ 2πr dr

9.3 Parallel axis theorem


C.M.
axis
z
Iz = ICM + M h2
h
Iz = Moment of inertia rotating about z-axis,
ICM = Moment of inertia rotating about the axis
C.M. passing through C.M.,
z-axis is parallel to the C.M. axis and h is the
distance between the two parallel axes.
M
Chapter 9 Rotational Dynamics 145

Proof:

z’
z
slab // to
h z & z’ axis
mass mi & For the Iz about the z-axis:
coordinate
(xi , yi )
X X
C.M. Iz = mi ri2 = mi (x2i + yi2 )
i i

Let (xCM , yCM ) be the x, y coordinates of the


C.M. measured from the x, y coordinate sys-
y’ tem.
y (xi , yi ) (
xi = x0i + xCM
(xCM, yCM)
ri yi = yi0 + yCM
z h x’
x

X
∴ Iz = mi [(x0i + xCM )2 + (yi0 + yCM )2 ]
i
X 2 2
= mi (x0i + 2x0i xCM + x2CM + yi0 + 2yi0 yCM + yCM
2
)
i
X 2 2
X X X
= mi (x0i + yi0 ) +2xCM mi x0i +2yCM mi yi0 + (x2CM + yCM
2
) mi
| {z }
|i {z } | i {z } | i {z } =h2 | i {z }
=ICM =M x0CM =0 0
=M yCM =0 =M
2
⇒ Iz = ICM + M h

Remark:
The following proof shows the vanishing of second and third terms in the R.H.S. of the
long equation.
Chapter 9 Rotational Dynamics 146


 ~ri = ~r 0i + ~rCM
P
m ~r
 ~rCM = P i i
mi
P P
The second equation becomes ~rCM mi = mi~ri which implies
X X
~rCM mi = mi (~r 0i + ~rCM )
X
⇒ mi~r 0i = 0

mi~v 0i = 0.
P
A further result is

N.B. The axis passing through the CM is the axis that has the smallest moment of
inertia as compared to other parallel axis.

9.4 Rotational inertia of solid bodies


X
Discrete form: I= mi ri2
i
Z
Integral form: I= r2 dm

Example
A uniform rod has mass m and length L. Find the moment of inertia about an axis
normal to it and passing through its end. Use direct integration and parallel axis theorem
to obtain the answer.

Solution
Direct Integration:

Partition the whole rod into many infinitesimal segments with length dx and consider one
of the segment at x. The mass of this segment is

dm = λ dx, where λ is the density (i.e. kg m−1 )


Chapter 9 Rotational Dynamics 147

Z L Z L
2
∴ I= x dm = λ x2 dx
0 0
M
But λ = , so
L   L  
λ M 1
I= x3 = L3 = M L2
3 0 3L 3

Parallel axis theorem:

Iz = ICM + M h2
 2
1 2 L
= ML + M
12 2
1
= M L2
3

L/2 L/2

C. M.

Example
A uniform rectangular plate of mass M and dimensions a × b is rotating about an axis
through the center of plate and is normal to the plate. Find the moment of inertia of this
plate about the axis.

z
z’

11111111
00000000
x
00000000
11111111dx
00000000
11111111
00000000
11111111
b 11111111
00000000
00000000
11111111
00000000
11111111
a
00000000
11111111
Chapter 9 Rotational Dynamics 148

Solution
Partition the plate into strips each having the width dx.
The mass of each strip is

dm = σ (adx), where σ is the density of plate

The moment of inertia of the strip about the z 0 -axis is


1 1
dICM = (dm) a2 = σa3 dx
12 12
The moment of inertia of the strip about z-axis is
1
dI = dICM + x2 dm = σa3 dx + aσx2 dx,
12
where σ = M/(ab).  
1 2 2
∴ dI = aσ a + x dx
12
∴ The total moment of inertia about z-axis is
Z b/2 Z b/2   2  b/2
x3

1 2 2 ax
Iz = dI = aσ a + x dx = aσ +
−b/2 −b/2 12 12 3 −b/2
    
1 2 1 3 M 1 2 1 3
= aσ a b+ b =a a b+ b
12 12 ab 12 12
M 2
= (a + b2 )
12
Example
Find the moment of inertia of a square of mass M and side L about its center without
using direct integration.

Solution
The moment of inertia of a square of mass M and side L about its center O is given by
IM, O = k M L2 , where k is a constant. Divide the square into four identical squares, each
M L
of mass and side as shown in figure. Then, consider the small square in the top left
4 2
corner of the figure.
Chapter 9 Rotational Dynamics 149

The moment of inertia of this small square about its center O0 is


   2
M L k
I M , O0 = k = M L2 .
4 4 2 16
Using the parallel axis theorem, the moment √ of inertia of this small square about O is
M 2L
I M , O = I M , O0 + (OO0 )2 , where OO0 = . Hence,
4 4 4 4
√ !2
k M 2L
IM ,O = M L2 +
4 16 4 4
k 1
= M L2 + M L2
16 32
But,

IM, O = 4 I M , O0
4 
2 k 2 1 2
k ML = 4 ML + ML
16 32
1
k =
6
1
Therefore, the required moment of inertia of the square is IM, O = M L2 .
6

Example
Find the moment of inertia of a uniform sphere of mass M and radius R about its axis.

Solution
Consider a sphere as a collection of many disks.
Chapter 9 Rotational Dynamics 150

Think about a thin disk of thickness dz and radius r in the figure, where r = R cos θ. The
mass of it is
dm = ρ (πr2 ) dz = ρ (πR2 cos2 θ) dz.
But, z = R sin θ, we have dz = R cos θ dθ and

dm = ρπR3 cos3 θ dθ.


R R r2
The moment of inertia of the sphere I = dI = dm. More specifically,
2
Z π/2 2 Z π/2
r
I = dm = r2 dm
−π/2 2 0
Z π/2
= (R cos θ)2 ρπR3 cos3 θ dθ
0
Z π/2
5
= R πρ cos5 θ dθ
0
Z π/2
= R5 πρ (1 − sin2 θ)2 d sin θ
0
Z π/2
= R5 πρ (1 − 2 sin2 θ + sin4 θ) d sin θ
0
 π/2
2 sin3 θ sin5 θ

5
= R πρ sin θ − +
3 5
0
 
8
= R5 πρ
15
Thus, we obtain
 
 M  8
I = R5 π 
4 
15
πR3
3
2
⇒ I = M R2
5
Chapter 9 Rotational Dynamics 151

Example
This question computes the moment of inertia of a hollow sphere if the result of a solid
sphere is known, and vice versa.

(a) Apply the result in the last example to determine the moment of inertia of a hollow
sphere of radius r and mass m.

(b) Hence, find the moment of inertia of a uniform sphere of mass M and radius R again.

Solution
(a) Consider a uniform sphere of mass M , density ρ, and radius r. The moment of inertia
2
of it about its diameter is I = M r2 . Rewrite it as a function r, then
5
 
2 4
I = 3
π ρ r r2
5 3
8
I = π ρ r5
15
The total differential of I is
8
dI = π ρ r4 dr
3
which is the moment of inertia of a spherical hollow of infinitesimal thickness dr and mass
dM = 4 π ρ r2 dr. Then
2
4 π ρ r2 dr r2

dI =
3
2
= (dM ) r2
3
2
Hence, we have IHollow = mr2 .
3

(b) A sphere is a collection of numerous hollows, Z


each of thickness dr and mass dM . So,
the moment of inertia of a uniform sphere is I = dI, where

2 2 8
(dM ) r2 = 4 πρ r2 r2 dr = πρ r4 dr

dI = IHollow =
3 3 3
So
Z Z R  
8 4 8 8 M
I= dI = πρ r dr = πρ R5 = π 4 R5
3 0 15 15 3
πR3
Then, the moment of inertia of a uniform sphere is given by
2
M R2
5
Chapter 9 Rotational Dynamics 152

9.5 Perpendicular axis theorem


For a laminar, if z- axis is normal to the plane of laminar, and x and y axes lie on the
laminar, then the moment of inertia about these axes are related by

Iz = Ix + Iy ,

Proof:
Z
Iz = r2 dm
Z
⇒ Iz = (x2 + y 2 ) dm
Z Z
⇒ Iz = x dm + y 2 dm
2

⇒ Iz = Iy + Ix

Example
Find the moment of inertia of a disk of mass m and radius r about its diameter.

Solution
The moment of inertia of a disk about the z-axis is Iz = mr2 /2, where the axis is normal
to the plane of disk and passes through the center. Suppose that the x and y axes lie on
the disk and they cut at the disk center, Iz = Ix + Iy = 2ID , where Ix = Iy = ID and ID
is the required moment of inertia. Therefore, ID = mr2 /4.

Example
A uniform laminar of mass m has an area bounded by y = x2 and y = 2.

(a) Find the moment of inertia of the laminar about the y-axis.
Chapter 9 Rotational Dynamics 153

(b) Find the moment of inertia of the laminar about the x-axis, hence, find the moment
of inertia of the laminar about the z-axis.

Solution

(a) Method 1:
Consider the vertical strip, as shown in the figure. The moment of inertia of this
strip about the y-axis is Iy = dIy , where dIy = [(2 − x2 ) σdx] x2 and σ is the
R
| {z }
dm
surface density.


Z 2
Iy = √
(2 − x2 ) σ x2 dx
− 2
"Z √ √ #
2 Z 2
= σ √
2x2 dx − √
x4 dx
− 2 − 2

4(23/2 ) 2(25/2 )
 
= σ −
3 5
3m 4(2 ) 2(25/2 )
 3/2 
= √ −
8 2 3 5
 
3m 8 8
= −
8 3 5
2m
⇒ Iy =
5
Chapter 9 Rotational Dynamics 154

Method 2:
Consider the horizontal strip, as shown in the figure.

R
The moment of inertia of this strip about the y-axis is Iy = dIy , where
1 √
dIy = dm (2x)2 and dm = σ (2x) dy = σ (2 y) dy.
12
Therefore,
Z Z
1
Iy = dIy = dm (2x)2
12
Z 2
1 √ √
= [σ (2 y) dy] (2 y)2
0 12
Z 2
2
= σ y 3/2 dy
3 0
4
25/2 σ

=
15  
4 5/2
 3m
= 2 √
15 8 2
2m
=
5

(b) Consider the shaded region which has length 2x and small height dy, where x ≥ 0.
When it rotates about the x-axis, it obtains the moment of inertia dIx .
Chapter 9 Rotational Dynamics 155

R √ 
Since Ix = dIx , where dIx = (dm) y 2 = (2x σ dy) y 2 = 2 y σ dy y 2 = 2y 5/2 σ dy,
therefore,
Z Z 2
Ix = dIx = 2σ y 5/2 dy
0
  2
7/2 11/2
y =2
σ
= 2σ
7/2 0 7
211/2
 
3m 12m
= √ =
7 8 2 7
12m 2m 74m
(c) Iz = Ix + Iy = + = .
7 5 35

9.6 Equilibrium of rigid body


For a rigid body to stay at equilibrium, the following conditions must be satisfied.
P~
1) Fext = 0
P
2) ~τext = 0 (about choice of reference point)
P~ P
Question: If we know Fext = 0 and ~τ = 0 about one particular point, say O, can
we conclude the total torque about any other choice of point?

Fi
ri −rP
P
ri
rP x
O

Rigid body
y

Refer to point O,

~τo = ~τ1 + ~τ2 + . . . ~τN


= (~r1 × F~1 ) + (~r2 × F~2 ) + . . . + (~rN × F~N )
= 0
Chapter 9 Rotational Dynamics 156

Now, we consider the torque about another point P ,

~τP = (~r1 − ~rP ) × F~1 + (~r2 − ~rP ) × F~2 + . . . + (~rN − ~rP ) × F~N
= [(~r1 × F~1 ) + (~r2 × F~2 ) + . . . + (~rN × F~N )] − [~rP × (F~1 + F~2 + . . . + F~N )]
| {z } | {z }
Given condition = 0
P ~
i Fi =0

= 0
P ~ P
If i Fi = 0 (i.e. translational equilibrium established) and ~τext = 0 about a given
P
point, then ~τext = 0 about any point and thus, equilibrium must be established.
Example

L As the bar is in equilibrium,


L X
4 F~i = 0
M i

m
~1 + R
∴ R ~ 2 + M~g + m~g = O
~

y ~1 + R
or R ~ 2 + (M + m)~g = O
~
R1
R2
rR2
rR1 Take upward as positive:
z
x R1 + R2 − (M + m)g = 0 (9.4)
rM
Take moment about O,
mg
Mg
~τ = ~rR1 × R~1 + ~rM × (M~g ) + ~rR2 × R~2
L L L
⇒ τ = − R1 + M g + R2 = 0
2 4 2
Therefore,
1
R1 − R2 − M g = 0 (9.5)
2
Summing up (9.4) and (9.5) gives
1 3
2R1 = (M + m)g + M g = mg + M g
2 2
1 3
R1 = mg + M g
2 4
1 1 1
R2 = R1 − M g = mg + M g
2 2 4
Chapter 9 Rotational Dynamics 157

Example
A non-uniform rod of mass m is at equilibrium at its position as shown in the figure. It
has a block of mass M fixed at its middle. As the rod is non-uniform, the center of mass
of the rod does not locate at its center, but instead it is displaced a distance from the
center point. Find the normal forces acting on it by the frictionless wall and the rough
floor.

Solution

frictionless wall

a
2
a
3
M
h
C.M.
rough
ladder mass = m
O
a

R2

R1

f O

Mg
mg

P ~
To obtain equilibrium, i Fi = 0.
(
R2 = f

R1 = (m + M )g
Chapter 9 Rotational Dynamics 158

Consider the torque about the axis through O and perpendicular to the paper.
a a
Mg + mg = R2 h
2   3
a a
⇒ R2 = f = Mg + mg
2h 3h
Example
A uniform beam of mass m is at equilibrium, as shown in the below figure. Find the
reaction force exerted on the hinge by the wall.

α
θ

y
T
α
θ
Fv
Mg
mg
x
O Fh

Solution
Label the x and y components of the reaction force at the hinge by Fh and Fv respectively.

Fh − T cos α = 0 (9.6)
Fv − mg − M g + T sin α = 0 (9.7)

Consider the torque about the axis passing through O and perpendicular to the paper.
 
L
T L sin(α + θ) − M gL cos θ − mg cos θ = 0
2
g(M + m/2) cos θ
∴ T =
sin(α + θ)
Chapter 9 Rotational Dynamics 159

From (9.6),
g(M + m/2) cos θ cos α
Fh = .
sin(α + θ)
From (9.7),
g(M + m/2) cos θ sin α
Fv = (m + M )g − .
sin(α + θ)

9.7 Non-equilibrium situation: pure rotation


Example
A block of mass m is connected to one end of a light string, while the other end of the
string is winded to a smooth pulley of mass M and radius R. Find the linear acceleration
of the block and the angular acceleration of the pulley when the block is released.

Solution

Frictionless pulley with mass M and radius R.

d2 y
M mg − T = ma, where a = 2
dt
⇒ T = mg − ma
R
T Torque acting on pulley,

d2 θ
T τ = RT = Iα, where α =
y dt2
where I = 12 M R2 for disk rotating about its center.
m
1
∴ R (mg − ma) = M R2 α
2
⇒ 2mg − 2ma = M Rα (9.8)
mg
If the rope runs through the pulley without slipping,

d2 θ d2 y
Rθ = y ⇒ R = i.e. Rα = a
dt2 dt2
Thus, (9.8) becomes
2mg − 2mRα = M Rα
Chapter 9 Rotational Dynamics 160

Hence,
2mg
α=
R(M + 2m)

2mg
a=
M + 2m
Example
Two blocks of mass m and 2m are connected by a light string. They are hanging over
a pulley of mass M and radius R. If the system has no friction, find the linear acceler-
ation of the blocks and the angular acceleration of the pulley when the blocks are released.

Solution
Consider the equations of motion for both
+ve blocks.

T1 − mg = ma (9.9)
2mg − T2 = 2ma (9.10)
T1 R
Total torque on pulley, i.e. τ = Iα
T2
1
RT2 − RT1 = ( M R2 ) α (9.11)
T1 2
The condition of rolling without slipping:
T2
m
a = Rα (9.12)

Put (9.9) and (9.10) into (9.11):


mg 2m
1
(2mg−2ma)R−(ma+mg)R = M R2 α (9.13)
2
Substitute (9.12) into (9.13), we obtain
2mg 1
2mg − 2m(Rα) − m(Rα) − mg = M Rα
2
1
⇒ mg = M Rα + 3mRα
2
mg mg
⇒ α= and a = Rα =
M R/2 + 3mR M/2 + 3m
Chapter 9 Rotational Dynamics 161

9.8 Non-equilibrium situation: rotational and trans-


lational motion
P ~ P
If i Fi 6= 0 and τi 6= 0 about any axis, the motion of the rigid body has both self-
i~
rotation and the motion of the C.M.
For the present course, we only focus on cases such that:
a) Axis of rotation passes through C.M.
b) Rotating axis always has the same direction in space.

Example
A solid cylinder of radius R and mass M is released from rest on an inclined surface such
that it rolls down without slipping on the surface. Find the linear and angular accelera-
tion of the cylinder.

mass = M N The equations of motion of the cylinder that


is normal and along the motion.
α
R N = M g cos θ
f
M g sin θ − f = M aC.M. (9.14)
Mgsinθ
aC.M. Total torque on the cylinder:
1
Mgcosθ τ = f R = Iα = M R2 α
θ 1
2
⇒ f = M Rα (9.15)
2
Put (9.15) into (9.14):
1
M g sin θ − M Rα = M aC.M.
2
But the condition of rolling without slipping is Rα = aC.M. , thus
1
M g sin θ − M aC.M. = M aC.M.
2
2 aC.M 2g
⇒ aC.M. = g sin θ and α = = sin θ
3 R 3R
Chapter 9 Rotational Dynamics 162

Example

A uniform solid cylinder of radius R and mass M is given


an initial velocity ω0 and then lowered on a uniform hori-
zontal surface. The coefficient of kinetic friction between
the cylinder and the surface is µk . Initially the cylinder
ωo slips as it moves along the surface, but after a time t,
pure rolling without slipping begins.

(a) What is the velocity vCM at time t?


(b) What is the value of t?

Solution

(a) During the time interval 0 to t:

N
α ωo

aCM

Mg

Consider the convention of motions (both linear and rotational) of the cylinder. The
positive directions are shown in green color in the figure. The frictional force f is a
non-zero constant when the cylinder is slipping. We know also that at t = 0, the C.M.
of cylinder is at rest and the frictional force starts to increase the linear speed of it
towards right while the rotational speed of it decreases gradually.

f = M aCM , where f = µk N = µk M g

⇒ aCM = µk g (9.16)
For the rotation of the cylinder,

Rf = Iα
M R2
 
⇒ R(µk M g) = α
2
Chapter 9 Rotational Dynamics 163

which gives
2µk g
α= (9.17)
R
Note that when the cylinder starts to move a 6= Rα as the cylinder is slipping and it
is referred to as the first stage of motion. The occurrence of pure rolling (i.e. without
slipping) will starts at time t when vf = Rωf .

vf − 0
a = (9.18)
t
−ωf − (−ω0 )
α = (9.19)
t
Eqs (9.16) and (9.18) give
vf
µk g =
t
Rωf
When rolling without slipping occurs, we have vf = Rωf . Thus, µk g = , which
t
gives
Rωf
t= (9.20)
µk g

From Eq. (9.19), we have

αt = −ωf + ω0 (9.21)

Substitute Eq. (9.17) and Eq. (9.20) into the above equation, we obtain
 
2µk g Rωf
= −ωk + ω0
R µk g
ω0
⇒ ωf =
3
Rω0
The linear speed of cylinder at time t is vf = .
3
Rω0
(b) From Eq. (9.20), the time t is .
3µk g
Chapter 9 Rotational Dynamics 164

Example
Two solid cylinders are sticked together and string
is winded on the cylinder with the smaller radius.
Assume that the small cylinder is very light and
it is negligible as compared to the large cylinder T
α
which has mass M . Find the linear and angu-
lar accelerations of the system if one end of the
string is fixed but the cylinders are allowed to fall
R
freely.
Ro a

Solution mass M
Consider the linear motion of the cylinders, the equation
of motion of this is given by Mg

Mg − T = Ma (9.22)

Consider the rotational motion of the cylinders, the equation of motion of this is given by

τ = R0 T = R0 M (g − a) (9.23)
1
τ = Iα = M R2 α (9.24)
2
(9.23) and (9.24) gives
1
R0 M (g − a) = M R2 α
2
R0
⇒ α = 2 (g − a) 2 (9.25)
R
For no slipping,
a = R0 α (9.26)

Substituting (9.26) into (9.25), we obtain


a R0
= 2 (g − a) 2
R0 R
2
2gR0
⇒ a= 2
R + 2R02
Hence,
a 2 gR0
α= = 2
R0 R + 2R02
Chapter 9 Rotational Dynamics 165

Example
A uniform thin rod of length 2L and mass M lies horizontally on two smooth supports,
as shown in the figure. The two ends of rod are labeled as A snd B.

If support at A is suddenly removed, at that instant what are

(a) the torque about B,

(b) the vertical reaction force at B.

Solution

(a) When support A is just removed, the rod is exerted by two forces, its weight and
the reaction force R at B by the right support. The torque acting on the rod about
B is τ = M gL.

(b) The moment of inertia of the rod about B is given by


1 4
IB = M (2L)2 + M (L2 ) = M L2 .
12 3
The angular acceleration of rod about B, that is α, is obtained by using Newton’s
second law for rotation.

M gL = IB α
4
⇒ M gL = M L2 α
3
3g
⇒ α =
4L
Chapter 9 Rotational Dynamics 166

Thus, at the instant, the linear acceleration of the center of rod is


 
3g 3
a = Lα = L = g.
4L 4

Consider the equation of motion of rod just after support A is removed.

Mg − R = Ma
 
3
⇒ Mg − R = M g
4
1
⇒ R = Mg
4

Example
A uniform thin rod of length 2L and mass M has one end hinged to the floor. Initially, it
is held vertical above the floor and it is disturbed to fall down. The rod makes an angle
θ with the vertical at time t. The moment of inertia of the rod about one end of rod is
4
M L2 .
3
(a) Find θ̈ as a function of θ.

(b) Find θ̇ as a function of θ.

Solution

(a)

The hinge exerts a force on the rod. The components of the force along and normal
to the horizontal are S and R respectively. The torque about the lower end of rod
is M g (L sin θ). Hence, we have
4
M g (L sin θ) = M L2 θ̈
3
3g
θ̈ = sin θ
4L
Chapter 9 Rotational Dynamics 167

! !
dθ̇ dθ̇ dθ dθ̇
(b) Since θ̈ = = = θ̇, we can write
dt dθ dt dθ
!
dθ̇ 3g
θ̇ = sin θ
dθ 4L
Z θ̇ Z θ
3g
⇒ θ̇ dθ̇ = sin θ dθ
0 4L 0
θ̇ θ
θ̇ 2 3g
⇒ = − cos θ
2 4L 0
0
2
θ̇ 3g
⇒ = − (cos θ − 1)
2 r4L
3g
⇒ θ̇ = (1 − cos θ)
2L

Example
A uniform thin rod of mass m and length l is resting with its lower end on a frictionless
floor, where the rod makes an angle θ0 to the vertical. Then, it is released to fall down.
Find the normal force exerted on the rod by the floor at the instant of release.

Solution

The equations of motion for translational and rotational motions.



 R − mg = mÿ
 
l 1
 R sin θ = ml2 θ̈ (Moment about C)
2 12
l
Note that the time derviatives of y = cos θ are
2
l l h i
ẏ = − θ̇ sin θ and ÿ = − θ̈ sin θ + θ̇ 2 cos θ
2 2
Chapter 9 Rotational Dynamics 168

Set the symbols for the motion of rod at t = 0 : θ = θ0 , θ̈ = θ̈0 , R = R0 , ÿ = ÿ0 .


The equations of motion at t = 0 become

 R0 − mg = mÿ0
(9.27)
 R0 sin θ0 = 1 mlθ̈0
6
l
Also, when the rod is just released, we have θ̇ = 0, thus ÿ0 = − θ̈0 sin θ0 .
2
From the first equation of (9.27) and the above relation.

ml
R0 − mg = − θ̈0 sin θ0 (9.28)
2
From the second equation of (9.27)

6R0 sin θ0
θ̈0 = (9.29)
ml
Substitute Eq. (9.29) into Eq. (9.28)
 
ml 6R0 sin θ0
R0 − mg = − sin θ0 (9.30)
2 ml
mg
⇒ R0 = (9.31)
1 + 3 sin2 θ0
Example
A solid sphere of mass m and radius r is at rest on a horizontal platform. The platform is
given a constant acceleration a such that the sphere rolls without sliding on the platform.

(a) Find the linear acceleration of the sphere.

(b) Find also the angular acceleration of the sphere.

Solution

(a)
There are some points to notice about the motion of the sphere.
Chapter 9 Rotational Dynamics 169

(i) When the platform starts to move, a frictional force exerts on the sphere and
it points to the right.
(ii) The sphere rolls backward relative to the platform (counterclockwise as shown
in the figure) because the frictional force produces a torque about the center
of sphere.
(iii) The center of sphere C accelerates towards the right with aC , where aC is
relative to a fixed observer. It is also true that if this observer is an inertial
one.

f = maC (9.32)

(iv) Note also that aC < a, where a is the acceleration of the platform relative to
a fixed observer because the center of sphere is moving back relative to the
platform. Moreover, the contact point P on the sphere is accelerating to the
right, relative to C.
(v) As the sphere rolls without sliding on the platform, the contact point P of
the sphere is not moving relative to the platform. That is to say, point P is
moving with the same speed with the platform. Since both the sphere and the
platform start from rest and accelerate, point P has the same acceleration as
the platform. Hence, we can write a = aC + rα. Therefore,

a − aC = rα
τ 
⇒ a − aC = r ,
I
2
where τ = rf and I = mr2 . Thus,
5
 
rf
a − aC = r 2
5
mr2
5f
⇒ a − aC = ,
2m
5(maC )
From Eq. (9.32), we have a − aC = ,
2m
5
⇒ a − aC = aC
2
2
⇒ aC = a
7
2 5
Remark: The sphere rolls backward with aC − a = a − a = − a relative to the
7 7
platform, i.e. to the left of the platform.
Chapter 9 Rotational Dynamics 170

2
(b) Refer to Eq.(9.32), the frictional force f = maC = ma, thus
7
r 27 ma

τ rf 5a
α= = = 2 2
=
I I 5
mr 7r

Example
A circular disk rotating about its axis with angular speed ω0 is placed gently with its axis
horizontal on a rough and inclined plane such that the friction acts up the plane. Given
that the coefficient of friction is µ and the inclination angle of the plane to the horizontal
is φ.

(a) Show that the disk will move upwards if µ > tan φ.

(b) Find the time that elapses before rolling takes place.

Solution

(a) The translational motion of the disk:

mg sin φ − f = mẍ (9.33)

The rotational motion of the disk:


ma2
fa = θ̈ (9.34)
2
ma2
where τ = f a is the torque exerted on the disk and is the moment of inertia of the
2
disk.
The friction exerted on the disk:

f = µR = µmg cos φ (9.35)


Chapter 9 Rotational Dynamics 171

Using equations (9.33) and (9.35), we have

mg sin φ − µmg cos φ = mẍ (9.36)

If the disk moves upward, ẍ < 0, then

sin φ − µ cos φ < 0


sin φ < µ cos φ
tan φ < µ

(b) From equation (9.36), we have

g sin φ − µg cos φ = ẍ

Integrating both sides of the above equation, we get

(g sin φ − µg cos φ) t + C = ẋ

When t = 0, ẋ = 0, so C = 0. Hence,

ẋ = (sin φ − µ cos φ) gt (9.37)

From equations (9.34) and (9.35), we have


ma2
(µmg cos φ) a = θ̈
2
2 µg
cos φ = θ̈
a
Integrating both sides of the above equation, we get
2 µgt
cos φ + C 0 = θ̇
a
When t = 0, θ̇ = −ω0 , so C 0 = −ω0 . Hence,
2 µgt
θ̇ = cos φ − ω0 (9.38)
a
The disk rolls without sliding, so ẋ = a θ̇ . Equations (9.37) and (9.38) give
 
2 µgt
(sin φ − µ cos φ) gt = a cos φ − ω0
a
(sin φ − 3 µ cos φ) gt = −a ω0
a ω0
t =
g (3 µ cos φ − sin φ)
Chapter 9 Rotational Dynamics 172

Example
A solid sphere of radius a on a rough plane is struck by a horizontal force F . The point
of impact is at a vertical distance d = 6a/7 above the center of the sphere. Pure rolling
of translational speed v 0 occurs after sliding a distance. Express v 0 in terms of v0 , where
v0 is the translational speed obtained by the sphere just after the impact.
[Hint: At the instant of strike, the friction on the sphere due to the rough plane is negli-
gible when it is compared to F .]

Solution
Suppose that the force F acts on the sphere by ∆t seconds such that the linear velocity
and the angular velocity of the sphere become v0 and ω0 respectively. The friction due to
the rough plane is ignorable when the force F strikes on the sphere.

Consider the impulse exerted on the sphere.

F ∆t = m v0 (9.39)
6a
Consider the moment of impulse exerted on the sphere. The moment arm is d = .
7
 
6a 2
F ∆t = ma2 ω0 (9.40)
7 5

Eliminating F and ∆t from equations (9.39) and (9.40), we get


6 2
a mv0 = ma2 ω0
7 5
7
v0 = a ω0 (9.41)
15
Chapter 9 Rotational Dynamics 173

7
Since v0 = a ω0 < a ω0 , the friction points left as shown in the figure below. This
15
frictional force is not ignorable because the force F is no longer acting on the sphere.

f = m ẍ (9.42)
2
f a = − ma2 θ̈ (9.43)
5
Using equations (9.42) and (9.43), we have
2
m ẍ a = − ma2 θ̈
5
2
ẍ = − a θ̈
5
2
ẋ = − a θ̇ + C
5

When t = 0, ẋ = v0 and θ̇ = ω0 . Hence,


2
v0 = − a ω0 + C
5
 
2 2 15 13
which gives C = v0 + a ω0 = v0 + a v0 = v0 .
5 5 7a 7
2 13
Hence, ẋ = − a θ̇ + v0 .
5 7

Pure rolling occurs when v 0 = ẋ = a θ̇. So


2 13
ẋ = − ẋ + v0
5 7
7 13
ẋ = v0
5 7
65
ẋ = v0
49
65
Hence, v 0 = v0
49
Chapter 10

Angular Momentum

10.1 Definition
Angular momentum of a point mass m about a point O is given by:
z
~` = m~r × ~v = ~r × p~, |~`| = |~r||~p| sin θ

d~` d d~r d~p


= (~r × p~) = × p~ + ~r ×
dt dt dt dt
l d~p
= ~v × (m~v ) + ~r ×
dt
O y d~` d~p
∴ = ~r × = ~r × F~ = ~τ
r v dt dt
θ where F~ is the resultant force acting on mass m.
x Note that both ~` and ~τ in the above equation must
be defined with respect to the same origin.
Example
y

b P A particle of mass m is released from rest at point


x
O P.
θ Torque on m with respect to O:

r τ = mgr sin θ = mgb (inward)


m
000
111
111
000 Moreover,
000
111
dy
θ ` = rm sin θ (inward)
dt
F=mg = mvy b

174
Chapter 10 Angular Momentum 175

d`
As τ = ,
dt
d dvy
∴ mgb = (mvy b) = mb
dt dt
d2 y
⇒ g= 2
dt

For the case of a system of particles, m1 , m2 , . . . , mN having angular momenta of ~`1 , ~`2 , . . . , ~`N .
Total angular momentum of the system:
X
L~ = ~`i (by definition)
i
~
dL X d~`i X
∴ = = ~τi (~τi = ~τi,int + ~τi,ext )
dt i
dt i

~
State without proof: Internal torques NOT contribute to change of L.

~
dL X
= ~τi,ext
dt i

The following figure shows the analogies between linear and angular momentum.

Linear momentum Angular momentum


F// p ∆ p// τ// L ∆L//

∆ p//
F// = τ// = ∆L//
∆t ∆t

F p +∆ p τ L +∆L
∆p ∆L
p L

∆p
F = τ = ∆L
∆t ∆t
Chapter 10 Angular Momentum 176

An interesting observation

~τ = ~r × (m~g ) (pointing inward)


∆L~
But, ~τ =
∆t
~ k ~τ and it will also point inward!
∴ ∆L

~ is along the spinning axis, the axis will


As L
move inward.

10.2 Angular momentum and angular velocity

ω A particle m is in circular motion.


Now taking the reference point as O.
θ ~` = ~r × p~ is not parallel to ω
~.
r’ 00
11
00
11 Indeed, not like the case in the simplest
00 p
11
case (i.e. the origin O is also the rotat-
ing center), angular momentum ~` and
θ r
angular velocity ω
~ are in general not
parallel.
y Under what other conditions will the ~`
O
and ω
~ vectors parallel?

x
Chapter 10 Angular Momentum 177

Consider two equal masses m1 and m2


z jointed by a light rod is rotating about the
center of rod. In this case, ~` // ω
~.
2
= 1+ 2 Generally, ~` is parallel to ω
~ when the rigid
p 11
2 00
00
11 body is symmetric about the rotational
11 ω
00
axis.
1
For a more general case, where m1 = 6 m2 ,
r’ 00
11
00
11 ~` 6= I~ω (as we have the relation p~ = m~v
00 p1
11
r2 for linear motion). What is the relation
r1 between ~` and ω
~?
From previous figure of the single particle
case,
y
O
`z = ` sin θ = rp sin θ
= r(mv) sin θ
x = r(mr0 ω) sin θ
(∵ v = r0 ω)

But since r0 = r sin θ,


2
∴ `z = mr0 ω = Iω

Notes:

• The z-component of angular momentum is equal to Iω. Though this result is only
obtained from single particle, it is also true for rigid body.

~ = I~ω . For such case, L


• If the rigid body is symmetric about the rotation axis, L ~
and ω
~ are both parallel to the rotating z-axis.

• In general, Lz = Iω.
If ~τ = τx î + τy ĵ + τz k̂,

dLz
τz =
dt

That is to say, if I is constant and τz = 0, ω will not be changed!


Chapter 10 Angular Momentum 178

Example

Total angular momentum of the system:

~ z = ~`M + ~`m
L
M
R ∴ Lz = IM ω + m v R
x O
|{z}
+ve mag.
|{z}
+ve mag.
(take pointing outward to be +ve)
1
= M R2 ω + mvR
2
y
Total external torque:

rm , m = mvR ~τz = ~rm × m~g ⇒ τz = mgR

m dLz
∵ τz =
dt
 
p= mv ∴ mgR =
d 1 2
M R ω + mvR
dt 2
mg 1
= M R2 α + mRa
2

where α is the angular acceleration of the pulley and a is the acceleration of the mass.
But α = a/R.
1 a
mgR = M R2 + mRa
2 R
2mg
⇒ a=
M + 2m

10.3 Conservation of angular momentum


As we know
~
dL
~τ =
dt
~
dL
if the total external torque acting on the system is zero, then dt
= 0. This implies angular
momentum is conserved.
Chapter 10 Angular Momentum 179

Example

1)
Ii If

Ii > If

Ii ωi = If ωf

∴ Angular velocity increases.

wi wf

2)
Lw −Lw
Ls
~ ω : initial wheel angular mo-
L
mentum,
~ s : final student and turn-table
L
angular momentum.
stationary
turn table

~i = L
L ~ω ~f = L
L ~ s + (−L
~ ω)
~s − L
= L ~ω
~i
= L
~ s = 2L
∴ L ~ω

10.4 Stability of spinning object

• ~τ // is applied for a time period of


Lf
∆t.
∆L = τ // ∆t
Li ∆L
τ//
• Spinning speed is changed but
Object is symmetric about the rotating axis not the spinning orientation.
Chapter 10 Angular Momentum 180

τ Lf
• ~τ⊥ is applied for a time period of
∆t.
θ ∆L
∆L = τ⊥ ∆t
Li
• Spinning orientation is changed.
Object is symmetric about the rotating axis
∆L τ⊥ ∆t
tan θ = =
Li Li

If Li is large, θ is small for fixed τ⊥ ∆t. That is to say, a large spinning speed or a large I
have a higher stability in spinning orientation.

Example
A running bicycle does not collapse despite of it’s body makes an tilting angle with the
floor. The spinning wheels keep the stability.
Chapter 10 Angular Momentum 181

The spinning top

z z

dL
dφ τ = r Mg
Lsinθ

L L + dL
L
r CM
θ
Mg
y y
O O

x x

~ is parallel to the rotation axis (i.e. ω


Consider a symmetric spinning top, i. e. L ~ ).
External torque on the top: |~τ | = M gr sin θ
~ = τ ∆t = M gr∆t sin θ.
Over a time interval ∆t, |∆L|
Moreover,

∆L = (L sin θ)∆φ
M gr∆t sin θ M gr∆t
⇒ ∆φ = =
L sin θ L
~  |L|
As a fact that |dL| ~ and ~τ ⊥ L
~ implies dL~ ⊥ L.
~ The external torque only changes
~ but not the magnitude of L.
the direction of L ~

~ (or the rotation axis) moves slowly about the vertical axis (called
Hence, the vector L
precession).
Average speed of precession:
∆φ M gr
ωp = =
∆t L
Chapter 10 Angular Momentum 182

Example
A rod of length 2l and mass m has a fine hole at its end. Initially, the rod moves with speed
v on a smooth horizontal table, where v is normal to the rod. A pin is then embedded in
the hole and the rod rotates about it.

(a) Find the angular speed of the rod about the pin.

(b) Find the impulse acting on the rod when the pin is introduced.

(c) Find the force acting on the pin when the rod is rotating.

Solution

(a)

Moment of inertia about O just before and after the pin is inserted.
1 4
IO = m(2l)2 + ml2 = ml2
12 3
Angular momentum about O: mvl = I0 ω gives
 
4 2
mvl = ml ω
3
3v
⇒ ω =
4l
(b) Let the required impulse be J, where

J = change of momentum of C.M. = mωl − mv


 
3v mv
Therefore, we obtain J = m l − mv = − . The negative sign means that
4l 4
the impulse is opposite to the direction of v.

(c) The required force T = force on the pin, pointing outward and along the rod.
2
9mv 2

2 3v
T = mω l = m l=
4l 16l
Chapter 10 Angular Momentum 183

Example
Two particles of mass m1 and m2 are connected by an inextensible and massless string
of length l. The system is placed on a smooth and horizontal plane with the string being
kept tight. Mass m1 is projected along a direction which is normal to the string and the
speed is v0 .

(a) Find the speed of the center of mass of the system and the angular speed of the
system about its center of mass.

(b) Hence, find the linear speeds of the individual masses just after the attack.

Solution

(a)

As there is no external force acting on the system, the speed of the CM does not
change. The total linear momentum of the system does not change.

(m1 + m2 ) vC = m1 v0
m1 v0
⇒ vC =
m1 + m2
Let C be the location of C.M. of the system. To locate it, we can write
(
r1 + r2 = l
r1 m1 = r2 m2 ,
where r1 and r2 are the distances of m1 and m2 from point C respectively. Hence,
we have

m2 l

 r1 =
m1 + m2


m1 l



 r2 =

m1 + m2
Chapter 10 Angular Momentum 184

At the moment of projection, the angular momentum of the system about C is


 
m1 m2
LC = r1 m1 v0 = lv0 = µlv0 ,
m1 + m2
1 1 1 m1 m2
where = + or µ = .
µ m1 m2 m1 + m2
Just after the projection, the angular momentum of the system about C is

LC = m1 r12 ω + m2 r22 ω
 2  2
m2 l m1 l
= m1 ω + m2 ω
m1 + m2 m1 + m2
 
m1 m2
= l2 ω
m1 + m2
⇒ LC = µl2 ω

By the conservation of angular momentum:

µlv0 = µl2 ω
v0
ω =
l

(b) The speed of m1 : v1 = vC + r1 ω, which gives


m1 v0 m2 l v 
0
v1 = + = v0
m1 + m2 m1 + m2 l
The speed of m2 : v2 = vC − r2 ω, which gives
m1 v0 m1 l v 
0
v2 = − =0
m1 + m2 m1 + m2 l

Example
A thin rod AB of mass M and length l is placed on a smooth and horizontal table. A
particle of mass m travels at speed v0 and its direction is normal to the rod. A perfectly
inelastic collision occurs when it hits the end of the rod at A. Find the angular speed
of the rod-particle system after the collision. Consider the angular momentum about the
following points to obtain the answer.
Chapter 10 Angular Momentum 185

(a) The center of mass of the system at C.

(b) The center of the rod at O.

(c) The end point of the rod at A.

(d) The end point of the rod B.

Solution
(a) Using the definition of the center of mass, we have

m l
lM = ·


m+M 2


M l


·

 lm =

m+M 2

The moment of inertia of the system about C is


M (4m + M ) l2
 
1 2 2 2
IC = M l + M lM + m lm =
12 12 (m + M )
Denote the linear velocity of the center of mass of the system as VC . The system rotates
about C with angular velocity ω after the collision.
Chapter 10 Angular Momentum 186

Conservation of angular momentum of the sytstem about C gives

lm m v0 = IC ω
M (4m + M ) l2
 
M l
· m v0 = ω
m+M 2 12 (m + M )
6 m v0
ω =
l (4m + M )

Remark:
Conservation of linear momentum of the system gives

m v0 + M (0) = (m + M ) VC
m v0
VC =
m+M
In fact, the velocity of the center of mass of the system never changes because there is no
net force exerted on the system.

(b) Conservation of angular momentum of the system about O gives


 
l
m v0 = (m + M ) VC lM + IC ω
2
M (4m + M ) l2
    
m v0 m l
= (M + m) · + ω
m+M m+M 2 12 (m + M )
 2
M (4m + M ) l2
   
m v0 l
= + ω
m+M 2 12 (m + M )

So
M (4m + M ) l2
   
l m M v0
= ω
2 m+M 12 (m + M )
6 m v0
ω =
l (4m + M )

(c) Conservation of angular momentum of the system about A gives

0 = −lm (m + M ) VC + IC ω
M (4m + M ) l2
     
M l m v0
= − (m + M ) · + ω
m+M 2 m+M 12 (m + M )
M (4m + M ) l2
 
m M l v0
= − + ω
2 (m + M ) 12 (m + M )
6 m v0
ω =
l (4m + M )
Chapter 10 Angular Momentum 187

(d) Conservation of angular momentum of the system about B gives


 
l
l m v0 = + lM (m + M ) VC + IC ω
2
M (4m + M ) l2
     
l m l m v0
= + · (m + M ) + ω
2 m+M 2 m+M 12 (m + M )
M (4m + M ) l2
 
(2 m + M ) m v0 l
= + ω
2 (m + M ) 12 (m + M )
6 m v0
ω =
l (4m + M )
Chapter 11

Work, Kinetic Energy and Potential


Energy

11.1 Work done by a constant force


y Consider a point mass m in a time interval of ∆t is
experiencing a constant force F~ . During this time
∆S ~
interval, the displacement of m is ∆S.

111
000
000
111 θ Work done by the force on the mass:
000
111
000
111 F
def
W = F~ · ∆S
~ = F ∆S cos θ .
x

Work can be either positive or neg-


ative.
Power is defined by:

dW
P =
dt

188
Chapter 11 Work, and Kinetic Energy and Potential Energy 189

11.2 Work done by a variable force

11.2.1 One dimensional case


Suppose there is a position dependent force F (x).

F
∆x
F(xi )
F(x)

x
x0 xN
xi
xi+1

positive negative
work done work done

• Divide the whole displacement from x0 to xN into N partitions with separation ∆x.

• Consider the i-th partition, xi → xi+1 and in this very small interval, F is approxi-
mately constant at F (xi ).

∴ Work done in this time interval:


∆W (xi ) = F (xi ) ∆x

dW
or F (x) =
dx

Total work done for the displacement from x0 to xN :


X X Z xN
Wx0 →xN = ∆Wi = F (xi )∆x = F (x) dx
i i x0

or it is equal to the total area of the figure with positive area for positive F (x) and
negative area for negative F (x).
Chapter 11 Work, and Kinetic Energy and Potential Energy 190

Example

x
The restoring force is F = −kx and the work
m F = −kx done by the system when the mass moves
from A to B:
Z xB
WA→B = F (x) dx
equilibrium position
x=0 ZxAxB
= −kx dx
F(x) xA
1
= − k(x2B − x2A )
2

F = −kx
Example

A mass is attached to the lower end of a light spring which is fixed to the ceiling. The
green dotted line shows the unstretched position of the light spring when there is no mass
under it. The motion of mass is governed by two forces, i.e. gravity and the restoring
force of spring. Therefore, the net force acting on the mass when the spring extends from
its natural length is F (y) = −mg − ky, where y is the coordinate of the mass. The work
done by the system when the mass moves from A to B:
Z yB
WA→B = F dy
yA
Z yB
= (−mg − ky) dy
yA
1
= −mg(yB − yA ) − k(yB2 − yA2 )
2
Chapter 11 Work, and Kinetic Energy and Potential Energy 191

11.2.2 Two dimensional case

y
Trajectory of a particle is given by:
F(r(t))
r(t) ∆r r(t) ~r(t) = fx (t)î + fy (t)ĵ

r(t + ∆ t) Force at any point ~r is given by:

F~ (t) = Fx (~r)î + Fy (~r)ĵ


x

Consider a particle moving from ~r(t) to ~r(t + ∆t) during the time interval ∆t.
If ∆t → 0, force experienced by particle in this time interval is constant and ≈ F~ (~r(t)).
Work done in this small time interval with displacement ∆~r:

∆W = F~ (~r(t)) · ∆~r
(Only the tangential force component contributes!!)
Z
∴ W = F~ · d~r

Example
A mass m is hanged by a string with length L initially. A force F which is always
horizontal is applied to lift the mass up to an angle φ. During the process, the mass
moves with constant speed so small that the centripetal force can be neglected. Find the
work done by the force F .
y

φm T
φ φ
L
F F

x
mg
Chapter 11 Work, and Kinetic Energy and Potential Energy 192

Solution
If centripetal force approaches zero, ax = 0 and ay = 0.

y F − T sin φ = 0 and T cos φ − mg = 0


⇒ F = mg tan φ

Consider the displacement ∆S from φ → φ + ∆φ:


∆φ = F~ · ∆S
~ = F~tang · ∆S
~
φ ∆W
∆S but also: = F ∆x

But
x = L sin φ ⇒ dx = L cos φ dφ
x
x ∴ ∆W = mg tan φ L cos φ dφ = mgL sin φ dφ
x+∆ x Hence,
Z φm
W = mgL sin φ dφ = mgL (1 − cos φm )
0

11.3 Work-energy theorem


vi vf Consider a particle m displaces from xi to xf .
111
000 F 111
000
000 111
111
111
000
000
000 x 111
111 000 111
000 During this displacement, the x-component of
000 000
111 111
000
111 000
111 x
O x xf the net force acting on m is Fx .
i
dv dv dx dv
∴ Fx = m =m = mv (11.1)
dt dx dt dx
Work done on the mass by the force:
Z xf
W = Fx dx
Zxixf
dv
= mv dx (using eq. (11.1))
dx
Zxivf
= mv dv
vi
1
∴ W = m(vf2 − vi2 )
2
1
Define kinetic energy K = mv 2 . We obtain W = Kf − Ki = ∆K.
2
If W is positive, vf > vi and ∆K > 0.
If W is negative, vf < vi and ∆K < 0.
Chapter 11 Work, and Kinetic Energy and Potential Energy 193

N.B. In inertia frames having relative motion, the absolute value of kinetic energy are
not the same, but the theorem W = ∆K holds in all inertia frames.

11.4 Work done and kinetic energy in rotational mo-


tion
y
Consider a rigid body moving through an angle dθ
about the rotational z-axis with a force acting on
point P .
F
Work done by the force:
φ
dS dW = (F sin φ) dS = F sin φ rdθ = τz dθ

P where τz is the z-component of the torque about O.
r
x

∴ If the rigid body is to displace from θi to θf ,


Z θf
W = τz dθ
θi

If the torque is constant,


W = τz θ
dW dθ
Power: P = = τz = τz ω.
dt dt

Consider each point of the rigid body, say


w
m1 , m2 , . . . , mN .
Kinetic energy of particle i is given by:
1 1 1
Ki = mi vi2 = mi (ri ω)2 = mi ri2 ω 2
v2 2 2 2
m2 ∴ Total rotational kinetic energy of the rigid body:
r2 X X1
K = Ki = mi ri2 ω 2
r1 2
v1 i i
m1
!
1 X
= mi ri2 ω 2
2 i

1
∴ K = Iω 2
2
I is the moment of inertia of the rigid body about the rotational axis.
Chapter 11 Work, and Kinetic Energy and Potential Energy 194

11.5 A combination of rotational and translational


motions
In previous sections, we have considered cases of pure translational (i.e. movement of
C.M. of rigid body) or pure rotational (about a fixed axis) motion. Now we turn into case
such that both the CM is moving and the rigid body is rotating.

y Consider a rigid body consisted of particles


m1 , m2 , . . . , mN .
mn Total K.E. of the body:
rn’ 1X
rn K= mi vi2 (11.2)
CM 2 i
Note that
rCM
~ri = ~rCM + ~ri0 ⇒ ~vi = ~vCM + ~vi0
x
where ~vi = velocity of mass i with respect to the Earth’s frame,
~vCM = velocity of the body’s center of mass with respect to the Earth’s frame,
~vi0 = velocity of mass i with respect to the body’s center of mass.

From (11.2), we obtain


1X 1X 2
K= mi (~vCM + ~vi0 ) · (~vCM + ~vi0 ) = 2
mi (vCM + 2~vCM · ~vi0 + vi0 )
2 i 2 i
But consider the second term:
X X X
~vCM · (mi~vi0 ) = ~vCM · (mi~vi − mi~vCM ) = ~vCM · 2
mi~vi − M vCM
i i i
P
As ~vCM = ( i mi~vi )/M ,
X
∴ ~vCM · (mi~vi0 ) = ~vCM · M~vCM − M vCM
2
=0
i
And the third term:
1X 2 1X 1
mi vi0 = mi (ri0 ω)2 = Iω 2
2 i 2 i 2
where ω is the angular velocity about an axis passing through the center of mass.
1 2 1
∴ K = M vCM + Iω 2
2 2
1st term: Translational term of the C.M. as if there is no rotation.
2nd term: Rotational term with rotation about the axis passing through the C.M. as if
the rotational axis does not move.
Chapter 11 Work, and Kinetic Energy and Potential Energy 195

11.6 Kinetic energy in collision


For elastic collision, ∆K = 0, i. e. Kf = Ki .
For inelastic collision, ∆K < 0, i. e. Kf < Ki .
For complete inelastic collision, the two colliding objects stick together after collision.

Example
A moving particle has speed u and mass m2 . It collides on a stationary particle which
has mass m1 . If the collision is elastic, find the final speeds of the particles.

m2 m1
u

Solution
By the conservation laws, we have

m1 v1 + m2 v2 = m2 u (11.3)
1
m v2
2 1 1
+ 21 m2 v22 = 21 m2 u2 (11.4)

From (11.3),
m2 u − m2 v2
v1 = (11.5)
m1
Substitute (11.5) into (11.4), we get:
 2
m2 u − m2 v2
m1 + m2 v22 = m2 u2
m1
⇒ m22 u2 + m22 v22 − 2m22 uv2 + m1 m2 v22 = m1 m2 u2

⇒ (m1 + m2 ) v22 − 2m2 uv2 + (m2 − m1 ) u2 = 0

Solve for v2 and then v1 , we find


   
2m2 m2 − m1
v1 = u and v2 = u.
m1 + m2 m1 + m2

11.7 Conservative force


Potential energy is only defined for conservative force in which a particle moving under
the force influence has constant mechanical energy.

Examples of conservative force:


Chapter 11 Work, and Kinetic Energy and Potential Energy 196

1) Spring
2) Gravitational force
3) Coulomb force

Example of non-conservative force - friction.

Definition

A conservative force is a force such that if a particle moves under the influence of this
force, the work done by the force on moving the particle from an arbitrary point A to
another arbitrary point B would be the same along any arbitrarily chosen path, e.g. path
1 and path 2, as shown in the figure.
Z Z
F~ · d~r = F~ · d~r
P ath 1 P ath 2

y
1
B

A
2

In other words, work done of a conservative force to move I an object along a closed path
(i.e. starting and ending at the same point) is zero, i.e. F~ · d~r = 0.
Remark:
There are some other rigorous definitions using the mathematical language, but they are
out of the scopes of this course, e.g.

1) F~ (~r) is conserved if and only if there exists a scalar function φ(~r) such that

∇φ(~r) = F~ (~r),

where φ is referred to as the potential function of the force, and ∇ is a gradient


operator.
2) F~ (~r) is conserved if and only if ∇ × F~ = 0.
Chapter 11 Work, and Kinetic Energy and Potential Energy 197

11.8 Potential energy


Consider a particle moves in the influence of a conservative force, which is position de-
pendent, i. e. F (x). Now the particle displaces from xi to xf , potential difference ∆U is
defined:
∆U = Uf − Ui = −W

where W is the work done by the force during the displacement xi to xf .


Z xf
Or ∆U = U (xf ) − U (xi ) = − F (x) dx
xi

def
If for a particular reference point x0 , the potential energy is defined as zero, i.e. U (x0 ) = 0.
Z x
U (x) = − F (x) dx
x0

The inverse of the above equation gives

dU
F (x) = −
dx
Examples
(a) The spring-mass system

F = −kx F = −kx

x Take the equilibrium position to be x = 0 so that


U (0) = 0.
x
m
Z
∴ U (x) − U (0) = − F (x) dx
0
Z x
⇒ U (x) = − (−kx) dx
0
equilibrium position 1
⇒ U (x) = kx2
x = 0, U = 0 2
Thus
dU 1
= k(2x) = kx = −F
dx 2
Chapter 11 Work, and Kinetic Energy and Potential Energy 198

(2) An object under the force of gravity

y Take U (0) = 0.
Z y
∴ U (y) − U (0) = − F (y) dy
0
Z y
y ⇒ U (y) = − (−mg) dy
0
F = −mg ⇒ U (y) = mgy

y = 0, U = 0 Thus
dU
= mg = −F
dy

11.9 Conservation of mechanical energy


∆U = Uf − Ui = −W (11.6)

Ui vi Uf vf

initial final
position position

R xf
But W = xi
F (x) dx is the work done by the force in the journey from xi → xf .
From previous chapter,
Z xf
1
W = F (x)dx = m(vf2 − vi2 ) = Kf − Ki = ∆K (11.7)
xi 2

Substitute (11.7) into (11.6), we have

Uf − Ui = Ki − Kf
⇒ Ui + Ki = Uf + Kf
⇒ ∆U = −∆K

In an isolating system whose only conservative force exists, mechanical energy of a particle
conserves.
Chapter 11 Work, and Kinetic Energy and Potential Energy 199

11.10 One dimensional conservative system

U(x)

E4
E3 K(xf )
K(xg )

E2
E1 U(xf )

E0 U(xg )

x
xa xb xc xd xe xf xg

• Particle experienced a conservative force field with potential energy U (x).

dU
• F (x) = −
dx
∴ At x = xa , xd , xf , xg , F = 0.

x = xd , xg : stable equilibrium - slightly displaced particle experiences a restor-


ing force

x = xf : unstable equilibrium - displaced particle experiences a force in the


same direction as displacement

x = xa : neutral equilibrium - displaced particle experiences no force

1
• U (x) + mv 2 = E, where E is the conserved total energy.
2
Example

If E = E4 as shown in the previous figure,

E4 = K(xg ) + U (xg ) at x = xg
E4 = K(xf ) + U (xf ) at x = xf

If the energy of the particle E is different, it will have different behavior as follows:

1) If E = E0 , particle stays stationary at x = xd .


Chapter 11 Work, and Kinetic Energy and Potential Energy 200

2) If E = E1 , particle stays in the region xc ≤ x ≤ xe .


3) If E = E2 , particle may stay in the two valleys. However if it is in one of the
valley.
4) If E = E3 , particle can stay in the region x > xb .
5) If E ≥ E4 , particle can be anywhere.

• If U (x) is known, it is possible to work out the particle’s position.


Chapter 12

Conservation of Energy

If external force acting on the system is not zero, the conservation of energy becomes

∆K + ∆U = Wext ,

where Wext is the work done on the system by the external force.

Example

Wspring Uspring Wspring


+K
K
K + Ugrav K + Ugrav
Wgrav Wgrav +Uspring

Earth Earth Earth Earth

System = Mass System = Mass + System = Mass + System = Mass +


∆K = Spring Earth Earth + Spring
Wspring + Wgrav ∆K + ∆Uspring = ∆K + ∆Ugrav = ∆K + ∆Ugrav +
Wgrav Wspring ∆Uspring = 0

201
Chapter 12 Conservation of Energy 202

Example
A thin rod of mass m and length l is held vertical on the top of a horizontal floor, where
the rod’s lower end is hinged to a fixed joint on the floor. The rod is released and it hits
the floor when it’s angular speed is ω. Obtain ω by the following methods.

(a) The Newton’s second law of motion for rotation.

(b) The work-energy theorem for a torque.

(c) The conservation of mechanical energy.

Solution

(a) The moment of inertia of the rod about its end is given by I, where
1 l 1
I= ml2 + m( )2 = ml2
12 2 3

Newton’s second law of motion for rotation states that τ = I θ̈. Hence, we have
l 1 2
mg( ) sin θ = ml θ̈
2 3
3g
⇒ sin θ = θ̈
2l
3g dθ̇
⇒ sin θ = θ̇ ( )
2l dθ
Z π/2 Z ω
3g
⇒ sin θ dθ = θ̇ dθ̇
2l 0 0
π/2
3g ω2
⇒ (− cos θ) =
2l 0 2
3g ω2
⇒ =
2l r2
3g
⇒ ω =
l
Chapter 12 Conservation of Energy 203

(b) Work done by the torque equals to the change of kinetic energy.
Z π/2
τ dθ = ∆K.E.
0
Z π/2
l 1 2
⇒ mg ( ) sin θ dθ = Iω − 0
0 2 2
Z π/2
l 1 1 2 2
⇒ mg ( ) sin θ dθ = ( ml ) ω
2 0 2 3
π/2
3g
⇒ (− cos θ) = ω2
l 0
r
3g
⇒ ω =
l
(c) Conservation of mechanical energy:

∆P.E. + ∆K.E. = 0
l 1 1 2 2
⇒ {0 − mg ( )} + { ( ml ) ω − 0} = 0
2 2 3
1 2
⇒ g = lω
3
r
3g
⇒ ω =
l

12.1 Internal energy in a system of particle


∆K + ∆U + ∆Eint = Wext
where Eint is the change in internal energy of the system.
Internal energy is the K.E. associated with the random motion of atoms and molecules
(usually related to the object temperature), or the P.E. associated with forces between
atoms.
Eint = Kint + Uint
Looking deeper into the cases of rigid body
Fext
Fext = M aCM for rigid body

Notice that Fext may not act on the C.M.


dxCM
Consider the force acting for a short period and in the period,
CM CM
C.M. displaces by dxCM . Multiply both sides by dxCM , we
have
Fext dxCM = M aCM dxCM
Chapter 12 Conservation of Energy 204

It is just a product of a force and the displacement dxCM . There is NO physical meaning
such as work done! Notice that the definition of work done F~ · d~x refers to an applied
force where it applies to a point mass and the mass is displaced by d~x.
dvCM
Fext dxCM = M aCM dxCM = M vCM dt
dt
⇒ Fext dxCM = M vCM dvCM

Consider the C.M. displaces from xi to xf and its velocity change from vCM,i to vCM,f .
Z xf
1 2 1 2
∴ Fext dxCM = M vCM,f − M vCM,i = KCM,f − KCM,i
xi 2 2
def
(∵ KCM = 21 M vCM
2
)

or Fext sCM = ∆KCM if Fext is constant.

This is the Center of Mass (COM) energy equation, where sCM is the displacement of the
center of mass. The COM equation is not the work-energy theorem for a particle. sCM is
the center of mass displacement but not the displacement of the point that the force acts
on.

∆K + ∆U + Eint = Wext

This is the Conservation of Energy (COE) equation.

12.2 Some examples of conservation of energy


1) A sliding block is stopped on a horizontal table with friction.
Center of mass (COM) energy equation: −f sCM = − 21 M vCM
2

Conservation of energy (COE) equation: Wf = − 12 M vCM


2
+ ∆Eint,block

2) Pushing a stick on a horizontal frictionless table.

S Fext

SCM
CM CM
Chapter 12 Conservation of Energy 205

Center of mass (COM) energy equation:


1 2
Fext sCM = M vCM
2
Conservation of energy (COE) equation:
1 2 1
Fext s = M vCM + Iω 2
2 2
If Fext is acted on center of mass,

s = sCM
1 2
Fext s = Fext sCM = M vCM
2

3) Ball rolling down an inclined plane without slipping

SCM
f

Mg
θ

Center of mass (COM) energy equation:


1 2
(M g sin θ − f ) sCM = M vCM
2
Conservation of energy (COE) equation:
1 2 1
M g sCM sin θ = M vCM + Iω 2
| {z } 2 2
M~g acts on CM

Notice that the frictional force does no work in the COE eq. as the instantaneous point
of contact between the ball and the plane does not move.
Chapter 12 Conservation of Energy 206

Example
Two men are pushing each other as shown in the figure. The man m2 is pushed away
from the man m1 by straightening their arms and the force between them is F .

(a) What is the speed of m2 just after losing contact?


(b) What is the change in internal energies for m1 and m2 ?

m2 is pushed
to move forward
m1 m2

frictionless floor

Solution

(a) Consider m2 as the interested system, COM equation of it is


1 2
F sCM = ∆KCM = m2 vCM,m
2 2

where sCM is the displacement of the center of mass of m2 .


r
2F sCM
∴ vCM,m2 =
m2

(b) For m2 , the COE equation is

∆K + ∆Eint,m2 = Wext
(
∆K = ∆KCM = |F sCM |
where .
Wext = |F s|
Note that s is the total extension of m1 ’s hand (i.e. the displacement of m2 ’s hand
when a force F is acting on it, where s 6= sCM ).

∴ ∆Eint,m2 = |F s| − |F scm |

For m1 , COE equation is

∆Eint,m1 = Wext = −|F s| (F~ opposite to ~s)


Chapter 12 Conservation of Energy 207

Example
A uniform sphere of radius r and mass m rolls without sliding on the inner surface of a
fixed and large spherical hollow of radius R, where R > r. The line joining OC makes
an acute angle θ with the lower vertical line through O, where O and C are the center of
the hollow and the sphere respectively. The moment of inertia of the sphere is (2/5)mr2 .
Show that the equation of motion of the sphere is given by
5g
θ̈ + sin θ = 0.
7(R − r)

Obtain your answer by the following methods.

(a) Conservation of mechanical energy.

(b) Newton’s second law of motion.


s
7(R − r)
Show further that the sphere oscillates with a period 2π when θ is small.
5g

Solution

(a)

Set point O be the reference zero of the gravitational potential.

P.E. = −mg(R − r) cos θ


1 2 1
K.E. = I φ̇ + m(R − r)2 θ̇2
2 2
Chapter 12 Conservation of Energy 208

where I is the moment of inertia of the sphere. By the conservation of mechanical


energy, we know that K.E. + P.E. = E, where E is a constant. Therefore,
1 2 1
I φ̇ + m(R − r)2 θ̇2 − mg(R − r) cos θ = E
2 2
Since the sphere rolls without slipping, we have arc BP equals arc QP , i.e. Rθ =
r(φ + θ). Differentiate on both sides with time, we obtain Rθ̇ = r(φ̇ + θ̇). Rearrange
it, we notice an useful relation
 
R−r
φ̇ = θ̇
r
Therefore,
  2
1 2 2 R−r 1
mr θ̇2 + m(R − r)2 θ̇2 − mg(R − r) cos θ = E
2 5 r 2
Simplify it, we have
7
(R − r)2 θ̇2 − g(R − r) cos θ = E
10
Differentiate on both sides with time
 
7
2 (R − r)2 θ̇ θ̈ + g(R − r) θ̇ sin θ = 0
10
7
⇒ (R − r) θ̈ + g sin θ = 0
5
Hence, we obtain
5g
θ̈ + sin θ = 0
7(R − r)

(b)
Chapter 12 Conservation of Energy 209

By the Newton’s law of motion


(
f r = −I φ̈ (rotational motion)
f − mg sin θ = m(R − r) θ̈ (linear motion)

A negative sign appears in the first equation, as the direction of torque is opposite
to the measurement of φ. Eliminate f , we have

I φ̈
− − mg sin θ = m(R − r) θ̈
r
2 2
Substitute the moment of inertia of the sphere, i.e. I = mr , and the relation
5
(R − r) θ̈ = r φ̈ into the above expression, we have
2
mr2 R − r
 
5
− θ̈ − m(R − r) θ̈ − mg sin θ = 0
r r
7
⇒ (R − r) θ̈ + g sin θ = 0
5
5g
⇒ θ̈ + sin θ = 0
7(R − r)

When θ is small, sin θ → θ. Therefore, the equation of motion of sphere becomes


5g
θ̈ + θ=0
7(R − r)
s
5g
This is in the form of S.H.M., e.g. θ̈ + ω 2 θ = 0, where ω = . As the
7(R − r)
s
7(R − r)
period T = 2π/ω, we have T = 2π .
5g
Remark:
If the sphere rolls without slipping, arc BP equals arc QP .

Rθ = r (θ + φ)
⇒ (R − r) θ = rφ
⇒ (R − r) θ̈ = rφ̈

Example
A wedge of mass M has one surface in the shape of a quarter of a cylinder with radius R.
A particle of mass m is initially at the highest point of the curved surface. Assume that
all surfaces are smooth. The system is released.
Chapter 12 Conservation of Energy 210

(a) When the particle is just about to leave the wedge, find the speed of the particle and
the wedge with respect to the ground.

(b) Find the work done on the wedge by the particle.

(c) Find the work done on the particle by the wedge.

Solution
(a) Let v and V be the velocity of the particle and the wedge respectively. Both values
are measured with respect to the ground. The conservation of momentum and energy
give the following equations.

mv − M V = 0 (12.1)
1 1
m v 2 + M V 2 = mgR (12.2)
2 2
Equation (12.1) gives
MV
v= (12.3)
m
Substituting this result into equation (12.2), we have
 2
1 MV 1
m + M V 2 = mgR
2 m 2
 
1 M
M + 1 V 2 = mgR
2 m
2 m2 g R
V2 =
M (m + M )
s
2gR
V = m
M (m + M )
Using equation (12.3) and the last result, we have
s !
M 2gR
v = m
m M (m + M )
r
2 gRM
=
m+M
Chapter 12 Conservation of Energy 211

(b) Denote the work done on the wedge by the particle as Wwp .

∆KEw = Wwp
1
M V 2 − 0 = Wwp
2
2 m2 g R
 
1
M − 0 = Wwp
2 M (m + M )
m2 gR
Wwp =
m+M
(c) Denote the work done on the particle by the wedge as Wpw and the work done on the
particle by the gravity as Wpw .

∆KEp = Wpg + Wpw


1
m v 2 − 0 = mgR + Wpw
2 
1 2gRM
m = mgR + Wpw
2 m+M
gRM m
Wpw = − mgR
m+M  
M
Wpw = mgR −1
m+M
m2 gR
Wpw = −
m+M
Hence, we have Wpw = −Wwp .

Example
A uniform rod of mass M and length l is placed on a smooth and horizontal plane. A
particle of mass m and speed v0 hits on one end of the rod elastically, where the speed
of the particle is normal to the rod. Discuss the subsequent motions of the rod and the
particle.
Chapter 12 Conservation of Energy 212

(a) Find the speeds of them just after the impulse.

(b) Find the conditions that

(a) the particle moves forward after the impulse.


(b) the particle is stopped after the impulse.
(c) the particle returns after the impulse.

Solution
(a) Denote the velocity of the rod and the particle as vM and v respectively after the
collision. Let the angular velocity of the rod be ω. The moment of inertia of the rod
1
about C is IC = M l2 , where C is the center of the rod.
12

The conservation of linear momentum:

mv0 = m v + M vM (12.4)

The conservation of kinetic energy due to the elastic collision:


1 1 1 1
mv02 = mv 2 + M vM2
+ IC ω 2 (12.5)
2 2 2 2
The conservation of angular momentum about C:
   
l l
(0) M (0) + mv0 = mv + IC ω
2 2
which gives
l l
mv0 = mv + IC ω (12.6)
2 2
Solving equations (12.4), (12.5), and (12.6), we have
12 m v0
ω =
(4 m + M ) l
2 m v0
vM =
4m + M
(4 m − M ) v0
v =
4m + M
Chapter 12 Conservation of Energy 213

(i) When 4m > M , the particle moves forward.


(ii) When 4m = M , the particle is stopped.
(i) When 4m < M , the particle moves backward.

Example
A stepladder consists of two legs held together by a hinge at the top and a horizontal
string near the bottom, and it rests on a horizontal surface as shown in figure. If the string
is suddenly cut, determine the acceleration of the hinge at that instant by the following
methods. The coordinate system has been set in the figure for your reference. Assume
that the legs to be uniform, identical to each other with mass m and length l, and neglect
all frictions.
[Hint: Let the coordinates of the centre of the right rod be (x, y) and consider its motions.]

(a) The conservation of mechanical energy.

(b) The Newton’s laws of motion (both translational and rotational motions).

Solution
(a)
Chapter 12 Conservation of Energy 214

Consider the right leg of the ladder. The center of mass of the right leg is located at
l


 x = sin θ

 2

 y = l cos θ



2
which gives
l


 ẋ = θ̇ cos θ

 2

 ẏ = − l θ̇ sin θ



2
Then, we have
l l


 ẍ = θ̈ cos θ − θ̇2 sin θ

 2 2
(12.7)
 ÿ = − l θ̈ sin θ − l θ̇2 cos θ



2 2
Let θ0 be the initial angle that the ladder made with the vertical, where θ0 = 30◦ . The
conservation of mechanical energy gives
         
l l 1 2 2 1 1
2 mg cos θ0 − mg cos θ = 2 m (ẋ + ẏ ) + ml θ̇2
2
2 2 2 2 12
1 2 2
gl [cos θ0 − cos θ] = ẋ2 + ẏ 2 + l θ̇
12
1 2 2 1 2 2
gl [cos θ0 − cos θ] = l θ̇ + l θ̇
4 12
1 2
g [cos θ0 − cos θ] = l θ̇
3
Differentiating both sides with respect to time, we get
1
g θ̇ sin θ = l (2 θ̇ θ̈)
3
So, we have
2
g sin θ = l θ̈
3
When θ = θ0 , we have
2
g sin θ0 = l θ̈0
3
3g
θ̈0 = sin θ0
2l
Chapter 12 Conservation of Energy 215

At t = 0, θ = θ0 = 30◦ and θ̇ = θ̇0 = 0, the second equation in (12.7) gives


    
l ◦ l l 3g 1 3g
ÿ0 = − sin 30 θ̈0 = − θ̈0 = − =−
2 4 4 2l 2 16

Let yA be the coordinate of the hinge at A. Since yA = 2 y, we get ÿA = 2 ÿ, then
 
3g 3g
ÿ0A = 2 ÿ0 = 2 − =− ,
16 8
where ÿ0A is the acceleration of the hinge just after the string is cut.
(b) Consider the forces acting on the right leg of the ladder.

Rewrite the center of mass of the right leg of the rod again.
l


 x = sin θ

 2

 y = l cos θ



2
gives
l


 ẋ = θ̇ cos θ

 2

 ẏ = − l θ̇ sin θ



2
Then, we have
l l


 ẍ = θ̈ cos θ − θ̇2 sin θ

 2 2

 ÿ = − l θ̈ sin θ − l θ̇2 cos θ





2 2
Chapter 12 Conservation of Energy 216

Knowing that θ0 = 30◦ and θ̇0 = 0 when t = 0, we have


 √ √
 l 3 3l

 ẍ 0 = · θ̈ 0 = θ̈0
2 2 4


(12.8)


 l 1 l
 ÿ0 = − · θ̈0 = − θ̈0

2 2 4
Consider the translational and rotational motions of the right leg of the ladder.

N = m ẍ (12.9)
R − mg = m ÿ (12.10)
   
l l 1
R sin θ − N cos θ = ml2 θ̈ (12.11)
2 2 12
Equations (12.9) and (12.10) give
√ !
3l
N0 = m θ̈0 (12.12)
4
R0 − mg = m ÿ0 (12.13)

Equation (12.11) becomes


   
l l 1
R0 sin θ0 − N0 cos θ0 = ml2 θ̈0
2 2 12

Using equations (12.12) and (12.13), we have


   √ !  √ !
l 1 m 3l l 3 1
(mg + m ÿ0 ) − θ̈0 = ml2 θ̈0
2 2 4 2 2 12
1 3l 1
(g + ÿ0 ) − θ̈0 = l θ̈0
4 16 12
13 l
g + ÿ0 = θ̈0
12
−4
Using the second equation in (12.8), we have θ̈0 = ÿ0 , then
l
 
13 l −4
g + ÿ0 = ÿ0
12 l
3g
ÿ0 = −
16
Since yA = 2 y, we get ÿA = 2 ÿ. Therefore, the acceleration of the hinge just after the
string is cut is given by
 
3g 3g
ÿ0A = 2 ÿ0 = 2 − =− .
16 8
Chapter 13

Gravitation

13.1 Newton’s law of universal gravitation


Two particles, each carrying mass, exert forces on each other. The nature of the force is
attractive. The magnitude of the force is linearly proportional to the mass of individual
particle and is inversely proportional to the square of the distance between particles. It
is also known as the inverse square law. Let’s consider the gravitational force between
particles m1 and m2 , as shown in the figure.

m1 F21 m2 ~r21 : position of m2 relative to m1


r21 F~21 : force experienced by m2 due to m1
Gm1 m2
F~21 = − 2
r̂21
r21

m1 F12 m2 ~r12 : position of m1 relative to m2


r12 F~12 : force experienced by m1 due to m2
Gm1 m2
F~12 = − 2
r̂12
r12

The constant G = 6.674 × 10−11 N·m2 /kg2 is called the universal gravitational constant.

217
Chapter 13 Gravitation 218

13.2 Gravitational field


A mass M exerts gravitational force on the objects around it. The gravitational force ex-
perienced by an object of unit mass at a distance R from M is defined as the gravitational
field, where
GM m
F 2 GM
g= = R = 2
m m R
Mostly, we refer to the force due to the earth’s attraction and the earth of mass ME is
assumed spherical in shape. We should note that g is also the gravitational acceleration
of the object due to gravitational pull. Consider an object of mass m, then

GME m
F 2 GME
g= = R =
m m R2
If an object is placed on the earth surface or near the earth surface, we obtain the gravi-
tational force F0 and the gravitational field g0 respectively.

m ME = Mass of the earth


RE = Radius of the earth
RE Gravitation pull on the mass m:
ME
GME m GME
F0 = 2
and g0 = 2
= 9.8 m/s2
RE RE
Chapter 13 Gravitation 219

13.3 Gravitational force due to a spherical shell


Shell theorem 1:
A uniform spherical shell attracts an external particle as if all the mass of the shell was
concentrated at the center.

Shell theorem 2:
A uniform spherical shell exerts no force on a particle located inside the shell.

As a reminder, shell theorems are valid when we deal with uniform and spherical objects.
For example, the gravitational force produced by a ring is not the same as the force
produced by a point mass which has all masses of the ring concentrated at the center of
the ring. Interestingly, the gravitational force exerted on a particle of mass m at point P
having a normal distance x from the center of a ring of mass M and radius R is
GM mx
F = 3
(R2 + x2 ) 2

GM m
It is not the inverse square law given by !
x2

Example
Determine the gravitational field at a point P due to a uniform and thin rod of mass m and
length l, where P is located at a distance a from one end of the rod, as shown in the figure.
Chapter 13 Gravitation 220

Solution
Divide the rod into numerous segments, each of infinitesimal length dx and mass dm =
m
λ dx, where λ represents the density of the rod, i.e. λ = . The gravitational field due
l
to the segment is
G dm G λ dx
dg = 2
=
x x2
Its direction points toward the right.
So, the gravitational field due to the entire rod is
Z Z a+l a+l  
G λ dx G λ 1 1 Gm
g = dg = 2
=− = −G λ − =
a x x a a+l a a (a + l)

13.4 Gravitational field due to the earth


The variation of the gravitational field is shown in the figure. Outside the earth (r > RE ),
the field follows the inverse square law according to shell theorem 1. It is a curve.
GME
g= (r > RE )
r2
Chapter 13 Gravitation 221

Inside the earth (r < RE ), the field varies linearly with r. The explanation is stated as
follows. Consider that the earth is a collection of numerous shells and a particle of unit
mass is placed at a distance r < RE inside the earth. Then, the shells with radii greater
than r exert no force on the particle according to shell theorem 2. However, the shells
having radii less than r exert forces on the particle. Let the total mass of those regions
be M . We may now apply shell theorem 1 again and replace those shells by a point mass
M situated at the center of the earth. The gravitational field inside the earth is
 
4 3  
G πr ρ
GM 3 4 4  ME  GME
g= 2 = = G π rρ = G πr  = r (r < RE )
r r 2 3 3 4 3
3
RE
πRE
3
It is a linear relation which has a zero value at the center of the earth. The maximum
value occurs at r = RE .

13.5 Effect of earth’s rotation

Consider a mass m hanged by a string and the string is deviated from the true vertical
line through center of earth.
F~c = m ~g0 +T~
|{z}
true
vertical
direction
Consider the force diagram in the right figure. Use the Cosine Law, we can write

T 2 = Fc2 + (mg0 )2 − 2Fc mg0 cos φ


Chapter 13 Gravitation 222

Note that T = mgeff where geff is the effective measured gravity.

For the case at the equator, i.e. φ = 0.

T 2 = Fc2 + (mg0 )2 − 2Fc mg0


⇒ T 2 = (mg0 − Fc )2
⇒ |T | = mgeff = mg0 − Fc
⇒ |T | = mgeff = mg0 − mω 2 RE

Therefore, when φ = 0

geff = g0 − ω 2 RE

Let α be the angle between the string and the real vertical axis.

Using Sine Law and let R be the distance between the particle and the z-axis.
sin α sin(π − α − φ)
=
Fc mg0
g0 sin α
⇒ = sin(α + φ)
ω2R
g0 sin α
⇒ = sin α cos φ + cos α sin φ
ω2R
g0
⇒ = cos φ + cot α sin φ
ω2R
g0
⇒ = cot φ + cot α
ω 2 R sin φ
g0 − ω 2 R cos φ
⇒ cot α =
ω 2 R sin φ

2 (g0 − ω 2 RE cos2 φ)
Or we can write, cot α =
ω 2 RE sin 2φ
Chapter 13 Gravitation 223

Using the Sine law again,


sin α sin φ
=
Fc T
sin α sin φ
⇒ 2
=
mω RE cos φ mgeff
RE ω 2
⇒ sin α = sin φ cos φ
geff
RE ω 2
⇒ sin α = sin 2φ
2geff

RE ω 2 π
As α is small, α ' sin α = sin 2φ. The maximum of α occurs when φ = and
2geff 4
thus

RE ω 2
αmax =
2geff

13.6 Gravitational potential energy

ra

m
111
000
000
111
000
111
000
111
M F a dr b

rb

Recall: ∆U = Uf − Ui = −Wif .
Consider a mass m displaces from a to b.
Z b Z rb
GM m
Wab = F~ · d~r = − dr
a ra r2
 rb
1
= −GM m −
r ra

 
1 1
= +GM m −
rb ra
Chapter 13 Gravitation 224

If rb > ra , Wab < 0; if rb < ra , Wab > 0.


Notice that Wab is the work done by the gravitational force in bringing m from a to b.

∴ In bringing the mass m from a to b,


 
1 1
∆U = Ub − Ua = −Wab = −GM m −
rb ra
Now, we take ra = r, Ua = U (r) and rb → ∞, where U (∞) = 0.
 
1
U (∞) − U (r) = −GM m −
r
GM m
∴ U (r) = − = Wr∞
r

Escape speed
A particle having an initial speed is launched on the surface of the earth. For the particle
to escape from the earth’s gravitational force field, it is energetic possible for it to travel
to infinity. The minimum speed for the particle to escape the earth’s attraction is called
the escape speed vesc , where
 
1 2 GM m
mv + − = 0
2 esc RE
r
2GM
vesc =
RE
Example
If an object is launched using the escape speed, find the time required when it reaches a
height RE from the surface of the earth, where RE is the radius of the earth.

Solution
Let the mass of the object be m, the equation of motion of the object:
GM m
− 2 = m r̈
r
It can be rewritten as
GM dṙ
− 2 = ṙ
Z rr Z dr
v
dr
−GM 2
= ṙ dṙ
RE r vesc
r 2 v

GM ṙ
=
r RE 2 vesc
 
1 1 1 2 2
GM − = (v − vesc )
r RRE 2
Chapter 13 Gravitation 225

2 2 GM
Substituting vesc = , then we have
RE
2 GM
v2 =
r
Hence,
r
dr 2 GM
=
dt r
Separating the variables and integrating both sides of the equation, we have
Z 2 RE
√ √ Z t
r dr = 2 GM dt
RE 0
2 R
2 3 E p
r2 = 2 GME t
3 RE
2 1 3 3
t = ·√ [(2 RE ) 2 − (RE ) 2 ]
3 2 GME
After simplification, we have
3
2 R2 3
t = √ E [2 2 − 1]
3 2 GM

13.7 Potential energy of many-particle system


m2

r12 r23

m1 r13
m3

 
Gm1 m2 Gm1 m3 Gm2 m3
U =− + +
r12 r13 r23

It is the formation energy of the mass system started from nothing. Particles are then
introduced one by one from infinity to specific positions by external agent. Thus, the
energy required by an external agent to take these three particles to separated infinity:
E = −U
Chapter 13 Gravitation 226

Example
Refer to the last figure, determine the work done required by an external agent to remove
m3 from the system.
Solution
Let Wext be the work done required by the external agent. Knowing that

Wext = ∆U

then we have
  
Gm1 m2 Gm1 m2 Gm1 m3 Gm2 m3
Wext = − − − + +
r12 r12 r13 r23
Gm1 m3 Gm2 m3
= +
r13 r23

13.8 Energy consideration of satellite motion

Consider a satellite orbiting a planet.


M
GM m
ω U =−
r
r 1 1 1
m K = mv 2 = m(ωr)2 = mω 2 r2
2 2 2

If the gravitational force provides the centripetal force,


GM m GM
2
= mω 2 r ⇒ = ω 2 r2
r r
1 GM m
∴ K=
2 r
1 GM m GM m GM m
∴ E =K +U = − =−
2 r r 2r
Chapter 14

Oscillations

14.1 Simple harmonic motion


A light and unstretched spring has one end fixed while the next end connected to a block
of mass m. The block is acted by a force such that the spring extends by a length A. The
system is set free to move thereafter and it oscillates.

The governing force on the block is the restoring force F which obeys the Hooke’s law,
F~ = −k~x, where k is a positive constant, or the force constant, x is the displacement
of block. Notice that the restoring force is always opposite to the displacement of block.
When the spring is extended, the force is a tensive one; when the spring is shortened, the
force is a compressive one. Both forces tend to recover the natural length of spring. The
equation of motion of block becomes

−kx = mẍ
k
⇒ ẍ = − x
m
The acceleration is linearly proportional to the displacement of the object, but the direc-
tions of them are opposite to each other. It is referred to as the simple harmonic motion
(SHM),

ẍ = −ω 2 x

227
Chapter 14 Oscillations 228

k p
where ω 2 = and ω = k/m is the angular speed of the oscillation. The period of
m p
oscillation is T = 2π/ω = 2π m/k.
dẋ
Since ẍ = ẋ , we have
dx
dẋ
ẋ = −ω 2 x
Z v dx Z x
2
⇒ ẋ dẋ = −ω x dx
0 A

⇒ v 2 = ω 2 (A2 − x2 )
Chapter 14 Oscillations 229

The speed is a maximum, ω 2 A2 , when the spring returns to the natural length, i.e. un-
stretched and uncompressed. The restoring force at this instant is zero. When the block
is at the extreme positions, x = ±A, the block is stationary and is said to be located
at the amplitude of oscillation. The acceleration so obtained has the largest magnitude,
ω 2 A, and the restoring force is the greatest, kA.

One can verify that the simplest solutions of ẍ = −ω 2 x could be x = A cos ωt or


x = A sin ωt. The selection of them depends on the initial conditions. If the initial
time t = 0 is measured when the block is at the positive extreme position and it begins
to return, x = A cos ωt. If the initial time t = 0 is measured when the block is at the
equilibrium position and it is moving along the positive direction, x = A sin ωt. If the
initial time t = 0 is measured at an arbitrary instant, an extra phase angle is inserted, e.g
x = A sin(ωt + φ).

14.2 Time varying quantities in SHM


We have studied the governing equation of SHM ẍ = −ω 2 x in the last section. The
velocity of the object varies with the displacement of it as follows.

v 2 = ω 2 (A2 − x2 )

Now, we try to obtain the displacement-time relation of the object. Since v = dx/dt, we
can write
dx √
= ω A2 − x2
dt
Separating the variables and integrating both sides, we get
Z x Z t
dx
√ = ω dt
A A2 − x2 0
Chapter 14 Oscillations 230

x
Z sin−1 ( A )
Substitute x = A sin θ, we have dx = A cos θ dθ and dθ = ωt, this becomes
π
2

x π
sin−1 − = ωt
A 2
Thus,
x π 
= sin + ωt
A 2
x
= cos ωt
A
x = A cos ωt

Hence, we obtain v = ẋ = −Aω sin ωt and a = ẍ = −Aω 2 cos ωt = −ω 2 x.

In summary, in a spring-mass system, if the particle is released at x = A it will oscillate


with amplitude A about the equilibrium position, i.e. x = 0. The equation of motion is
ẍ = −ω 2 x and

 x = A cos ωt


v = −Aω sin ωt

 a = −Aω 2 cos ωt

p
where ω = k/m is the angular velocity. The maximum magnitude of velocity is
2
vmax = ωA when x = 0, and the maximum magnitude of acceleration r is amax = ω A
2π m
when x = A or x = −A. The period of oscillation is T = = 2π . It is worth to
ω k
notice that the velocity relates the displacement by v 2 = ω 2 (A2 − x2 ). This expression is
also the direct consequence of conservation of energy.

Remark r
k
In the early beginning, we have set ω = when we obtained ẍ = −ω 2 x. Then, what
m
is the nature of this constant? Why is it named as the angular frequency of oscillation?
In fact, the motion of the particle is periodic because
  

x = A cos ωt = A cos(ωt + 2π) = A cos ω t +
ω

If we set T = , we have x = A cos [ω (t + T )] , where T is the period of oscillation.
ω
Therefore, ω is the angular frequency of oscillation.
Chapter 14 Oscillations 231

Example
A simple pendulum has a bob of mass m connected to one end of a light string of length
L, while the next end is attached to a fixed point P . The system is displaced slightly, it
oscillates to-and-fro along a small arc of a circle of center P . Find the equation of motion
of the system if the oscillation is small.

Solution

For small oscillation, the angle θ is small, the tangent force along the arc of circle is
roughly horizontal, it is given by

−mg sin θ ≈ −mgθ.

It is nearly horizontal, as θ is small. On the other hands, the horizontal displacement of


bob from O is x = L tan θ ≈ Lθ. Therefore, the equation of motion becomes

−mgθ = mẍ
x
⇒ −mg ( ) = mẍ
L
g
⇒ ẍ = − x
L
p p
The angular speed ω = g/L, and the period of oscillation is T = 2π/ω = 2π L/g.

Example
The liquid in a uniform U -tube is disturbed such that the liquid oscillates inside it. At
time t, the liquid in one limb is higher than the equilibrium line by x. Find the equation
of motion of the liquid if friction is ignored. The total length of liquid in the tube is 2h.
Chapter 14 Oscillations 232

Solution
The excess pressure on the whole liquid equals the pressure difference on the two sides of
S at the bottom of U -tube , i.e.

Excess pressure = excess height × liquid density × g = 2xρg.

Since pressure = force per unit area, we have


the force on liquid = pressure × area of cross-section of the tube = (2xρg) A. The
mass of liquid inside the tube is 2hAρ. Therefore, the equation of motion of the liquid
is −2xρgA = 2hAρẍ. The negative sign appears as the direction of the restoring force is
opposite to the displacement of liquid. After simplification, we obtain
g
ẍ = − x = −ω 2 x,
h
p
where ω 2 = g/h. The period of oscillation is T = 2π/ω = 2π h/g.

Example
A spring has force constant k. If the spring is cut into two equal halves, find the force
constant of the new springs.

Solution
Before cutting:
If a force F is applied to the spring such that there is an extension e in the spring, we
can write F = ke.
Chapter 14 Oscillations 233

After cutting:
The new pieces have the same length which is half of the original one. Let k 0 be the force
constant of the new springs. Imagine that they are connected as shown in the figure and
e
a force F is applied to one end of it again, then the extension of each spring is . So, we
e 2
0 e
 
0 0
have F = k . Hence, ke = k gives k = 2 k. To conclude, a shorter spring has
2 2
a greater force constant.

14.3 Phase diagram


Consider the one-dimensional motion, it is governed by the second order differential equa-
tion. We need two initial conditions x(t0 ), ẋ(t0 ) to determine the motion.
In general, let’s consider {x(t), ẋ(t)} as a point in a two-dimensional space called phase
space. As t varies, the point P (x, ẋ) describing the state of the oscillating particle will
move along a certain phase path in the phase space, all the phase paths constitutes the
phase diagram of the oscillator. Recall that

ẋ2 = ω 2 (A2 − x2 )

Rearranging it, we have the phase diagram described by


x2 ẋ2
+ =1
A2 A2 ω0 2
It is a family of ellipse on the diagram and each ellipse satisfies

x(t) = A sin(ω0 t − δ)
ẋ(t) = A ω0 cos(ω0 t − δ) .

Since the total energy of the system is


1
E = kA2
2
Chapter 14 Oscillations 234

therefore
x2 ẋ2
2E
+ 2E
=1.
k m

Figure 14.1: The phase path

Each phase path corresponds to a definite total energy of the oscillator (E = constant).
*No two phase paths of the oscillator can cross. If they could cross, this would imply that
for a given set of initial condition x(t0 ), ẋ(t0 ), (i.e. the coordinates of the crossing point),
the motion could proceed along different phase paths. But this is impossible since the
solution of the differential equation is unique.
*The motion of the representative point P (x, ẋ) will always be in a clockwise direction.
This is because, say, in the upper half plane of figure (14.1), when x > 0, ẋ is always
decreasing, while for x < 0, ẋ is always increasing.

14.4 Potential energy and kinetic energy in a spring-


mass system
In a spring-mass system, the spring is extended from its natural length by using an applied
force F . If the extension is x, the total work done by the force is
Z x Z x
1
F dx = kx dx = kx2 .
0 0 2
The displacement x could be negative when the spring is being compressed. However,
the expression is the same for extension and compression. This work done also gives the
potential energy stored in the spring. Thus, the total mechanical energy of an oscillating
Chapter 14 Oscillations 235

mass connected with it is


1 2 1 2 1 1
mv + kx = mω 2 A2 = kA2 = E = constant.
2 2 2 2
We have applied the relation v 2 = ω 2 (A2 − x2 ). In the figures below, U and K are the
potential energy and kinetic energy respectively.

Example
A light spring of spring constant k has one of its end fixed at O on a smooth incline and
the next end of it is attached to a particle of mass m. The length of the spring is along
the greatest slope of the incline which has an elevated angle θ = 30◦ . Initially, the particle
is held at a position such that the spring force is zero. Let’s mark this position as A on
the incline. Then it is released and it descends along the incline. The particle reaches the
equilibrium position at B and the lowest point of the motion at C, as shown in the figure.

(a) If e is the extension of the spring when the particle reaches B, find e.

(b) Find the speed of the particle when it reaches B. Express your answer in terms of m,
g, and k.

(c) If d is the extension of the spring when the particle reaches C, find d.

(d) Show that the particle performs the simple harmonic motion about point B. What
are the amplitude and the period of motion?
Chapter 14 Oscillations 236

Solution
mg
(a) ke = mg sin 30◦ gives e = .
2k
(b) By the conservation of mechanical energy, we have
1 1
−mge sin 30◦ + mv 2 + ke2 = 0
2 2
 mg   1  1 1  mg 2
−mg + mv 2 + k = 0
2k 2 2 2 2k
mg 2
v2 =
r4k
mg 2
v =
4k
(c) At the lowest point C, the speed of the particle is zero. By the conservation of
mechanical energy again, we have
1
−mgd sin 30◦ + ke2 = 0
  2
d 1
−mg + k d2 = 0
2 2
mg
d =
k
(d) Let x be the displacement of the particle measured from B down the incline. The
upward force acting on the particle is the restoring force k (e + x) while the downward
force on it is the down plane force mg sin 30◦ . So, the particle travels with acceleration a
and it is given by

mg sin 30◦ − k (e + x) = ma
mg  mg 
−k − kx = ma
2 2k
−kx = ma
k
a = − x
m
It is r
the governing equation of a simple harmonic motion, where the angular frequency is
k
ω= . The particle vibrates about B with amplitude AB = BC = e, and the period
m r
2π m
of oscillation is T = = 2π .
ω k

Example
Three identical springs having force constant k = 2 N/m and natural length 1 m lie on the
medians of an equilateral triangle on a smooth and horizontal plane. Each of them has
Chapter 14 Oscillations 237

one end fixed on the vertex of the triangle and the next end attaches to a particle of mass
m at the centroid G of the triangle. The springs are unstretched as shown in the figure.
The mass is displaced to point P and then released. What will be the kinetic energy of
m when it returns to the centroid?

Solution
Initially, the springs occupy no elastic potential energy because they are unstretched.
When the mass is displaced to P , spring a is extended, and spring b and c are compressed.
The length of extension, GP = 0.5 m and the length of compression is given by 1 − CP =
1 − 1 cos 30o = 0.134 m. By the conservation of mechanical energy, we have

The kinetic energy of m at G = The total elastic potential energy stored at P

Therefore,
1 2 1 2 1 2
K.E. = kx + kx + kx
2 a 2 b 2 c
1 1 1
K.E. = (2N/m) (0.5m)2 + (2N/m) (0.134m)2 + (2N/m) (0.134m)2
2 2 2
K.E. = 0.286 Joule

Example
Two massless springs, each with force constant k and unstretched length l0 are connected
in a straight line as shown in the left figure.
Chapter 14 Oscillations 238

Find an expression for the work done which moves the point of attachment between the
two springs a perpendicular distance x from the equilibrium point. Show that the work
kx4
done for such movement is given by 2 when x << l0 .
4l0
p · (p − 1) 2 p · (p − 1) · (p − 2) 3
[Hint: (1 + y)p = 1 + py + y + y + · · · , where p is real.]
1·2 1·2·3

Solution
Denote the extension of each spring by e when the joint of springs is displaced by x.
q
e = l02 + x2 − l0

As the horizontal component of the restoring force of each spring


! cancels each other, the
x
q
applied force F = 2ke sin θ = 2k ( l02 + x2 − l0 ) p 2 . The work done by the
l0 + x2
force is
Z x
W = F dx
0
Z x !
l0 x
W = 2k x− p 2 dx
0 l0 + x2
q
W = kx2 − 2kl0 l02 + x2 + 2kl02
p
Using the hint, we obtain the binomial expression of l02 + x2
(  2 )1/2
x
q
l02 + x2 = l0 1 +
l0
1 x2 1 x4
 
= l0 1 + − + ... ...
2 l02 8 l04
When x  l0 ,
1 x2 1 x 4
  
2
W = kx − 2kl0 l0 1 + − + 2kl02 .
2 l02 8 l04
kx4
Therefore, W = .
4l02
Chapter 14 Oscillations 239

14.5 Second order differential equations


In a spring-mass system, the equation of motion is
k
mẍ = −kx, where ω02 ≡
m
We have
ẍ + ω02 x = 0 , (14.1)

it is a second order linear ordinary differential equation. Readers should know the meaning
of some useful terms about differential equations.

* second order: involves ẍ

* linear: involves only ẍ, ẋ or x but not x2 or in general xn where n > 1 and is integer.
For example ẍ + ẋ2 + x = 0 is not a linear equation.

* ordinary: only one variable t is involved, not partial derivatives.

* homogeneous and with constant coefficients: e.g. the R.H.S. of Eq.(14.1) is zero
and every term in the L.H.S. of Eq.(14.1) contains x or its derivatives only. The
coefficient of each term is a constant.

As long as the displacements are small and the elastic limits are not exceeds, a linear
restoring force can be used for problems of stretched springs, elastic springs, bending
beams, etc. But we must emphasize that such calculations are only approximation, be-
cause essentially every restoring force in nature is more complicated than the simple
Hooke’s law force.

* Damped oscillations —— usually resulted from friction.

* Driven (or force) oscillation

This damping oscillation may be counteracted if some mechanisms supply energy to the
system from an external source at a rate which equals to that absorbed by damping
medium.

Solutions of second order differential equations

d2 y dy
2
+a + by = f (x)
dx dx
or y 00 + ay 0 + by = f (x) .
Chapter 14 Oscillations 240

(I) For linear homogeneous equation:

y 00 + ay 0 + by = 0 . (14.2)

This equation has the following important properties:

a . If y1 (x) is a solution of Equation(14.2), then c1 y1 (x) is also a solution.

b . If y1 (x) and y2 (x) are solutions, then y1 (x) + y2 (x) is also a solution.

c . If y1 (x) and y2 (x) are linearly independent solutions, then the complementary
solution is given by y = c1 y1 (x) + c2 y2 (x)

Let y = erx , so y 0 = rerx , y 00 = r2 erx


Using these expressions for y 0 and y 00 in Eqn(14.2), we find a characteristic equation

r2 + ar + b = 0

with solution
a 1√ 2
r=− ± a − 4b .
2 2
The complementary solution for Eqn(14.2)

y = c1 er1 x + c2 er2 x r1 6= r2
y = c1 erx + c2 xerx r1 = r2 ≡ r .

If the root r1 and r2 are imaginary, the solutions given by c1 er1 x and c2 er2 x are still correct.
We may write the solution alternatively as

y = µeαx sin(βx + δ)
y = µeαx cos(βx + δ) .

(II) For linear inhomogeneous equation:

y 00 + ay 0 + by = f (x) .

Let yc be the complementary solution of

y 00 + ay 0 + by = 0

and let yp be the particular solution of

y 00 + ay 0 + by = f (x) (14.3)
Chapter 14 Oscillations 241

then y = yc + yp is a general solution of Eqn(14.3), because

y 00 + ay 0 + by = (yc00 + ayc0 + byc ) + (yp00 + ayp0 + byp )


= 0 + f (x) .

Example
Solve the differential equations.
d2 x
(a) + 9 x = 0.
dt2
d2 x
(b) + 9 x = 2 sin 4 t.
dt2
Solution
(a) This equation is sometimes written as ẍ + 9 x = 0. The characteristic equation of it
is λ2 + 9 = 0, which gives λ = ±3 i. So x = A sin(3 t + δ), where A and δ are constants
to be determined by the initial conditions.
(b) The complementary solution of the equation is xc = A sin(3 t + δ). The particular
solution of the equation can be obtained by the method of undetermined coefficients. We
substitute the trial function yp = B1 sin 4 t + B2 cos 4 t into the equation which leads the
2
answers of B1 and B2 . Here, we have B1 = − and B2 = 0. It is important to note that
7
B2 must be zero as the LHS of the equation has even order differential operators only but
the inhomogeneous term is a sine function. Therefore, the term ”B2 cos 4t” in the trial
2
function can always be ignored. The particular solution is xp = − sin 4 t. Hence, the
7
general solution is
2
y = yc + yp = A sin(3 t + δ) − sin 4 t
7

14.6 Damped oscillation


The motion of the simple harmonic oscillation is called a free oscillation. Actually, in
physical case, there are dissipative or frictional force. In the presence of retarding force

F~r = −b~v ,

where b > 0, we have

mẍ = −kx − bẋ ,


or mẍ + bẋ + kx = 0 .
Chapter 14 Oscillations 242

b
We label the damping parameter as β (β ≡ ), and the characteristic angular frequency
r 2m
k
as ω0 (ω0 ≡ ). Now we obtain
m
ẍ + 2β ẋ + ω0 2 x = 0

set x = ert , we have auxiliary equation

r2 + 2βr + ω0 2 = 0

The roots are:


p
r1 = −β + β 2 − ω0 2
p
r2 = −β − β 2 − ω0 2 .

The general solution is


p p
x(t) = e−βt [A1 exp( β 2 − ω0 2 t) + A2 exp(− β 2 − ω0 2 t)] .

There are three general cases of interest:


(1) Underdamping: β 2 < ω0 2 , (see figure 14.2)
(2) Critical damping: β 2 = ω0 2 , (see figure 14.3)
(3) Overdamping: β 2 > ω0 2 , (see figure 14.3)

The motion of three cases is shown schematically in figure 14.3 for specific initial condi-
tions.

Figure 14.2: The decreasing amplitudes in a damped oscillation

For the phase diagram of damped oscillator, we first, write down the expressions for the
displacement and velocity

x(t) = Ae−βt cos(ω1 t − δ) (14.4)


ẋ(t) = −Ae−βt [β cos(ω1 t − δ) + ω1 sin(ω1 t − δ)] (14.5)
Chapter 14 Oscillations 243

Figure 14.3: The types of damping oscillation

Underdamped motion (β 2 < ω0 2 )

For the case of underdamped motion, it is convenient to define

ω1 2 ≡ ω0 2 − β 2

where ω1 2 > 0, then

x(t) = e−βt [A1 eiω1 t + A2 e−iω1 t ]


or = e−βt A cos(ω1 t − δ)

ω1 is the angular frequency of the damped oscillator.


* The maximum amplitude of the motion of the damped oscillator decreases with time
because of the factor e−βt , β > 0 and the envelope of displacement versus time curve is
given by
xen = ±A e−βt .

Critical damped motion (β 2 = ω0 2 )

if β 2 = ω0 2 , the roots of the auxiliary equation are then equal. Therefore

x(t) = (A + Bt) e−βt .

i.e. no oscillation at all.


Chapter 14 Oscillations 244

Overdamped motion (β 2 > ω0 2 )

p
Let ω2 = β 2 − ω0 2
x(t) = e−βt [A1 eω2 t + A2 e−ω2 t ]

note that ω2 < β, so x(t) will not go to infinity as t is very large. Now the motion is not
periodic.
ẋ(t) = A1 (ω2 − β) e(ω2 −β )t + A2 (−β − ω2 ) e−(ω2 +β) t

14.7 Driven oscillation


The simplest case of driven oscillation is that in which an external driving force varying
harmonically with time is applied to the oscillator. The total force on the particle is then

F~ = −kx − bẋ + F0 cos ωt

where we consider a restoring linear force and a viscous damping force in addition to the
driving force. The equation of motion becomes

mẍ + bẋ + kx = F0 cos ωt


or ẍ + 2β ẋ + ω0 2 x = A cos ωt , (14.6)
F0 k b
where A = , ω0 2 = , and β = . The solution of Eqn(14.6) consists two parts:
m m 2m
the complementary solution + particular solution i.e.

xc (t) + xp (t) .

We already have the complementary solution for the homogeneous equation, the only
thing we need is the particular solution. For the particular solution, we try

 xp (t) = D cos(ωt − δ)


ẋ = −Dω sin(ωt − δ)

 ẍ = −Dω 2 cos(ωt − δ)

substitute back into Eqn(14.6), we have

−Dω 2 cos(ωt − δ) − 2βDω sin(ωt − δ) + ω0 2 D cos(ωt − δ) = A cos ωt .

Since (
cos(ωt − δ) = cos ωt cos δ + sin ωt sin δ
sin(ωt − δ) = sin ωt cos δ − cos ωt sin δ .
Chapter 14 Oscillations 245

Therefore

D(ω0 2 − ω 2 )(cos ωt cos δ + sin ωt sin δ) − A cos ωt − 2βDω(sin ωt cos δ − cos ωt sin δ) = 0

or

{A − D[(ω0 2 − ω 2 ) cos δ + 2ωβ sin δ]} cos ωt − {D[(ω0 2 − ω 2 ) sin δ − 2ωβ cos δ]} sin ωt = 0

therefore

(ω0 2 − ω 2 ) sin δ − 2ωβ cos δ = 0


2ωβ
⇒ tan δ =
ω0 2 − ω 2
or
2ωβ

 sin δ = p
2 − ω 2 )2 + 4ω 2 β 2



 (ω0

ω 2 − ω2


 cos δ = p 2 0 2 2 .


(ω0 − ω ) + 4ω 2 β 2
So
A
D = 2
(ω0 − ω 2 ) cos δ + 2ωβ sin δ
Then
A
D = p (14.7)
(ω0 2 − ω 2 )2 + 4ω 2 β 2

and
A
xp = p cos(ωt − δ)
(ω0 2 − ω 2 )2 + 4ω 2 β 2

where
2ωβ
δ = tan−1 ( )
ω0 2 − ω 2
The general solution is

x(t) = xc (t) + xp (t)


√ √
−βt β 2 −ω0 2 t − β 2 −ω0 2 t
= e (A1 e + A2 e ) + xp (t) .

* The quantity δ represents the phase difference between the driving force and the resul-
tant motion; a real delay occurs between the action of the driving force and the response
Chapter 14 Oscillations 246

of the system.
* xc (t) here represents transient effects (i.e. the effect that dies out), and the terms con-
tained in this solution damp out with time because of the factor e−βt . The term xp (t)
represents the steady- state effects and contains all the information for t large compared
1
with .
β
1
For large t  , x(t) ≈ xp (t) which is a steady state solution.
β

Figure 14.4: The complementary and particular solutions

14.8 Resonance phenomena


Consider the frequency ωR which makes the maximum amplitude D. (i.e. the amplitude
resonance frequency). We set

dD
=0
dω ω=ωR
Chapter 14 Oscillations 247

Alternatively, we can consider the completing the square in Eqn(14.7) to obtain the min-
imum of the denominator in Eqn(14.7). Hence, the maximum of D is obtained when

⇒ ωR 2 = ω0 2 − 2β 2
p
ωR = ω0 2 − 2β 2

The maximum amplitude is given by


A
Dmax = p 4
ω0 − ωR4
And, we define the quality factor as
ωR
Q≡

Figure 14.5: The Q factors

Figure 14.6: The variation of phase angle with the driving frequency
Chapter 15

Wave Motions

15.1 Mechanical waves


A physical medium is being disturbed and the disturbance propagates through it. This
propagation is called a mechanical wave. However, the matter of the medium does not
transfer over a distance when the energy is transferred. Examples of mechanical waves
are string wave, sound wave, and water wave, etc. The waves that can propagate without
a medium are electromagnetic waves.

Types of waves
(I) Transverse wave
A traveling wave or pulse that causes the elements or the disturbed medium to move
perpendicular to the direction of propagation is called a transverse wave. In the figure,
the particle motion and the direction of propagation is normal to each other.

248
Chapter 15 Wave Motions 249

(II) Longitudinal wave


A traveling wave or pulse that causes the elements or the disturbed medium to move
parallel to the direction of propagation is called a longitudinal wave. In the figure, the
displacement of the coils is parallel to the propagation.

15.2 Sinusoidal waves


A continuous wave can be created by shaking the end of the string in simple harmonic
motion. The shape of the wave can be described by y = f (x) and it is called sinusoidal if
the waveform is a sine curve. The shape remains the same but moves toward the right if
the wave function
 

y(x, t) = A sin (x − vt)
λ
The wave moves toward the left if
 

y(x, t) = A sin (x + vt)
λ
where v is the velocity of the wave and λ is the wavelength.
Chapter 15 Wave Motions 250

Let’s review some key quantities about wave motion.

• Wavelength λ: It is the minimum distance between any two identical points on a


wave, e.g. the distance between successive crests and the distance between successive
troughs. The SI unit of wavelength is meter.

• Frequency f : It represents the number of oscillations per unit time.

• Velocity v: It is the distance that the wave travels per unit time, where v = f λ.

• Wave number k: It states the radian per unit distance, i.e. k = . The SI unit of
λ
k is radian per meter.

• Angular frequency ω: It is the measure of how many radians the waves change in
one second, where ω = 2 πf . It is labeled as ω.
1 2π
• Period T : The time for one oscillation, it is labeled as T , where T = = .
f ω
There is a relation which links the angular frequency ω and the wave number k:
ω
v=
k
Generally, a phase angle is introduced to the wave functions

y(x, t) = A sin(kx − ωt + φ) (toward the right)

The wave moves toward the left if

y(x, t) = A sin(kx + ωt + φ) (toward the left)

We should note that v is different from vy . For the wave traveling toward the right, we have
∂y
vy = = −ym ω cos(kx − ωt + φ), which is the velocity of the particle in the medium.
∂t
∂ 2y
The acceleration of the particle is given by ay = 2 = −ym ω 2 sin(kx − ωt + φ) = −ω 2 y
∂t
because the particle oscillates about its equilibrium position and it performs SHM.

Example
A sinusoidal wave with speed u = 0.08 m/s has a waveform at time t = 0 sec as shown
in the figure. (a) Write down the wave equation. (b) Sketch the waveform at t = T /8,
where T is the period.
Chapter 15 Wave Motions 251

Solution
(a) The sinusoidal wave has amplitude 0.04 m and a wavelength λ = 0.4 m. So, the wave
2π 2π
number k = = = 5 π m−1 . The angular speed ω = uk = (0.08 ms−1 ) (5 πm−1 ) =
λ 0.4 m
2 π −1
s .
5

The wave moves toward the right ensures the argument of the trigonometric function has
a form (kx − ωt). Hence, the wave equation can be written as

y(x, t) = ym sin(kx − ωt + φ) ,

where φ is the phase angle in the expression and it becomes


 

y(x, t) = 0.04 sin 5 π (0.1) − t+φ
5
Plugging into the
 conditions when t  = 0 sec, x = 0.1 m and y = 0.04 m, we have
2π π 
0.04 = 0.04 sin 5 π (0.1) − (0) + φ , that is sin + φ = 1 and φ = 0. The wave
 5 2

equation is y = 0.04 sin t .
5
Chapter 15 Wave Motions 252

2π T 5
(b) The period T = = 5 sec. At time t = , (i.e. t = sec), the wave has y-
ω   8 8
2π 5  π
displacement y = 0.04 sin 5 πx − ( ) = 0.04 sin 5πx − . The graph of it is a
5 8 4
λ
right shift of the the initial waveform by = 0.05 m.
8

15.3 Wave speed on a stretched string


Consider a small string element within the pulse, of length δs, forming an arc of a circle
of radius R and subtending an angle 2 θ at the center of that circle. Here, we focus on
flat curve for which θ is very small and sin θ ≈ θ. The pulse travels with a constant speed
v to the right. But, the mass element is actually not moving along the wave according to
the definition of transverse wave. Imagine an observer moving toward the right with the
same speed as the pulse, the pulse seems not moving with respect to him. Nevertheless,
the mass element travels toward the left with speed v with respect to him.

Now, we apply Newton’s second law to discuss the motion of the mass element with
respect to this inertia observer. A tensive force F~ pulls tangentially on this element at
each end. The sum of the horizontal components balance each other and the sum of
vertical components is given by
 
X δs
Fy = 2 F sin θ ≈ 2 F θ = F
R
Chapter 15 Wave Motions 253

Then
v2
 
δs
F = (δm) ay = (µ δs) ,
R R
where ay = v 2 /R and δm = µ δs. Simplifying the expression, we get
s
F
v=
µ

15.4 Derivation of wave equation


Consider the transverse wave on a stretched string. The y-component of the net force
acting on the segment δx:
X
Fy = F sin θ2 − F sin θ1 ≈ F (tan θ2 − tan θ1 )
 
∂y
Since tan θi = So
∂x xi
 
X ∂y
Fy = F δ (15.1)
∂x

By Newton’s second law:

∂ 2y
X  
Fy = δmay = µδs (µ = linear linear density of string)
∂t2

Combining with equation (15.1)


     
∂y ∂y ∂y
δ δ δ
µ ∂ 2y
 
∂x ∂x ∂x
= (Because ≈ )
δx F ∂t2 δs δx
∂ 2y µ ∂ 2y
 
=
∂x2 F ∂t2
Chapter 15 Wave Motions 254

s
∂ 2y ∂ 2y
 
F 1
As v = , the wave equation becomes 2
= 2 .
µ ∂x v ∂t2

Example
∂ 2y 1 ∂ 2y
Show that y(x, t) = f (x ± vt) is a solution to the wave equation = .
∂x2 v 2 ∂t2
Solution
Let z = z(x, t) = x ± vt and y = f (z).

∂y df ∂z df
= =
∂x dz 
∂x  dz
∂ 2y d df ∂z d2 f
2
= =
∂x dz dz ∂x dz 2
On the other hand,
∂y df ∂z df
= = ±v
∂t dz 
∂t  dz
2 2
∂ y d 2 df ∂z 2d f
= ±v = v
∂t2 dz dz ∂t dz 2

∂ 2f 1 ∂ 2f
So, = and y(x, t) = f (x ± vt) satisfy the wave equation.
∂x2 v 2 ∂t2

15.5 The rate of energy transmission


The kinetic energy dK associated with a string element of mass dm is given by
1
dK = (dm) u2 ,
2
where u is the transverse speed of the oscillating string element. Differentiating y with
respect to t while keeping x as a constant, we get
∂y
u= = −ω ym cos(kx − ωt)
∂t
Using this relation and putting dm = µ dx, we have
1
dK = (µ dx) (−ω ym )2 cos2 (kx − ωt)
2
dK 1
The rate of energy transmission is = µ v ω 2 ym2
cos2 (kx − ωt). Then, the average
dt 2
rate at which the KE is transported is
 
dK 1
= µ v ω 2 ym 2
dt avg 4
Chapter 15 Wave Motions 255

1
because the average value of cos2 (kx − ωt) over one period is . The elastic potential
2
energy is also carried along with the wave, and the rate is the same as that of KE. So,
the total power transmitted is
1
Pavg = µ v ω 2 ym
2
2
Example
A stretched string has linear density µ = 525 g/m and is under tension F = 45 N. A
sinusoidal wave is sent with frequency f = 12 Hz and amplitude ym = 8.5 mm along the
string from one end. At what average rate does the wave transport energy?

Solution
We know that ω = 2 πf = (2 π) (120 Hz) = 754 rad/s and
s s
F 45 N
v= = = 9.26 m/s.
µ 0.525 kg/m
The average power transmitted along the string is
1
Pave = µv ω 2 ym
2
2
1
= (0.525 kg/m) (9.26 m/s) (754 rad/s)2 (0.0085 m)2
2
≈ 100 W

15.6 Reflection and transmission


When the travelling wave meets a fixed end (left figure) or a heavier string (right figure),
the reflected wave is inverted. In the right figure, the transmitted wave has lower speed
compared with the incident wave.
Chapter 15 Wave Motions 256

When the travelling wave meets a free end (left figure) or a light string (right figure),
the reflected wave is not inverted. In right figure, the transmitted wave has higher speed
compared with the incident wave.

Denote the heights of the incident, transmitted and reflected pulses as hin , ht , and hr ,
respectively. The reflection coefficient r is the height of the reflected pulse divided by
the height of the incident pulse, and the transmission coefficient t is the height of the
transmitted pulse divided by the height of the incident pulse. More precisely, we refer to
the displacement of the pulse instead of the height of them, because hr could be negative.

hr ht
That is, r = and t = . The expressions for r and t, quoted without proofs, are
hin hin
v2 − v1 2 v2
r= and t= ,
v2 + v1 v2 + v1
where v1 and v2 are the speeds of the pulses at different regions.
Chapter 15 Wave Motions 257

Note that t is never negative and that r is negative if v2 < v1 . This implies the transmit-
ted pulse is never inverted and the reflected pulse is inverted if v2 < v1 . One can also see
that t = 1 + r so ht = hin + hr . If hin is positive, hr could be positive or negative.

Example
Two wires of different linear mass densities are soldered together end-to-end and then
stretched under a tension FT The wave speed in the first wire is twice that in the second.
A harmonic wave traveling in the first wire is incident on the junction of the wires.

(a) If the amplitude of the incident wave is A, what are the amplitudes of the reflected
and transmitted waves?

(b) What is the ratio µ2 /µ1 of the mass densities of the wires?

(c) What fraction of the incident average power is reflected at the junction and what
fraction is transmitted?

Solution
v2 − v1 2 v2
(a) Knowing that r = and t = , we have
v2 + v1 v2 + v1
v2 − 2 v2 1 2 v2 2
r= =− and t= =
v2 + 2 v2 3 v2 + 2 v2 3
Let the reflected amplitude and transmitted amplitudes be Ar = rA and At = tA respec-
1 2
tively. Then, we get Ar = − A and At = A.
3 3
FT FT
(b) Since v12 = and v22 = , we have
µ1 µ2
FT FT
µ1 = and µ2 = .
v12 v22

µ2 v2 (2 v2 )2
Hence, = 12 = = 4.
µ1 v2 v22
Chapter 15 Wave Motions 258

(c) Knowing that


1
Pin, avg = µ1 v1 ω 2 A2
2
 2
1 2 2 1 2 1 1
Pr, avg = µ1 v1 ω Ar = µ1 v1 ω − A = µ1 v1 ω 2 A2
2 2 3 18
 2
1 2 2 1 2 2 2
Pt, avg = µ2 v2 ω At = µ2 v2 ω A = µ2 v2 ω 2 A2
2 2 3 9
we have
1
Pr, avg µ1 v1 ω 2 A2 1
= 18 =
Pin, avg 1 9
µ1 v1 ω 2 A2
2
2
Pt, avg µ2 v2 ω 2 A2 4
 2
v1 v2 4 v1

4 2 v2

8
= 9 = · · = · = =
Pin, avg 1 9 v2 v1 9 v2 9 v2 9
µ1 v1 ω 2 A2
2

15.7 Doppler effect


Whenever there is a relative motion between the sound source and the observer, there is
a change of frequency to be received by the observer. There are two cases to study.

(I) Moving observer


A sound wave is emitted from a stationary source, The wave travels in air with velocity
v, having frequency f and wavelength λ, where v = f λ. For an observer moving toward
the source with a speed u, the sound seems to have a higher speed, e.g. v + u. As a result,
more wavefronts move past the observer in a given time than if the observer had been at
rest. To the observer, the sound has a frequency, f 0 , that is higher than the frequency of
the source.

v0
 
0 v+u v+u
f = = = f >f
λ λ v
Chapter 15 Wave Motions 259

If the observer moves away from the source, the sound seems to have a lower speed, e.g.
v − u. As a result, less wavefronts move past the observer in a given time than if the
observer had been at rest. To the observer, the sound has a frequency, f 0 , that is lower
than the frequency of the source.

v0
 
0 v−u v−u
f = = = f <f
λ λ v

Combining the two results, we have


 
0 v±u
f = f,
v

where the plus sign is used when the observer moves toward the source and the minus
sign is used when the observer moves away from the source.

(II) Moving source


When the source moves, the Doppler effect is not due to the sound wave appearing to have
a higher or lower speed, but a variation in the magnitude of wavelength. Consider, then,
a source moving toward the observer with speed u, the sound waves have one compression
and then another compression in time T , where T = 1/f . The wave travels a distance
vT , and the source travels a distance uT .

As a result, the new wavelength of the sound waves is vT − uT = (v − u) T , which is


shorter than that when the source is at rest. The new frequency of the sound waves is
obtained by v = f 0 λ0 , that is
 
0 v v
f = = f >f.
(v − u) T v−u
Chapter 15 Wave Motions 260

When the source reverses its direction, the new wavelength of the sound waves, vT +uT =
(v + u) T , is longer than that when the source is at rest.
 
0 v v
f = = f <f,
(v + u) T v+u

Combining the two results, we have


 
0 v
f = f,
v∓u

where the minus sign is used when the source moves toward the observer and the plus
sign is used when the source moves away from the observer.

General cases
Doppler effect for both moving source and observer is concluded in a simple formula:
 
0 v ± uo
f = f
v ∓ us

Example
The frequency of a car horn is 400 Hz. If the horn is honked as the car moves with a speed
us = 34 m/s toward a stationary observer. Take the speed of sound in air to be 343 m/s.

(a) Find the frequency received.

(b) Find the frequency received if the car is stationary as the horn is honked and the
observer moves with a speed uo = 5 m/s toward the car.

Solution    
0 v 343
(a) The frequency received is f = f= (400 Hz) = 440 Hz.
v − us 343 − 34
   
0 v + uo 343 + 5
(b) The frequency received is f = f= (400 Hz) = 406 Hz.
v 343
Chapter 15 Wave Motions 261

15.8 Shock waves


When the speed vS of a source exceeds the wave speed v, an envelope of wave fronts in
the form of a cone appears. It is the shock wave. The apex half-angle θ is given by
vt v
sin θ = =
vs t vs
The angle is called the Mach angle and the value of the above ratio is the Mach number.
In the figure, a wave source of speed vS is moving from S0 to the right. After time t, the
wave source travels a distance vs t from S0 and the wave generated at S0 has propagated a
distance vt from S0 , where v is the propagation speed of the wave in the medium. vS > v.

15.9 Superposition
Superposition principle:
If two or more traveling waves are moving through a medium, the resultant value of the
wave function at any point is the algebraic sum of the values of the wave functions of the
individual waves. Waves that obey this principle are called linear waves. These waves are
generally characterized by having amplitudes much smaller than their wavelengths.
Chapter 15 Wave Motions 262

Let’s apply the principle to two sinusoidal waves traveling in the same direction. They
have the same amplitude, frequency, and wavelength but differ in phase.
Chapter 15 Wave Motions 263

y1 = A sin(kx − ωt) y2 = A sin(kx − ωt + φ)

Making use the trigonometric identity


   
X +Y X −Y
sin X + sin Y = 2 sin cos ,
2 2

the resultant wave function is

y = y1 + y2
= A sin(kx − ωt) + A sin(kx − ωt + φ)
   
φ φ
= 2 A cos sin kx − ωt +
2 2

If φ = 0, we have cos(φ/2) = 1. The individual waves have the same y at everywhere.


The two waves are in phase and the resultant wave has amplitude 2A. In general, con-
structive interference occurs when φ = 0, 2 π, 4 π, · · · , radians. Or, we can say it
occurs when φ is an even multiple of π. In terms of path difference, it is 0, λ, 2 λ, . . .

If φ = π, we have cos(φ/2) = 0. The two waves are out of phase and the resultant
wave has zero amplitude everywhere. In general, destructive interference occurs when
φ = π, 3 π, 5 π, · · · , radians. Or, we can say it occurs when φ is an odd multiple of π.
In terms of path difference, it is λ/2, 3 λ/2, 5 λ/2, · · ·

Example
Two identical loudspeakers placed 3.00 m apart are driven by the same oscillator. A lis-
tener is originally at point O, located 8.00 m from the center of the line connecting the two
speakers. The listener then moves to point P , which is a perpendicular distance 0.350 m
from O, and she experiences the first minimum in sound intensity. What is the frequency
of the oscillator?
Chapter 15 Wave Motions 264

Solution
The first minimum occurs when the two waves reaching the listener at point P are 180◦
out of phase. in other words, when their path difference equals λ/2. From the shaded
triangles, we have
p
r1 = (8.00 m)2 + (1.15 m)2 = 8.08 m
p
r2 = (8.00 m)2 + (1.85 m)2 = 8.21 m

Hence, we obtain the path difference ∆r = r2 − r1 = 8.21 m − 8.08 m = 0.13 m. Because


this path difference must equal λ/2, we get λ = 0.26 m. Using f = v/λ, we have

343 ms−1
f= = 1.3 kHz
0.26 m

15.10 Standing wave

Consider two traveling waves having the same amplitude and wavelength but traveling in
opposite directions in the same medium.

y1 = A sin(kx − ωt) y2 = A sin(kx + ωt)

Adding these two functions, we get

y = y1 + y2
= A sin(kx − ωt) + A sin(kx + ωt)
= (2 A sin kx) cos ωt

where y is the wave function of a standing wave.


Chapter 15 Wave Motions 265

The amplitude of the simple harmonic motion of an element of the medium is always zero
if sin kx = 0. That is, when
kx = 0, π, 2 π, 3 π, . . .

Using the expression k = 2 π/λ, we get


λ 3λ nλ
x = 0, , λ, ,··· = n = 0, 1, 2, 3, . . .
2 2 2
These points of zero amplitude are called nodes.

The amplitude of the simple harmonic motion of an element of the medium is always the
greatest if sin kx = ±1. That is, when
π 3π 5π
kx = , , , ...
2 2 2
Using the expression k = 2 π/λ again, we get
λ 3λ 5λ nλ
x= , , ,··· = n = 1, 3, 5, . . .
4 4 4 4
These points of greatest amplitude are called antinodes.

Some points to note:

• The distance between adjacent nodes is equal to λ/2.

• The distance between adjacent antinodes is equal to λ/2.

• The distance between a node and an adjacent antinode is equal to λ/4.

Example
Two waves traveling in opposite directions produce a standing wave. The individual wave
functions are

y1 = 4.0 sin(3.0 x − 2.0 t) y2 = 4.0 sin(3.0 x + 2.0 t)

where x is measured in centimeters and y is measured in seconds.

(a) Find the amplitude of the simple harmonic motion of the element of the medium
located at x = 2.3 cm.

(b) Find also the positions of the nodes and antinodes if one end of the string is at x = 0.
Chapter 15 Wave Motions 266

Solution
(a) The resultant wave is y = y1 + y2 = (2 A sin kx) cos ωt = (8.0 sin 3.0 x) cos 2.0 t.
Putting x = 2.3 cm, we get y = (8.0 sin 6.9) cos 2.0 t. Hence, y = 4.6 cos 2.0 t. The ampli-
tude of the resultant wave at x = 2.3 cm is 4.6 cm.

2π 2π 2π nλ nπ
(b) Since k = , we have λ = = . The nodes are located at x = = ,
λ k 3.0 rad 2 3.0 rad
nλ nπ
where n = 0, 1, 2, . . . . The antinodes are located at x = = , where
4 6.0 rad
n = 1, 3, 5, . . . .

Standing waves in a string


The boundary condition results in the string having a number of discrete natural patterns
of oscillation, called normal modes.

v v
Fundamental frequency (first harmonic): λ1 = 2 L, so f1 = =
λ1 2L
v v
Second harmonic: λ2 = L, so f2 = =
λ2 L
2L v 3v
Third harmonic: λ3 = , so f3 = =
3 λ3 2L

In general, the various normal modes for a string of length L fixed at both ends are
2L v nv
λn = , so fn = = , where n = 1, 2, 3, . . .
n λn 2L
Hence, we have
fn = n f1
s
T
Using the relation v = , we have
µ
s
n T
fn =
2L µ
Chapter 15 Wave Motions 267

Standing waves in air columns

15.11 Beats
Beating is the periodic variation in amplitude at a given point due to the superposition
of two waves having slightly different frequencies. Let’s consider two sound waves y1 of
ω1 and y2 of ω2 , where ω1 is close to ω2 . At x = 0,

y1 = A cos(ω1 t) y2 = A cos(ω2 t)

Putting ω = 2 πf , we have

y1 = A cos(2 πf1 t) y2 = A cos(2 πf2 t)


Chapter 15 Wave Motions 268

Making use the trigonometric identity


   
X +Y X −Y
cos X + cos Y = 2 cos cos ,
2 2
then we have

y = y1 + y2
= A cos(2 πf1 t) + A cos(2 πf2 t)
      
f1 − f2 f1 + f2
= 2 A cos 2 π t cos 2 π t
2 2

 
f1 − f2
yenvelope = 2 A cos 2 π t
2
A maximum in the amplitude of the resultant wave is detected whenever
 
f1 − f2
cos 2 π t = ±1
2
As there are two ”maxima” between one period, so the beat frequency is

fbeat = |f1 − f2 |

Example
When a 440-Hz tuning fork is struck simultaneously with the a string of a slightly out-
of-tone guitar, 3.00 beats per second are heard. The guitar string is tightened a little
to increase its frequency. As the guitar string is slowly tightened, you hear the beat fre-
quency slowly increase. What was the initial frequency of the guitar string?

Solution
Because 3.00 beats per second were heard initially, the initial frequency of the guitar
string was either 437 Hz or 443 Hz. The greater the difference between the frequency of
Chapter 15 Wave Motions 269

the string and the frequency of the tuning fork, the greater the beat frequency. The
frequency of the string increases with an increase in the tension. So, the initial frequency
of the guitar string is 443 Hz.

Figure courtesy of the following books:


(1) P.A Tipler and G. Mosca: Physics for Scientists and Engineers (Freeman, 2008, 6th edition)
(2) R. Serway and J.W. Jewett: Physics for Scientists and Engineers (Thomson, 2014, 9th edition)
(3) J.S. Walker: Physics (Pearson, 2014, 5th edition)

S-ar putea să vă placă și